DHA-US1114 Concussion Training for Service Members

Which of the following are common causes of concussion?

  • Correct answers: Falls, Vehicle crashes, A direct blow to the head or the head striking an object, Being near a blast.

  1. What is true about concussions?

    • Correct answer: With early diagnosis and treatment most Service Members can return to duty quickly following a concussion.

  2. Four Airmen are involved in a football collision during downtime. They all received a blow to the head. Which of the Airmen should receive a medical examination?

    • Correct answer: All of the Airmen because they were all involved in a potentially concussive event.

  3. Which of the following are signs that a person may have sustained a concussion?

    • Correct answers: Blank or vacant look, Slow to get up after a direct or indirect blow to the head, Disorientation, Lying motionless on the ground.

  4. Which of the following are symptoms of concussion?

    • Correct answers: Nausea/vomiting, Ringing in the ears, Difficulty concentrating, Dizziness, Visual disturbances, Balance difficulty, Irritability.

  5. The DVBIC (TBI Center of Excellence) website is the best place to locate information and resources related to concussion.

    • Correct answer: True.

  6. Which of the following are associated with sustaining multiple concussions?

    • Correct answers: Recovery may take longer for a second or third concussion, You will be at greater risk of sustaining another concussion after the first concussion, You may experience more severe symptoms with multiple concussions.

  7. Which of the following can happen when a Service Member does not get evaluated and treated for a concussion?

    • Correct answers: The Service Member is putting himself/herself at risk for persistent symptoms, The Service Member may be delaying recovery time, The Service Member is putting the mission at risk.

  8. Which of the following statements about concussion is the most accurate?

    • Correct answer: A concussion may occur with or without loss of consciousness.

  9. Your buddy Staff Sergeant Grayson is involved in a potentially concussive event. What should you do in this situation?

    • Correct answer: Encourage Staff Sergeant Grayson to go and get a medical evaluation right away.

  10. Master Sergeant Newton collided with another Service Member during a football game and was diagnosed with a concussion. Which of the following activities should he AVOID for the next 24 hours?

    • Correct answers: Drinking alcohol, Catching up on reports, Light physical activities, Doing crossword puzzles, Downhill skiing.

  11. Airman Johnson collided with another Service Member during a hockey game and was diagnosed with a concussion. This is the second concussion that Airman Johnson has sustained in a year. When should Airman Johnson return to duty?

    • Correct answer: He should return to duty when his medical provider clears him to return to duty.

  12. In which of the following ways can a concussion impact your performance?

    • Correct answers: You may feel irritable, You may make mistakes that you wouldn't usually make.

Mental Health: E5 and Above CBT

Q. When should you apply the 6 steps of iCOVER?

A - During the event

Q. As a leader, you should be aware of changes in which of the following COSR categories?

A - All the Above

Q. When should you apply the 8 steps of Psychologic First Aid?

A - After the event

Pre-Deployment Financial Readiness Training

A(n) helps manage money and plan for financial goals and prepare for financial emergencies.

A: Spending Plan

Select the steps involved in creating and using a spending plan.

A: All the above

When being deployed, the most common types of special entitlements are ____

A: Family separation, Hazardous duty, hardship pay

____ is extra income typically paid to service members for the performance of certain duties or for service in certain locations

A: Special Pay and Entitlements

What are some ways to build your savings account?

A: Automatic Transfers, Allotments

The ___ is a DOD-provided benefit that offers an annual return rate of 10% compounded monthly and paid quarterly up to $10,000 put into it

A: Savings Deposit Program

When Experiencing financial challenges, ____ offers confidential counseling, education information, and referrals to other resources, if necessary

A: Financial Counselors

Someone at risk for financial problems can display which warning signs?

A: All the above

Mishandling debt or credit can affect your

A; Security Clearnace

Integrated Defense

Q: What should you NEVER do when encountering a UAS?

Answer: All

Approach an airborn or grounded UAS

Pick up a UAS on the ground

Group together during an attack unless under cover

Ignore UAS threat calls over the radio or loadspeaker

Q: What actions should you take when recognizing signs of possible observation or attack from a UAS? (Select all that apply)

A: All

Direct attention

Report the incident

Observe the UAS

Notice features

Execute appropriate action

Q: Which techniques are used to assist in identifying targets during daylight? (Select all that apply)

A: A combination of facial recognition, behavioral, and uniform/equipment identification

Q: When conducting a challenge to determine the identity of an individual approaching your DFP, what should you command just loud enough for them to hear?

A: “HALT”

Q: ______ is an area that a weapon can cover effectively from a given position

A: Field of Fire

Q: A _____ is a list of recorded information within your field of fire that you can effectively engage with a weapon from a static position that can be used as a reference

A: Range Card

Q: Personal entering an Entry Control Point (ECP) should be properly identified before allowing entry. Which items are commonly used to verify personnel? (Select all that apply)

A: Visitor passes

A: Entry Authorization Lists (EALs)

A: Identification cards (Common Access Cards, Dependent, Civilian, Contractor, etc.)

Q: _____ and _____ barriers slow down and manipulate movement of people and vehicles as they approach the ECP.

A: Active and Passive

2024 Cyber Awareness Challenge

Which of the following statements about Protected Health Information (PHI) is false?

It is a type of Controlled Unclassified Information (CUI)

Which of the following is an example of a strong password?

d+Uf_4RimUz

What is the goal of an insider Threat Program?

Deter, detect, mitigate the risks associated with insider threats

Which of the following is a best practice for using government email?

Do not send mass e-mails

Which of the following is NOT a best practice for protecting your home wireless network for telework?

Use your routers pre-set service set identifier (SSID) and password

Beth taps her phone at a payment terminal to pay for a purchase. Does this pose a security risk?

Yes, there is a risk that the signal could be intercepted and altered.

Which of the following contributes to your online identity?

All of these

How can you prevent virus and malicious code?

Scan all external files before uploading to your computer

When is the safest time to post on social media about your vacation plans?

After the trip

Which of the following is a best practice for managing connection requests on social networking sites?

Validate connection requests from people with whom you share mutual connections.

What conditions are necessary to be granted access to sensitive compartmented information (SCI)?

Top Secret clearance and indoctrination into the SCI program.

After a classified document is leaked online, it makes national headlines. Which of the following statements is true of the leaked information that is now accessible by the public?

You should still treat it as classified even though it has been compromised.

How can you protect your home computer?

Turn on the password feature.

Which of the following is not appropriate use for your common access card?

Using it as a photo identification with a commercial entity

Which of the following is not a best practice for protecting data on a mobile device?

Disable automatic screen, locking after a period of inactivity

Mabel is a government employee who needs to share a document containing contractor proprietary information with his supervisor. Which of the following describes the most appropriate way from able to do this?

Encrypt it and send it via digitally signed Government e-mail.

Which of the following is permitted when using an unclassified laptop within a collateral classified space?

A Government-issued wired headset with microphone

Which of the following is a best practice when browsing the internet?

Only accept cookies from reputable, trusted websites

Which of the following is an example of behavior that you should report?

Taking sensitive information home for telework without authorization.

Which of the following uses of removable media is appropriate?

Encrypting data stored on removable media.

Which of these is not a potential indicator that your device may be under a malicious code attack?

A notification for a system update that has been publicized

Terri sees a post on her social media feed that says there is a smoke billowing from the Pentagon. The post includes a video that shows smoke billowing from a building that is not readily identifiable as the Pentagon Jerry is not familiar with the source of the post which of the following describes what Terry has likely seen?

This is probably a post designed to attract Terri's attention to click on a link and steal her information

Which of the following is permitted within a Sensitive Compartmented Information Facility (SCIF)?

An authorized Government-owned Portable Electronic Device (PED)

You receive an email with a link to schedule a time to update software on your government furnished laptop. Your IT department has not scheduled software updates like this in the past and has not announced this software update. The e-mail is not digitally signed. What action should you take?

Report the e-mail to your security POC or help desk.

Which of the following is true of transmitting or transporting SCI?

Printed SCI must be retrieved promptly from the printer

Your meeting notes are unclassified. This means that your notes:

Do not have the potential to damage national security.

What type of information does this personnel roster represent?

Controlled Unclassified Information (CUI)

When e-mailing this personnel roster, which of the following should you do?

All 3 – Encrypt, Digitally, Use

Confined Spaces

To safely enter a confined space, the entrant must familiarize themselves with the ___ as well as keeping in direct close contact with the attendant.

-Work to be performed

What is the most important part when performing a confined space entry?

- Communication

What is the meat and potatoes for a written Confined Space Program?

-Confined Space Master Entry Plan

The most common toxic chemicals in confirned spaces are __ and Carbon Dioxide.

- Hydrogen Sulfide

Before any Confined Space operation can begin supervisors must estabilish a __.

- Written Confined Space Program

T o False: Before any worker can enter a confined spac, they must be briefed on al procedures, safeguards, and emergency egress procedure.

-True

What must be referenced to find the required Personal Protective Equipment (PPE) for confined space entry?

-Master Entry Plan

According to the Mine Safety Appliance (MSA) guidelines, atmospheric testing devices require calibration every __.

- 6 Months

To enter a permit-required confined space, you must have an authorization in place that will be outlined in your ___.

-Master Entry Plan (MEP)

To safely enter a confined space, the entrant must familiarize themselves with the __ as well as keeping in direct close contact with the attendant.

-Master Entry Plan

Confined space operation communication may be __.

-verbal and written

Which hazard control method is where you wuold have permanently installed equipment for confined space operations?

-Engineering Controls

The confined space __ must ensure that all the required Personal Protective Equipment (PPE) is available and in serviceable condition.

-Supervisor

Who is responsible for completeing the Confined Space Entry Permit?

-Supervisor

Which is the best way to remove hazards from a confined space?

-Engineering Controls

To get an authorization, the worker must notify their __ to start the process.

- supervisor

T o F: The Emergency Rescue Teams must be Fire Department personnel only.

- False

What is required by regulation when a confined space is designate as permit-required and could be inadvertently entered?

-Signs

What agency is not an approving authority for the Master Entry Plan?

-Wing Commander

T o F: Safe oxygen content levels are between 19.5 and 23.5 percent.

- True

A __ confined space is dangerous to enter due to a hazardous atmosphere or a potention engulfment.

-permit-required

T o F: Liquids or solids can flow into the confined space causing drowning, suffocation, burns and other injuries.

-True

When may the attendent leave during a confined space entry?

-When properly relieved by a qualified attendant

The supervisor will ensure all that the workers invovled have the proper training and ___.

-Certificates

Barriers and __ can be used to prevent entry of unauthorized personnel.

-Signs

What should always be up to date on your Atmospheric Testing Meter?

-Calibration

Completed permits will be kept on file for __ year(s).

-1 year.

T o F: The attendant can be authroized to perform multiple duties while attending a confined space entry if it does not interfere with their primary duties.

-True

Who is the last person to sign the permite to close it out?

-Entry Supervisor

T o F: Excessive heat is not a hazard to worry about in Confined Space Operations.

-False

The supervisor will ensure that all the workers involved have the proper training and __.

-Certifications

What is the least preferred method to control confined space hazards?

-Personal Protective Equipment

What duty(s) should the standby worker be trained in?

-All positions

Biol 202 Unc

Question: Transcription

Answer: The process by which RNA Pol enzymes and other transcriptional proteins and enzymes use the template strand of DNA to synthesize a complementary RNA strand. DNA is converted into RNA.

==================================================

Question: Translation

Answer: The process by which mRNA is used to direct protein synthesis. RNA is converted into protein.

==================================================

Question: mRNA (messenger)

Answer: A form of RNA transcribed from a gene and subsequently translated to produce a polypeptide or a protein; responsible for protein synthesis. Conveys the genetic message of DNA to ribosomes for translation.

==================================================

Question: tRNA (transfer)

Answer: Not translated. Encoded in the genome. Each different one binds a specific amino acid to the ribosome, and then deposits its amino acid to the mRNA for inclusion in the protein chain.

==================================================

Question: rRNA (ribosomal)

Answer: Not translated. Combines with proteins to form the ribosome, which is responsible for translation. Sometimes interacts with mRNA to initiate translation.

==================================================

Question: snRNA (small nuclear)

Answer: Found in the nucleus of eukaryotic cells. Participates in mRNA processing. Some can unite with nuclear proteins to form the complexes responsible for intron removal.

==================================================

Question: miRNA (micro)

Answer: Active in plant and animal cells but not bacteria. Regulate mRNA protein production post-transcription through RNA interference.

==================================================

Question: siRNA (small interfering)

Answer: specialized RNA that helps protect plant and animal genomes from virus production and spread of transposable elements within the genome.

==================================================

Question: ribozymes

Answer: Catalytically active RNA that can activate cellular reactions such as the removal of introns by self-splicing.

==================================================

Question: promoter sequence

Answer: Upstream of the coding sequence, immediately 5' to the start of transcription. Regulates transcription by controlling access of RNA polymerase to the gene.

==================================================

Question: terminator sequence

Answer: located immediately downstream - to the 3' end of the coding sequence. This region regulates the cessation of transcription.

==================================================

Question: holoenzyme

Answer: an intact complex with full enzymic capacity (one example is RNA Pol)

core enzyme + sigma subunit

==================================================

Question: core enzyme

Answer: The component in bacterial RNA Pol that actively carries out transcription.

==================================================

Question: Sigma subunit

Answer: the protein to which the RNA Pol binds. Directs the RNA polymerase core enzymes to promoters.

==================================================

Question: Promoter

Answer: double-stranded DNA sequence that is the binding site for the RNA Pol

==================================================

Question: -10 and -35 Consensus Sequences

Answer: What are the two components of prokaryotic promoters?

==================================================

Question: Closed promoter complex

Answer: The point in bacterial transcription initiation when the RNA Pol loosely binds the promoter (the -10 to -35 region)

==================================================

Question: consensus sequences

Answer: short regions of highly similar DNA located in the same position relative to the start of transcription in different gene promoters.

==================================================

Question: Open Promoter Sequence

Answer: the stage in transcription initiation in which the RNA Pol is bound and a small section of the DNA opens up to allow transcription from the template strand.

==================================================

Question: intrinsic termination

Answer: In bacterial termination; DNA-sequence dependent termination mechanism. Inverted repeats cause the formation of a 3' stem-loop structure followed by a string of U's. Most common.

==================================================

Question: rho-dependant termiation

Answer: Bacterial termination in which you must have a rho protein.

==================================================

Question: Template

Answer: Which strand is transcribed?

==================================================

Question: Introns

Answer: ...

==================================================

Question: Exons

Answer: ...

==================================================

Question: Chromatin

Answer: The complex of nucleic acids and proteins that make up chromosomes

==================================================

Question: RNA Pol I (euk)

Answer: Transcribes 3 rRNA genes

==================================================

Question: RNA Pol II (euk)

Answer: Transcribes mRNAs that encode polypeptides; most snRNA genes

==================================================

Question: RNA Pol III (euk)

Answer: Transcribes all tRNA genes as well as one snRNA.

==================================================

Question: RNA Pol II & III (euk)

Answer: responsible for miRNA and siena synthesis

==================================================

Question: Band Shift Assay

Answer: Used to locate promoters. If consensus sequences are in the DNA fragment, proteins bind to them. A slower migration means that the proteins have bound, and that the promoter consensus sequence is present.

==================================================

Question: DNA Footprint Protection Assay

Answer: You run 2 groups - 1 experimental (transcription protein added) and 1 control (no protein added.) In the place where the bands do NOT form with the experimental sample this is where the promoter region is because the proteins have bound.

==================================================

Question: TATA box

Answer: Most common eukaryotic promoter consensus sequence; located around position -25. Most strongly conserved promoter element in eukaryotes.

==================================================

Question: Transcription factors (TF)

Answer: Proteins that help RNA Pol II recognize and bind to promoter consensus sequences in eukaryotes. These proteins bind to promoter regulatory sequences and influence transcription initiation by interacting with the RNA pol.

==================================================

Question: TFIID

Answer: protein that binds to the TATA box

==================================================

Question: enhancers

Answer: DNA regulating sequences that increase transcription of specific genes. They bind specific proteins that interact with the proteins at promoter sequences and up the transcription levels. Often upstream but possibly downstream of the gene they regulate. The proteins form a bridge that bends the DNA.

==================================================

Question: silencers

Answer: DNA regulating sequences that repress transcription of target genes. They bind TF's called repressor proteins, making the DNA bend but in this case reducing the transcription levels.

==================================================

Question: Signal transduction pathway

Answer: communicate the need for specific regulatory molecules such as TF's in transcription. A series of events that release a regulatory molecule in order to get what is needed.

==================================================

Question: 5' end capping, 3' polyadenylation, intron splicing

Answer: What are the three main ways to modify mRNA (from pre-mRNA to mature mRNA)?

==================================================

Question: 5' end capping

Answer: addition of a modified nucleotide to the 5' end of mRNA in order to modify it

==================================================

Question: 3' Polyadenylation

Answer: cleavage at the 3' end of mRNA and addition of a tail of multiple adenines in order to form the poly-A tail in order to modify mRNA

==================================================

Question: Intron splicing

Answer: RNA splicing to remove introns and ligate exons

==================================================

Question: mature mRNA

Answer: fully processed product of transcription that moves to the cytoplasm for translation.

==================================================

Question: pre mRNA

Answer: initial transcription product, must be modified for translation to occur

==================================================

Question: 5' splice site

Answer: located on the end of an intron; contains a consensus sequence with a GU located at the 5' end of the intron.

==================================================

Question: 3' splice site

Answer: located on the 3' end of an intron; consensus sequence of 11 nucleotides containing a pyrimidine-rich region and an AG at the 3' end.

==================================================

Question: branch point adenine

Answer: near the 3' end of an intron - joins with a guanine from the 5' splice site in order to form a lariat intron.

==================================================

Question: snRNPs

Answer: snRNA-protein subunits that make up the spliceosome. U1 - U6

==================================================

Question: spliceosome

Answer: A snRNA-protein complex that removes introns from pre-mRNA. Made up by 5 snRNPs.

==================================================

Question: lariat intron

Answer: the structure formed when the 5' end of an intron binds to the branch site

==================================================

Question: SR Proteins

Answer: Proteins that recruit spliceosome components to 3' and 5' splice sites in intron splicing. Bind to exonic splicing enhancers.

==================================================

Question: Exonic Splicing Enhancers (ESE's)

Answer: Bind with SR proteins in intron splicing to ensure that the splicing components bind to the 3' and 5' splice sites in intron splicing instead of to other splice sites not near exons.

==================================================

Question: C-terminal domain (CTD)

Answer: Plays an important role in the coupling of the steps in pre-mRNA processing by functioning as an assembly platform and regulator of the machinery needed for the process. Binding of processing proteins to this allows mRNA to be modified as it is transcribed.

==================================================

Question: N-terminal

Answer: amino terminal end of a polypeptide corresponding to the 5' end of mRNA

==================================================

Question: C-terminal

Answer: carboxyl terminal end of a polypeptide corresponding to the 3' end of mRNA

==================================================

Question: 5' UTR

Answer: located between 5' end of mRNA and the start codon. Doesn't undergo translation.

==================================================

Question: 3' UTR

Answer: located between 3' end of mRNA and the stop codon. Doesn't undergo translation.

==================================================

Question: P site (peptidyl)

Answer: site on the ribosome where AAs are joined by a peptide bond

==================================================

Question: A site (aminoacyl)

Answer: site on the ribosome at which incoming charged tRNAs match their anticodon sequence with mRNA codons

==================================================

Question: E site (exit)

Answer: site on the ribosome where an uncharged tRNA exits

==================================================

Question: Shine dalgarno sequence

Answer: In bacterial translation the consensus sequence at the 5' end of mRNA that orients the start codon on the ribosome.

==================================================

Question: Scanning

Answer: the process by which the small ribosomal subunit locates the start codon

==================================================

Question: fMet

Answer: AA that initiated bacterial translation

==================================================

Question: Kozak sequence

Answer: consensus sequence of eukaryotic mRNA that contains the start codon

==================================================

Question: release factors (RF)

Answer: molecules that bind mRNA stop codons and contribute to translation termination

==================================================

Question: inosine (I)

Answer: another nucleotide that can be substituted in in 3rd base wobble

==================================================

Question: tRNA synthetases

Answer: group of enzymes that catalyze the charging of tRNA and the attachment of the appropriate AA

==================================================

Question: codon

Answer: set of 3 nucleotides that codes for a specific AA

==================================================

Question: sickle cell disease (SCD)

Answer: A disease affecting the beta-globin gene that is part of hemoglobin protein

==================================================

Question: hemolobin

Answer: protein that transports oxygen in blood.

==================================================

Question: SNP (single nucleotide polymorphism)

Answer: A single base-pair difference in a genome seen by comparing different genome.

==================================================

Question: Restriction enzyme

Answer: DNA-digesting enzyme that cuts DNA at specific recognition sites and generates cleavage at it's restriction sequence.

==================================================

Question: Molecular probe

Answer: single stranded nucleic acids that bind to target molecules allowing for identification of a specific protein, DNA or RNA sequence.

==================================================

Question: Northern blotting

Answer: mRNA transfer from an electrophoresis gel to a permanent membrane or filter.

==================================================

Question: Southern blotting

Answer: DNA transfer from an electrophoresis gel to a permanent membrane or filter.

==================================================

Question: Western blotting

Answer: protein transfer from an electrophoresis gel to a permanent membrane or filter.

==================================================

Question: Transition mutation

Answer: purine replaces purine or pyrimidine replaces pyrimidine

==================================================

Question: Transversion mutation

Answer: purine replaces pyrimidine or vice versa

==================================================

Question: reverse mutation (reversion)

Answer: converts a mutant allele to a wild-type allele

==================================================

Question: spontaneous mutation

Answer: mutation that occurs due to random spontaneous changes in the nucleotide sequence

==================================================

Question: Deamination

Answer: loss of an amino group from a nucleotide base spontaneously. Cytosine gets replaced with uracil.

==================================================

Question: Depurination

Answer: A mutation that occurs when a purine base (A or G) is removed from the sugar-phosphate back bone; effectively stops replication. Usually fixed, though. Very common.

==================================================

Question: mutagen

Answer: An agent capable of damaging DNA and causing a mutation.

==================================================

Question: Frameshift

Answer: What types of mutations are caused by intercalating agents?

==================================================

Question: thymine dimer

Answer: the type of lesion formed on DNA due to UV exposure.

==================================================

Question: Ames Test

Answer: To determine what chemicals are hazardous to our health by increasing genetic mutation frequency. (To identify the rate of reversion mutations)

Run 2 parallel tests - a control and an experimental. The control liver enzyme is plated on a his- plate. A possible mutagen is added to the experimental test, and then it is also plated on a his- plate. Then you can compare the experimental to the control, and if there is more liver growing on the experimental you have a positive result, reversion. This means that the experimental was probably a mutagen.

==================================================

Question: Base excision repair

Answer: A segment of the damage strand is taken out and replaced by the activity of the DNA Pol and DNA ligase

==================================================

Question: Nucleotide excision repair

Answer: If you can't repair the modified nucleotide then you can exploit the double-stranded DNA structure and use the undamaged side as a template.Remove the damaged nucleotides and replace them using complementary base pairing.

==================================================

Question: DNA recombination repair

Answer: reestablish normal DNA by exchanging a damaged segment of DNA with a normal segment from another chromatid.

==================================================

Question: Translesion DNA Synthesis

Answer: allows DNA replication by these alternative polymerases across lesions that block DNA Pol III, the main replicating polymerase in E. Coli.

==================================================

Question: non homologous end joining

Answer: An error prone repair process that repairs double stranded breaks before DNA replication.

Protein complex binds DNA ends. Ends are trimmed; nucleotides lost. DNA ligase ligates the ends, forming and intact duplex.

==================================================

Question: constitutive transcription

Answer: bacterial transcription - continuous with no regulated control - continually form routine tasks

==================================================

Question: regulated transcription

Answer: bacterial transcription - regulated control - need agile and calibrated environmental responses

==================================================

Question: negative control

Answer: binding of a repressor protein to a regulatory bacterial DNA sequence prevents transcription of a gene or group of genes

==================================================

Question: positive control

Answer: binding of an activator protein to a regulatory bacterial DNA sequence stimulates transcription of a gene or group of genes

==================================================

Question: DNA-binding & allosteric

Answer: 2 functional domains of a repressor protein

==================================================

Question: Operons

Answer: clusters of genes undergoing coordinated transcription regulation by a shared regulatory region. Common in bacterial genomes. Share transcription control.

==================================================

Question: Lac z-

Answer: no functional beta-galactose

==================================================

Question: Lac Y-

Answer: no functional permease

==================================================

Question: Lac I-

Answer: can't bind to operator

==================================================

Question: Lac Is

Answer: Cant bind to inducer

==================================================

Question: low glucose level

Answer: High cAMP level

==================================================

Question: cis-acting regulatory sequences

Answer: regulate transcription of genes on the same chromosome

==================================================

Question: trans-acting regulatory proteins

Answer: can identify and bind to target sequences on ANY chromosome

==================================================

Question: locus control region (LCR)

Answer: enhancer elements that regulate the transcription of multiple gene packages in closely related genes

==================================================

Question: Insulator sequence

Answer: prevents enhancers from inappropriately activating nearby genes, located between enhancers and promoters to shield the promoter

==================================================

Question: Open promoter

Answer: promoters that reside in open chromatin, resulting in constitutive transcription. Have a nucleosome depleted region; no TATA box.

==================================================

Question: Covered promoters

Answer: Promoter in which nucleosomes are adjacent to the start site preventing transcription initiation. Nucleosomes must be displaced or removed.

==================================================

Question: SWI/SNF

Answer: Chromatin remodelers that open chromatin structure by displacing nucleosomes. Allow binding of TF to initiate transcription.

==================================================

Question: ISWI complex

Answer: Chromatin remodelers that control placement of nucleosomes into regions that makes the region transcriptionally silent

==================================================

Question: SWR1 Complex

Answer: Chromatin remodelers that replace common histone protein 2A with different kind, altering its pairings and interactions.

==================================================

Question: Genomic imprinting

Answer: alleles that show different expression depending on whether or not they are maternal or paternal.

==================================================

Question: RNAi

Answer: A regulatory gene-silencing mechanism in which dsRNA targets complementary strands for inactivation.

==================================================

Question: dicer

Answer: enzyme that cuts the dsRNA into small fragments in RNAi

==================================================

Question: recombinant DNA technology

Answer: Lab techniques for amplifying, maintaining and manipulating specific DNA sequences in vitro and in vivo.

==================================================

Question: recombinant DNA vector

Answer: vector and DNA from different sources used in DNA cloning

==================================================

Question: nonrecombinant DNA vector

Answer: In DNA cloning this is made when the intended vector doesn't pick up a DNA insert.

==================================================

Question: transgenic organism

Answer: an organism harbouring a transgene (a gene modified in vitro by recombinant DNA technology and introduced into the genome.)

==================================================

Question: Expression vector

Answer: a cloning vector that has the DNA sequences required for DNA fragments to be inserted and be transcribed and translated.

==================================================

Biointeractive The Eukaryotic Cell Cycle And Cancer

Question: Unicellular

Answer: only way to reproduce

==================================================

Question: Mulitcellular

Answer: Replaces dead or damaged cells and more than one way to reproduce

==================================================

Question: Why is cell division important to an adult organism?

Answer: Skin + Digestive system cells are continuously renewed

==================================================

Question: What is the role of growth factors?

Answer: It signals cell division

==================================================

Question: What is differentiation ?

Answer: Differentiation is when a cell receives other signal from other nucleuses

==================================================

Question: What are cell cycle regulators?

Answer: Cyclin-Dependent Kinase. Through phosphorylation

==================================================

Question: Where does cell division occur?

Answer: Meiosis for sex cells and Mitosis for body cells

==================================================

Question: Where does cell differentiation occur?

Answer: Crypt-Right after moving into the lumen

==================================================

Question: Where does apoptosis occur?

Answer: Top of the villi/peak

==================================================

Question: What is one harmless result of too little cell division?

Answer: Hair loss

==================================================

Question: What is one harmless result of too much cell division?

Answer: Warts

==================================================

Question: List in order the Cell Cycle phases

Answer: 1. G1 2. S 3. G2 4.M

==================================================

Question: What is G1 in the Cell Cycle?

Answer: G1-Gap1 and is the first checkpoint and its job is cell division.

==================================================

Question: What is S in the Cell Cycle?

Answer: S-Synthesis it is the second checkpoint and its job is to make sure there are no errors

==================================================

Question: What is G2 in the Cell Cycle?

Answer: G2-Gap2 and it is checkpoint 3 and its purpose is to make sure the DNA is intact

==================================================

Question: What is M in the cell cycle?

Answer: M-Mitosis it is the fourth checkpoint(final checkpoint) its purpose is to make sure the chromosomes are unattached

==================================================

Question: What is the purpose of a checkpoint during the cell cycle?

Answer: To ensure proper division of the cell

==================================================

Question: What is one potential outcome when error occur in the cell cycle?

Answer: The cell can mutate

==================================================

Question: What is the type of protein that regulates the cell cycle is encoded by proto-oncogenes?

Answer: APC/C

==================================================

Question: What type of protein that regulates the tumor suppressor genes?

Answer: p53

==================================================

Question: What are the most important cell cycle regulators?

Answer: CDK's-Cyclin-Independent Kinases

==================================================

Question: What is a kinase?

Answer: Enzymes that add phosphate to other proteins.

==================================================

Question: When are cyclins present inside the cell during cell cycle?

Answer: Cyclin is only active when the cyclin is attached to CDK

==================================================

Question: What do active CDK's do?

Answer: Regulate how the time of each phase

==================================================

Question: What are the phase events for G1 ?

Answer: Increase in size

==================================================

Question: What are the phase events for S/Synthesis?

Answer: Replicate the Dna

==================================================

Question: What are the phase events for G2?

Answer: Organelles duplicate

==================================================

Question: What are the phase events for M/Mitosis?

Answer: Two daughter cells

==================================================

Question: What are the Regulatory Process for G1?

Answer: CDK,p53,and RB

==================================================

Question: What are the Regulatory process for S?

Answer: CDK and BRCA 1

==================================================

Question: What are the Regulatory process for G2?

Answer: CDK and p53

==================================================

Question: What are the Regulatory process for M?

Answer: apc/c and mad

==================================================

Question: What is the GO phase in the cell cycle?

Answer: The cell doesn't divide

==================================================

Question: What is and example of a cell that is permanently in GO?

Answer: Neurons and Muscle cells.

==================================================

Question: What is and example of a cell that can leave GO to go through the cell cycle again?

Answer: Liver cells

==================================================

Question: Name two reasons why cells can form tumors?

Answer: Too much cell division and too little cell death

==================================================

Question: What causes uncontrolled cell division?

Answer: Mutations of proteins that regulate the cell cycle

==================================================

Question: What occurs if the apc gene is mutated?

Answer: They don't differentiate into epithelial cells in the lumen

==================================================

Question: What do mutated proto-oncogenes do?

Answer: They cause the cell cycle to speed up

==================================================

Question: What do mutated tumor suppressor genes do?

Answer: They deregulate the cell cycle

==================================================

Question: What do proto-oncogene need to cause cancer?

Answer: Proto-oncogenes require 1 allele to be mutated to be considered dominant. In the results there is a gain in the function.

==================================================

Question: What do tumor suppressor genes need to cause cancer?

Answer: they need 2 alleles to be mutated to be considered recessive in the results there is a loss in the function.

==================================================

Question: Difference between a proto-oncogene and an oncogene?

Answer: If one of these two is mutated the cell cycle goes too fast (pedal to the medal)

==================================================

Question: Difference between a mutated tumor suppressor gene allele versus two mutated tumor suppressor gene alleles?

Answer: If one is mutated the cell cycle is OK but if both are mutated the cell cycle wont function.

==================================================

Question: Name 2 types of cancer treatments and how they affect the body?

Answer: Chemotherapy-uses radiation to kill all cells but as well as it is killing the cancer cells it is also killing the good cells so the immune system decreases.

Immunotherapy- Is a treatment that uses the person immune system to fight off the cancer cells by stimulation to fight the cancer cells harder

==================================================

N 400 Preguntas En Español

Question: ¿Ha declarado alguna vez ser ciudadano de los EE.UU. (por escrito o en cualquier forma)?

Answer: Have you ever claimed to be a U.S. Citizen?

==================================================

Question: ¿Se ha registrado alguna vez para votar en cualquier elección federal, estatal o local en los Estados Unidos?

Answer: Have you ever registered to vote in any Federal, State, or local election in the United States?

==================================================

Question: ¿Ha votado alguna vez en cualquier elección federal, estatal o local en los Estados Unidos?

Answer: Have you ever voted in any Federal, State, or local election in the United States?

==================================================

Question: ¿Tiene ahora o alguna vez ha tenido un título hereditario o una orden de nobleza de cualquier país extranjero?

Answer: Do you now have, or did you ever have, a hereditary title or an order of nobility in any foreign country?

==================================================

Question: ¿Alguna vez ha sido declarado legalmente incompetente o ha sido confinado a una institución para enfermos mentales?

Answer: Have you ever been declared legally incompetent, or been confined to a mental institution?

==================================================

Question: ¿Debe algún impuesto atrasado Federal, Estatal, o local?

Answer: Do you owe any Federal, State, or local taxes?

==================================================

Question: Desde que se convirtió en Residente Permanente ¿alguna vez ha dejado de declarar impuestos Federales, Estatales, o locales?

Answer: Have you ever not filed a Federal, State, or local tax return since you became a permanent resident?

==================================================

Question: Desde que se convirtió en Residente Permanente ¿se ha declarado una persona "no-residente"de los EE.UU. en una declaración de impuestos Federales, Estatales, o locales?

Answer: Have you called yourself a "non-U.S. resident" on a Federal, State, or local tax return since you became a Permanent Resident?

==================================================

Question: ¿Ha sido alguna vez miembro de, involucrado con, o asociado de alguna manera con alguna organización, asociación, fondo, fundación, partido, club, sociedad, o grupo similar en los Estados Unidos o en cualquier otra parte del mundo?

Answer: Have you ever been a member of, involved in, or in any way associated with any organization, association, fund, foundation, party, club, society, or similar group in the United States or in any other location in the world? If so, what are they?

==================================================

Question: Ha sido alguna vez miembro de, o ha estado asociado de alguna manera (directamente o indirectamente) con:
A. ¿El partido comunista?
B. ¿Cualquier partido totalitario?
C. ¿Una organización terrorista?

Answer: Have you ever been a member of, or in any way associated with the Communist Party?
...any other totalitarian party
...a terrorist organization

==================================================

Question: ¿Alguna vez ha abogado (directamente o indirectamente) por el derrumbamiento de algún gobierno mediante la fuerza o la violencia?

Answer: Have you ever advocated the overthrow of any government by force or violence?

==================================================

Question: ¿Alguna vez ha perseguido (directamente o indirectamente) a alguna persona a causa de su raza, religión, origen nacional, pertenencia a un grupo social u opinión política?

Answer: Have you ever persecuted any person because of race, religion, national origin, membership in a particular social group, or political opinion?

==================================================

Question: ¿Entre el 23 de marzo de 1933 y el 8 de mayo de 1945 trabajo o se asoció en cualquier forma
(directa o indirecta) con:

¿El gobierno Nazi de Alemania?

Answer: Did you work for or associate in any way with the Nazi government of Germany?

==================================================

Question: ¿Alguna vez estuvo involucrado en cualquier forma con cualquiera de los siguientes:
¿El Genocidio?

Answer: Were you ever involved in any way with genocide?

==================================================

Question: ¿La Tortura?

Answer: Were you ever involved in any way with torture?

==================================================

Question: ¿Matar o intentar matar a alguien?

Answer: Were you ever involved in any way with killing, or trying to kill, someone?

==================================================

Question: ¿Lastimar fuertemente o intentar lastimar a una persona a propósito?

Answer: Were you ever involved in any way with badly hurting, or trying to hurt, a person on purpose?

==================================================

Question: ¿Forzar o intentar forzar a alguien a tener cualquier tipo de contacto o relación sexual?

Answer: Were you ever involved in any way with forcing, or trying to force, someone to have any kind of sexual contact or relations?

==================================================

Question: ¿No permitir a alguien practicar su religión?

Answer: Were you ever involved in any way with not letting someone practice his or her religion?

==================================================

Question: ¿Alguna vez fue miembro de, o alguna vez sirvió en, ayudo a, o de otra manera participo en cualquiera de los siguientes grupos:
¿Unidad Militar?

Answer: Were you ever a member of, or did you serve in a military unit?

==================================================

Question: ¿Unidad Paramilitar? (un grupo de personas que actúan como un grupo militar, pero no son parte del militar oficial)

Answer: Were you ever a member of, or did you serve in a paramilitary unit?

==================================================

Question: ¿Unidad de Policía?

Answer: Were you ever a member of, or did you serve in a police unit?

==================================================

Question: ¿Unidad de defensa propia?

Answer: Were you ever a member of, or did you serve in a self-defense unit?

==================================================

Question: ¿Unidad Vigilante? (un grupo de personas que actúan como la policía pero no son parte de la policía oficial)

Answer: Were you ever a member of, or did you serve in a vigilante unit?

==================================================

Question: ¿Grupo de Rebeldes?

Answer: Were you ever a member of, or did you serve in a rebel group?

==================================================

Question: ¿Grupo de Guerrillas? (un grupo de personas que utilizan armas en contra de, o de otra manera atacan físicamente al militar, la policía, el gobierno, u otras personas)

Answer: Were you ever a member of, or did you serve in a guerrilla group?

==================================================

Question: ¿Milicia? (un ejército de personas, que no es parte del militar oficial)

Answer: Were you ever a member of, or did you serve in a militia?

==================================================

Question: ¿Organización Insurgente? (un grupo que utiliza armas y lucha en contra de un gobierno)

Answer: Were you ever a member of, or did you serve in an insurgent organization?

==================================================

Question: ¿Alguna vez fue un trabajador, voluntario, soldado o de otra manera alguna vez sirvió
en cualquiera de los siguientes:
¿Prisión o cárcel?

Answer: Were you ever a worker, volunteer or soldier, or did you ever serve in a prison or jail?

==================================================

Question: ¿Campo de Prisión?

Answer: Were you ever a worker, volunteer or soldier, or did you ever serve in a prison camp?

==================================================

Question: ¿Centro de Detención? (un lugar en donde las personas son forzados a permanecer)

Answer: Were you ever a worker, volunteer or soldier, or did you ever serve in a detention facility?

==================================================

Question: ¿Campo de Trabajos Forzados? (un lugar en donde las personas son forzados a trabajar)

Answer: Were you ever a worker, volunteer or soldier, or did you ever serve in a labor camp?

==================================================

Question: ¿Algún otro lugar en donde las personas fueron forzadas a permanecer?

Answer: Were you ever a worker, volunteer or soldier, or did you ever serve in any other place where people were forced to stay?

==================================================

Question: ¿Alguna vez fue parte de algún grupo o alguna vez ayudo a algún grupo, unidad u organización que utilizo una arma en contra de alguna persona o amenazó con hacerlo?

Answer: Were you ever a part of a group, or did you ever help any group, unit or organization that used a weapon against any person, or threatened to do so?

==================================================

Question: ¿Alguna vez vendió, dio, o proporciono armas a cualquier persona, o ayudo a otra persona a vender, dar o proporcionar armas a alguna persona?

Answer: Did you ever sell, give, or provide weapons to any person or help another person sell weapons?

==================================================

Question: ¿Alguna vez recibió algún tipo de entrenamiento militar, paramilitar (grupo de personas que actúan como un grupo militar pero no es parte de la militaría oficial) o para el uso de armas?

Answer: Did you ever receive any type of military, paramilitary, or weapons training?

==================================================

Question: ¿Alguna vez recluto (pregunto), alisto (suscribió), conscripto (requirió) o utilizo a alguna persona menor de 15 años de edad para servir o ayudar en una fuerza armada o grupo?

Answer: Did you ever recruit, enlist, conscript or use any person under age 15 to serve in or help an armed force or group?

==================================================

Question: ¿Alguna vez utilizo a cualquier persona menor de 15 años de edad para hacer cualquier cosa que ayudará o apoyara a las personas en combate?

Answer: Did you ever use any person under age 15 to do anything that helped or supported people in combat?

==================================================

Question: ¿Alguna vez ha cometido, asistido en cometer, o intentado cometer, un crimen o delito por el
cual no fue arrestado?

Answer: Have you ever committed a crime, assisted in committing, or attempted to commit a crime or offense for which you were not arrested?

==================================================

Question: ¿Alguna vez ha sido arrestado, citado, o detenido por algún oficial de la ley (incluyendo todos o alguno de los siguientes: agentes de inmigración o de la fuerzas armadas de EE.UU.) por cualquier razón?

Answer: Have you ever been arrested, citied, or detained by any law enforcement officer for any reason?

==================================================

Question: ¿Alguna vez ha sido acusado de cometer, intentar cometer, o asistir en cometer un crimen u delito?

Answer: Have you ever been charged with committing, attempting to commit, or assisting in committing a crime or offense?

==================================================

Question: ¿Alguna vez ha sido condenado por un crimen u ofensa?

Answer: Have you ever been convicted of a crime or offense?

==================================================

Question: ¿Alguna vez lo han puesto en una sentencia alternativa o un programa de rehabilitación (e.g.,desvió, proceso diferido, adjudicación suspendida, o adjudicación diferida.)?

Answer: Have you ever been placed in an alternative sentencing or rehabilitative program?

==================================================

Question: ¿Alguna vez ha recibido una sentencia suspendida, libertad provisoria, o libertad condicional?

Answer: Have you ever received a suspended sentence, been placed on probation, or been paroled?

==================================================

Question: ¿Alguna vez ha estado en la cárcel o prisión?

Answer: Have you ever been in jail or prison?

==================================================

Question: ¿Ha sido alcohólico habitual?

Answer: Have you ever been a habitual drunkard?

==================================================

Question: ¿Ha sido prostituta o recurrido a alguien con fines de prostitución?

Answer: Have you ever been a prostitute or procured anyone for prostitution?

==================================================

Question: ¿Ha vendido o contrabandeado sustancias controladas, drogas ilegales o narcóticos?

Answer: Have you ever sold or smuggled controlled substances, illegal drugs, or narcotics?

==================================================

Question: ¿Ha estado casado con más de una persona al mismo tiempo?

Answer: Have you ever been married to more than one person at the same time?

==================================================

Question: ¿Se ha casado con alguien para obtener un beneficio migratorio?

Answer: Have you ever married someone in order to obtain an immigration benefit?

==================================================

Question: ¿Ha ayudado a alguna persona a entrar o intentar entrar a los Estado Unidos ilegalmente?

Answer: Have you ever helped anyone enter or try to enter the United States illegally?

==================================================

Question: ¿Ha participado en juego ilegal o recibido ingresos procedentes de algún juego ilegal?

Answer: Have you ever gambled illegally or received income from illegal gambling?

==================================================

Question: ¿Ha dejado de dar apoyo económico o pensión alimenticia a sus dependientes?

Answer: Have you ever failed to support your dependents or to pay alimony?

==================================================

Question: ¿Ha dado una declaración falsa para obtener cualquier beneficio público en los Estados
Unidos?

Answer: Have you ever made any misrepresentation to obtain any public benefit in the United States?

==================================================

Question: ¿Alguna vez ha dado alguna información o documento falso, fraudulento o para engañar a
algún oficial del gobierno de los EE.UU.?

Answer: Have you ever given any U.S. Government official(s) any information or documentation that was false, fraudulent or misleading?

==================================================

Question: ¿Alguna vez ha mentido a algún oficial del gobierno de los EE.UU. para entrar o ser admitido a los Estados Unidos o para recibir beneficios de inmigración en los Estados Unidos?

Answer: Have you ever lied to any U.S. Government official to gain entry or admission into the United States or to gain immigration benefits while in the United States?

==================================================

Question: ¿Alguna vez lo han removido, excluido o deportado de los Estados Unidos?

Answer: Have you ever been removed, excluded, or deported from the United States?

==================================================

Question: ¿Alguna vez ha recibido una orden para ser removido, excluido o deportado de los Estados Unidos?

Answer: Have you ever been ordered removed, excluded, or deported from the United States?

==================================================

Question: ¿Alguna vez ha sido puesto en procedimientos de remoción, rescisión, exclusión o deportación?

Answer: Have you ever been placed in removal, exclusion, rescission, or deportation proceedings?

==================================================

Question: ¿Existe actualmente procedimientos de remoción, exclusión, rescisión, o deportación en su contra (incluyendo procedimientos administrativos que hayan sido cerrados)?

Answer: Are removal, exclusion, rescission, or deportation proceedings currently pending against you? (Is the U.S. saying that you have to leave the U.S. soon?)

==================================================

Question: ¿Alguna vez ha servido en las Fuerza Armadas de los EE.UU.?

Answer: Have you ever served in the U.S. Armed Forces?

==================================================

Question: ¿Es actualmente un miembro de las Fuerzas Armadas de los EE.UU.?

Answer: Are you currently a member of the U.S. Armed Forces?

==================================================

Question: ¿Alguna vez ha sido disciplinado bajo consejo de guerra, administrativamente separado, o disciplinado, o recibido un despido que no haya sido honorable de las Fuerzas Armadas de los EE.UU.?

Answer: Have you ever been court-martialed, administratively separated, or disciplined, or have you received an other than honorable discharge, while in the U.S. Armed Forces?

==================================================

Question: ¿Alguna vez ha sido despedido del entrenamiento o servicio en las Fuerzas Armadas de los EE.UU. por no ser ciudadano de los EE.UU.?

Answer: Have you ever been discharged from training or service in the U.S. Armed Forces because you were an alien?

==================================================

Question: ¿Alguna vez ha dejado los Estados Unidos para evitar ser reclutado a las Fuerzas Armadas de los EE.UU.?

Answer: Have you ever left the United States to avoid being drafted in the U.S. Armed Forces?

==================================================

Question: ¿Alguna vez ha solicitado cualquier excepción del servicio militar de las Fuerzas Armadas de los EE.UU.?

Answer: Have you ever applied for any kind of exemption from military service in the U.S. Armed Forces?

==================================================

Question: ¿Alguna vez ha desertado de las Fuerzas Armadas de los EE.UU.?

Answer: Have you ever deserted from the U.S. Armed Forces?

==================================================

Question: ¿Es usted un varón que vivió en los EE.UU. en cualquier momento entre sus 18 y 26 años de edad? (Esto no incluye tiempo en el que usted vivió legalmente como no-migrante.).

Answer: Are you a male who lived in the United States at any time between your 18th and 26th birthday?

==================================================

Question: ¿Apoya usted la Constitución y el sistema de gobierno de los Estados Unidos?

Answer: Do you support the Constitution and form of government of the United States?

==================================================

Question: ¿Comprende usted el juramento completo de lealtad de los Estados Unidos?

Answer: Do you understand the full Oath of Allegiance to the United States?

==================================================

Question: ¿Qué significa el juramento de lealtad de los Estaus Unidos?

Answer: It means I am loyal to the US and its constitution, and I will follow the laws of this country.

==================================================

Question: ¿Está usted dispuesto a tomar completamente el juramento de lealtad de los Estados Unidos?

Answer: Are you willing to take the full Oath of Allegiance to the United States?

==================================================

Question: ¿Si la ley lo requiere, está dispuesto a tomar las armas en nombre de los Estados Unidos?

Answer: If the law requires it, are you willing to bear arms on behalf of the United States?

==================================================

Question: ¿Si la ley lo requiere, está dispuesto a prestar servicios como no combatiente en las Fuerzas Armadas de los EE.UU.?

Answer: If the law requires it, are you willing to perform noncombatant services in the U.S. Armed Forces?

==================================================

Question: ¿Si la ley lo requiere está dispuesto a realizar trabajo de importancia nacional bajo dirección civil?

Answer: If the law requires it, are you willing to perform work of national importance under civilian direction?

==================================================

Question: ¿En la ceremonia de naturalización está dispuesto a renunciar a algún título(s) hereditario u orden de nobleza que usted tenga en un país extranjero?

Answer: In the naturalization ceremony, are you willing to renounce any hereditary titles or titles of nobility that you hold in a foreign country?

==================================================

N 400 Preguntas En Español

Question: ¿Ha declarado alguna vez ser ciudadano de los EE.UU. (por escrito o en cualquier forma)?

Answer: Have you ever claimed to be a U.S. Citizen?

==================================================

Question: ¿Se ha registrado alguna vez para votar en cualquier elección federal, estatal o local en los Estados Unidos?

Answer: Have you ever registered to vote in any Federal, State, or local election in the United States?

==================================================

Question: ¿Ha votado alguna vez en cualquier elección federal, estatal o local en los Estados Unidos?

Answer: Have you ever voted in any Federal, State, or local election in the United States?

==================================================

Question: ¿Tiene ahora o alguna vez ha tenido un título hereditario o una orden de nobleza de cualquier país extranjero?

Answer: Do you now have, or did you ever have, a hereditary title or an order of nobility in any foreign country?

==================================================

Question: ¿Alguna vez ha sido declarado legalmente incompetente o ha sido confinado a una institución para enfermos mentales?

Answer: Have you ever been declared legally incompetent, or been confined to a mental institution?

==================================================

Question: ¿Debe algún impuesto atrasado Federal, Estatal, o local?

Answer: Do you owe any Federal, State, or local taxes?

==================================================

Question: Desde que se convirtió en Residente Permanente ¿alguna vez ha dejado de declarar impuestos Federales, Estatales, o locales?

Answer: Have you ever not filed a Federal, State, or local tax return since you became a permanent resident?

==================================================

Question: Desde que se convirtió en Residente Permanente ¿se ha declarado una persona "no-residente"de los EE.UU. en una declaración de impuestos Federales, Estatales, o locales?

Answer: Have you called yourself a "non-U.S. resident" on a Federal, State, or local tax return since you became a Permanent Resident?

==================================================

Question: ¿Ha sido alguna vez miembro de, involucrado con, o asociado de alguna manera con alguna organización, asociación, fondo, fundación, partido, club, sociedad, o grupo similar en los Estados Unidos o en cualquier otra parte del mundo?

Answer: Have you ever been a member of, involved in, or in any way associated with any organization, association, fund, foundation, party, club, society, or similar group in the United States or in any other location in the world? If so, what are they?

==================================================

Question: Ha sido alguna vez miembro de, o ha estado asociado de alguna manera (directamente o indirectamente) con:
A. ¿El partido comunista?
B. ¿Cualquier partido totalitario?
C. ¿Una organización terrorista?

Answer: Have you ever been a member of, or in any way associated with the Communist Party?
...any other totalitarian party
...a terrorist organization

==================================================

Question: ¿Alguna vez ha abogado (directamente o indirectamente) por el derrumbamiento de algún gobierno mediante la fuerza o la violencia?

Answer: Have you ever advocated the overthrow of any government by force or violence?

==================================================

Question: ¿Alguna vez ha perseguido (directamente o indirectamente) a alguna persona a causa de su raza, religión, origen nacional, pertenencia a un grupo social u opinión política?

Answer: Have you ever persecuted any person because of race, religion, national origin, membership in a particular social group, or political opinion?

==================================================

Question: ¿Entre el 23 de marzo de 1933 y el 8 de mayo de 1945 trabajo o se asoció en cualquier forma
(directa o indirecta) con:

¿El gobierno Nazi de Alemania?

Answer: Did you work for or associate in any way with the Nazi government of Germany?

==================================================

Question: ¿Alguna vez estuvo involucrado en cualquier forma con cualquiera de los siguientes:
¿El Genocidio?

Answer: Were you ever involved in any way with genocide?

==================================================

Question: ¿La Tortura?

Answer: Were you ever involved in any way with torture?

==================================================

Question: ¿Matar o intentar matar a alguien?

Answer: Were you ever involved in any way with killing, or trying to kill, someone?

==================================================

Question: ¿Lastimar fuertemente o intentar lastimar a una persona a propósito?

Answer: Were you ever involved in any way with badly hurting, or trying to hurt, a person on purpose?

==================================================

Question: ¿Forzar o intentar forzar a alguien a tener cualquier tipo de contacto o relación sexual?

Answer: Were you ever involved in any way with forcing, or trying to force, someone to have any kind of sexual contact or relations?

==================================================

Question: ¿No permitir a alguien practicar su religión?

Answer: Were you ever involved in any way with not letting someone practice his or her religion?

==================================================

Question: ¿Alguna vez fue miembro de, o alguna vez sirvió en, ayudo a, o de otra manera participo en cualquiera de los siguientes grupos:
¿Unidad Militar?

Answer: Were you ever a member of, or did you serve in a military unit?

==================================================

Question: ¿Unidad Paramilitar? (un grupo de personas que actúan como un grupo militar, pero no son parte del militar oficial)

Answer: Were you ever a member of, or did you serve in a paramilitary unit?

==================================================

Question: ¿Unidad de Policía?

Answer: Were you ever a member of, or did you serve in a police unit?

==================================================

Question: ¿Unidad de defensa propia?

Answer: Were you ever a member of, or did you serve in a self-defense unit?

==================================================

Question: ¿Unidad Vigilante? (un grupo de personas que actúan como la policía pero no son parte de la policía oficial)

Answer: Were you ever a member of, or did you serve in a vigilante unit?

==================================================

Question: ¿Grupo de Rebeldes?

Answer: Were you ever a member of, or did you serve in a rebel group?

==================================================

Question: ¿Grupo de Guerrillas? (un grupo de personas que utilizan armas en contra de, o de otra manera atacan físicamente al militar, la policía, el gobierno, u otras personas)

Answer: Were you ever a member of, or did you serve in a guerrilla group?

==================================================

Question: ¿Milicia? (un ejército de personas, que no es parte del militar oficial)

Answer: Were you ever a member of, or did you serve in a militia?

==================================================

Question: ¿Organización Insurgente? (un grupo que utiliza armas y lucha en contra de un gobierno)

Answer: Were you ever a member of, or did you serve in an insurgent organization?

==================================================

Question: ¿Alguna vez fue un trabajador, voluntario, soldado o de otra manera alguna vez sirvió
en cualquiera de los siguientes:
¿Prisión o cárcel?

Answer: Were you ever a worker, volunteer or soldier, or did you ever serve in a prison or jail?

==================================================

Question: ¿Campo de Prisión?

Answer: Were you ever a worker, volunteer or soldier, or did you ever serve in a prison camp?

==================================================

Question: ¿Centro de Detención? (un lugar en donde las personas son forzados a permanecer)

Answer: Were you ever a worker, volunteer or soldier, or did you ever serve in a detention facility?

==================================================

Question: ¿Campo de Trabajos Forzados? (un lugar en donde las personas son forzados a trabajar)

Answer: Were you ever a worker, volunteer or soldier, or did you ever serve in a labor camp?

==================================================

Question: ¿Algún otro lugar en donde las personas fueron forzadas a permanecer?

Answer: Were you ever a worker, volunteer or soldier, or did you ever serve in any other place where people were forced to stay?

==================================================

Question: ¿Alguna vez fue parte de algún grupo o alguna vez ayudo a algún grupo, unidad u organización que utilizo una arma en contra de alguna persona o amenazó con hacerlo?

Answer: Were you ever a part of a group, or did you ever help any group, unit or organization that used a weapon against any person, or threatened to do so?

==================================================

Question: ¿Alguna vez vendió, dio, o proporciono armas a cualquier persona, o ayudo a otra persona a vender, dar o proporcionar armas a alguna persona?

Answer: Did you ever sell, give, or provide weapons to any person or help another person sell weapons?

==================================================

Question: ¿Alguna vez recibió algún tipo de entrenamiento militar, paramilitar (grupo de personas que actúan como un grupo militar pero no es parte de la militaría oficial) o para el uso de armas?

Answer: Did you ever receive any type of military, paramilitary, or weapons training?

==================================================

Question: ¿Alguna vez recluto (pregunto), alisto (suscribió), conscripto (requirió) o utilizo a alguna persona menor de 15 años de edad para servir o ayudar en una fuerza armada o grupo?

Answer: Did you ever recruit, enlist, conscript or use any person under age 15 to serve in or help an armed force or group?

==================================================

Question: ¿Alguna vez utilizo a cualquier persona menor de 15 años de edad para hacer cualquier cosa que ayudará o apoyara a las personas en combate?

Answer: Did you ever use any person under age 15 to do anything that helped or supported people in combat?

==================================================

Question: ¿Alguna vez ha cometido, asistido en cometer, o intentado cometer, un crimen o delito por el
cual no fue arrestado?

Answer: Have you ever committed a crime, assisted in committing, or attempted to commit a crime or offense for which you were not arrested?

==================================================

Question: ¿Alguna vez ha sido arrestado, citado, o detenido por algún oficial de la ley (incluyendo todos o alguno de los siguientes: agentes de inmigración o de la fuerzas armadas de EE.UU.) por cualquier razón?

Answer: Have you ever been arrested, citied, or detained by any law enforcement officer for any reason?

==================================================

Question: ¿Alguna vez ha sido acusado de cometer, intentar cometer, o asistir en cometer un crimen u delito?

Answer: Have you ever been charged with committing, attempting to commit, or assisting in committing a crime or offense?

==================================================

Question: ¿Alguna vez ha sido condenado por un crimen u ofensa?

Answer: Have you ever been convicted of a crime or offense?

==================================================

Question: ¿Alguna vez lo han puesto en una sentencia alternativa o un programa de rehabilitación (e.g.,desvió, proceso diferido, adjudicación suspendida, o adjudicación diferida.)?

Answer: Have you ever been placed in an alternative sentencing or rehabilitative program?

==================================================

Question: ¿Alguna vez ha recibido una sentencia suspendida, libertad provisoria, o libertad condicional?

Answer: Have you ever received a suspended sentence, been placed on probation, or been paroled?

==================================================

Question: ¿Alguna vez ha estado en la cárcel o prisión?

Answer: Have you ever been in jail or prison?

==================================================

Question: ¿Ha sido alcohólico habitual?

Answer: Have you ever been a habitual drunkard?

==================================================

Question: ¿Ha sido prostituta o recurrido a alguien con fines de prostitución?

Answer: Have you ever been a prostitute or procured anyone for prostitution?

==================================================

Question: ¿Ha vendido o contrabandeado sustancias controladas, drogas ilegales o narcóticos?

Answer: Have you ever sold or smuggled controlled substances, illegal drugs, or narcotics?

==================================================

Question: ¿Ha estado casado con más de una persona al mismo tiempo?

Answer: Have you ever been married to more than one person at the same time?

==================================================

Question: ¿Se ha casado con alguien para obtener un beneficio migratorio?

Answer: Have you ever married someone in order to obtain an immigration benefit?

==================================================

Question: ¿Ha ayudado a alguna persona a entrar o intentar entrar a los Estado Unidos ilegalmente?

Answer: Have you ever helped anyone enter or try to enter the United States illegally?

==================================================

Question: ¿Ha participado en juego ilegal o recibido ingresos procedentes de algún juego ilegal?

Answer: Have you ever gambled illegally or received income from illegal gambling?

==================================================

Question: ¿Ha dejado de dar apoyo económico o pensión alimenticia a sus dependientes?

Answer: Have you ever failed to support your dependents or to pay alimony?

==================================================

Question: ¿Ha dado una declaración falsa para obtener cualquier beneficio público en los Estados
Unidos?

Answer: Have you ever made any misrepresentation to obtain any public benefit in the United States?

==================================================

Question: ¿Alguna vez ha dado alguna información o documento falso, fraudulento o para engañar a
algún oficial del gobierno de los EE.UU.?

Answer: Have you ever given any U.S. Government official(s) any information or documentation that was false, fraudulent or misleading?

==================================================

Question: ¿Alguna vez ha mentido a algún oficial del gobierno de los EE.UU. para entrar o ser admitido a los Estados Unidos o para recibir beneficios de inmigración en los Estados Unidos?

Answer: Have you ever lied to any U.S. Government official to gain entry or admission into the United States or to gain immigration benefits while in the United States?

==================================================

Question: ¿Alguna vez lo han removido, excluido o deportado de los Estados Unidos?

Answer: Have you ever been removed, excluded, or deported from the United States?

==================================================

Question: ¿Alguna vez ha recibido una orden para ser removido, excluido o deportado de los Estados Unidos?

Answer: Have you ever been ordered removed, excluded, or deported from the United States?

==================================================

Question: ¿Alguna vez ha sido puesto en procedimientos de remoción, rescisión, exclusión o deportación?

Answer: Have you ever been placed in removal, exclusion, rescission, or deportation proceedings?

==================================================

Question: ¿Existe actualmente procedimientos de remoción, exclusión, rescisión, o deportación en su contra (incluyendo procedimientos administrativos que hayan sido cerrados)?

Answer: Are removal, exclusion, rescission, or deportation proceedings currently pending against you? (Is the U.S. saying that you have to leave the U.S. soon?)

==================================================

Question: ¿Alguna vez ha servido en las Fuerza Armadas de los EE.UU.?

Answer: Have you ever served in the U.S. Armed Forces?

==================================================

Question: ¿Es actualmente un miembro de las Fuerzas Armadas de los EE.UU.?

Answer: Are you currently a member of the U.S. Armed Forces?

==================================================

Question: ¿Alguna vez ha sido disciplinado bajo consejo de guerra, administrativamente separado, o disciplinado, o recibido un despido que no haya sido honorable de las Fuerzas Armadas de los EE.UU.?

Answer: Have you ever been court-martialed, administratively separated, or disciplined, or have you received an other than honorable discharge, while in the U.S. Armed Forces?

==================================================

Question: ¿Alguna vez ha sido despedido del entrenamiento o servicio en las Fuerzas Armadas de los EE.UU. por no ser ciudadano de los EE.UU.?

Answer: Have you ever been discharged from training or service in the U.S. Armed Forces because you were an alien?

==================================================

Question: ¿Alguna vez ha dejado los Estados Unidos para evitar ser reclutado a las Fuerzas Armadas de los EE.UU.?

Answer: Have you ever left the United States to avoid being drafted in the U.S. Armed Forces?

==================================================

Question: ¿Alguna vez ha solicitado cualquier excepción del servicio militar de las Fuerzas Armadas de los EE.UU.?

Answer: Have you ever applied for any kind of exemption from military service in the U.S. Armed Forces?

==================================================

Question: ¿Alguna vez ha desertado de las Fuerzas Armadas de los EE.UU.?

Answer: Have you ever deserted from the U.S. Armed Forces?

==================================================

Question: ¿Es usted un varón que vivió en los EE.UU. en cualquier momento entre sus 18 y 26 años de edad? (Esto no incluye tiempo en el que usted vivió legalmente como no-migrante.).

Answer: Are you a male who lived in the United States at any time between your 18th and 26th birthday?

==================================================

Question: ¿Apoya usted la Constitución y el sistema de gobierno de los Estados Unidos?

Answer: Do you support the Constitution and form of government of the United States?

==================================================

Question: ¿Comprende usted el juramento completo de lealtad de los Estados Unidos?

Answer: Do you understand the full Oath of Allegiance to the United States?

==================================================

Question: ¿Qué significa el juramento de lealtad de los Estaus Unidos?

Answer: It means I am loyal to the US and its constitution, and I will follow the laws of this country.

==================================================

Question: ¿Está usted dispuesto a tomar completamente el juramento de lealtad de los Estados Unidos?

Answer: Are you willing to take the full Oath of Allegiance to the United States?

==================================================

Question: ¿Si la ley lo requiere, está dispuesto a tomar las armas en nombre de los Estados Unidos?

Answer: If the law requires it, are you willing to bear arms on behalf of the United States?

==================================================

Question: ¿Si la ley lo requiere, está dispuesto a prestar servicios como no combatiente en las Fuerzas Armadas de los EE.UU.?

Answer: If the law requires it, are you willing to perform noncombatant services in the U.S. Armed Forces?

==================================================

Question: ¿Si la ley lo requiere está dispuesto a realizar trabajo de importancia nacional bajo dirección civil?

Answer: If the law requires it, are you willing to perform work of national importance under civilian direction?

==================================================

Question: ¿En la ceremonia de naturalización está dispuesto a renunciar a algún título(s) hereditario u orden de nobleza que usted tenga en un país extranjero?

Answer: In the naturalization ceremony, are you willing to renounce any hereditary titles or titles of nobility that you hold in a foreign country?

==================================================

Nclex Important Topics

Question: Vitamins (A,B,C,D,E) - know what each vitamin benefits

Answer: Vitamin A - eyes, immune system, and reproduction
Vitamin B - make energy from food
Vitamin C - helps your body heal - immune deficiency
Vitamin D - Helps your bones
Vitamin E - Healthy skin and eyes

==================================================

Question: KNOW SIDE EFFECTS OF COMMON MEDICATIONS

Answer: should be taken with food - do not give to a patient who has not eaten

==================================================

Question: Nsaids (ibuprofen):

Answer: Metronidazole - treats various of infections

may discolor urine. Also pyridum makes urine orange (medicine for UTI to reduce burning)

==================================================

Question: Flagyl

Answer: common side effect is discoloration of urine

==================================================

Question: Levadopa

Answer: know that antacids (Maalox, Mylanta) are used to keep the PH of gastric contents above 3.5. They DO NOT reduce gastric secretions.

==================================================

Question: GI medications

Answer: reduce gastric secretions - pepcid or prilosec

==================================================

Question: H2 blockers

Answer: elevate blood sugar, be careful with diabetics.
Also, there usually are questions regarding steroid use. Always taper off of steroids to avoid addisons disease. Steroids should always be taken with food.

==================================================

Question: Steroids and immunosuppresants

Answer: Lopressor - It can treat high bp, heart failure, and chest pain. check patients apical heart rate before admin. Hold medication if HR less than 60 or SBP less than 90. Contact physican.

==================================================

Question: Metoprolol

Answer: Hydrazine (Apresoline)

==================================================

Question: Which IV medication can be used if a patient is in a hypertensive crisis with a heart rate of 50?

Answer: - swelling of tongue, lips, or face. Usually caused by ace inhibitors. This is a medical emergency. Pril (enalapril, lisinopril)
- There is almost always a question on this.
- First priority is airway. Most patients present as an allergic reaction (swollen face, lips, tongue) but are awake. A good way of checking if the patient has a patent airway is if they are speaking complete sentences. In severe cases, a patient will have respiratory symptoms (stridor, wheezing, SOB). GIVE EPI! If this is the case, get a physician immediately and prepare for intubation. There are rare cases where the doctor cannot pass the ET tube and the patient will require an emergency cric.
Most often medication used for treatment is IV BENADRYL, SALUMEDROL (STEROIDS), PEPCID (HISTAMINE BLOCKER), AND IV FLUIDS.

==================================================

Question: ANGIOEDEMA

Answer: Insulin regular - 15-30 minutes, fast acting
Humalog (lispro) - given with meals (short acting), usually smaller unit amount (1-5 unit)
Levemir (lantus) - given at night, long acting, usually large unit amount (10-20 units)
There almost always is a select all that apply question about mixing insulin, even though in clinical practice we NEVER mix insulin. Remember, inject air into cloudy, then inject air into clear, draw clear, then draw cloudy. They try to trick you with these questions and one of the answers will not include (use an alcohol wipe) and that will not be one that you will select.

==================================================

Question: In the most severe case, a patient may be in anaphylactic shock (low blood pressure) and require 2 large bore IV's and a lot of IV fluids and possibly pressors

Answer: For some reason NCLEX focuses a lot on tic bites.
- Know the signs and symptoms of tic bite/lyme disease.
Bullet eye rash (look up other symptoms)


Also know what type of clothing to wear when walking in the forest (you want to wear light colored clothes so the tic does not mistake you for a tree)

==================================================

Question: INSULIN:

Answer: Answer: 1
Rationale: Lyme disease is a multisystem infection that results from a bite by a tick carried by several species of deer. Persons bitten by Ixodes ticks can be infected with the spirochete Borrelia burgdorferi. Lyme disease cannot be transmitted from one person to another. Toxoplasmosis is caused from the ingestion of cysts from contaminated cat feces. Histoplasmosis is caused by the inhalation of spores from bat or bird droppings.

==================================================

Question: Know the difference of short acting and long acting insulin:

Answer: Answer: 2
Rationale: The hallmark of stage I is the development of a skin rash within 2 to 30 days of infection, generally at the site of the tick bite. The rash develops into a concentric ring, giving it a bullseye appearance. The lesion enlarges up to 50 to 60 cm, and smaller lesions develop farther away from the original tick bite. In stage I, most infected persons develop flu-like symptoms that last 7 to 10 days; these symptoms may reoccur later. Neurological deficits occur in stage II. Arthralgias and joint enlargements are most likely to occur in stage III.

==================================================

Question: TIC BITES

Answer: Answer: 4
Rationale: A blood test is available to detect Lyme disease; however, the test is not reliable if performed before 4 to 6 weeks following the tick bite. Antibody formation takes place in the following manner: immunoglobulin M is detected 3 to 4 weeks after Lyme disease onset, peaks at 6 to 8 weeks, and then gradually disappears; immunoglobulin G is detected 2 to 3 months after infection and may remain elevated for years. Options 1, 2, and 3 are incorrect.

==================================================

Question: The camp nurse prepares to instruct a group of children about Lyme disease. Which of the following information would the nurse include in the instructions? 1. Lyme disease is caused by a tick carried by deer.
2. Lyme disease is caused by contamination from cat feces.
3. Lyme disease can be contagious by skin contact with an infected individual.
4. Lyme disease can be caused by the inhalation of spores from bird droppings.

Answer: Answer: 3
Rationale: In the prevention of Lyme disease, individuals need to be instructed to use an insect repellent on the skin and clothes when in an area where ticks are likely to be found. Long-sleeved tops and long pants, closed shoes, and a hat or cap should be worn. If possible, one should avoid heavily wooded areas or areas with thick underbrush. Socks can be pulled up and over the pant legs to prevent ticks from entering under clothing.

==================================================

Question: The client is diagnosed with stage I of Lyme disease. The nurse assesses the client for which characteristic of this stage? 1. Arthralgias
2. Flu-like symptoms
3. Enlarged and inflamed joints
4. Signs of neurological disorders

Answer: - These are usually pretty straight forward but the questions are usually regarding psychiatric patients.
Ex - the monsters are going to get me!
- Bring the patient back to reality and say I am a nurse and you are in the hospital.
Ex - a mom is in the lobby of a clinic and her children (age 2 and 4) are running around the waiting room screaming and playing. Best nursing response "children can be tough to handle at that age". Don't say something stupid like "control your children" or something like that.
Pretty much never say anything that can offend someone. If you don't know the answer, get someone who does.

==================================================

Question: A female client arrives at the health care clinic and tells the nurse that she was just bitten by a tick and would like to be tested for Lyme disease. The client tells the nurse that she removed the tick and flushed it down the toilet. Which of the following nursing actions is appropriate? 1. Refer the client for a blood test immediately.
2. Inform the client that there is not a test available for Lyme disease.
3. Tell the client that testing is not necessary unless arthralgia develops.
4. Instruct the client to return in 4 to 6 weeks to be tested, because testing before this time is not reliable.

Answer: It is ok to milk a chest tube, all that means is you raise the tubing (like a foley) and drain the drainage into the device.
When a chest tube is connected to suction, there should be no bubbles or fluctuation
When a chest tube is connected to gravity (no suction), bubbles and fluctuation (rise and fall of the little ball) are expected.
Sample question - a patient with a chest tube develops sudden shortness of breath. After applying oxygen via NRB and contacting the physician, the nurse should anticipate what? Stat portable chest xray to check for pneumothorax

==================================================

Question: A Cub Scout leader who is a nurse is preparing a group of Cub Scouts for an overnight camping trip and instructs the scouts about the methods to prevent Lyme disease. Which statement by one of the Cub Scouts indicates a need for further instructions? 1. "I need to bring a hat to wear during the trip."
2. "I should wear long-sleeved tops and long pants."
3. "I should not use insect repellent because it will attract the ticks."
4. "I need to wear closed shoes and socks that can be pulled up over my pants."

Answer: Once you have a few patients on a vent they are really easy, but for students they seem confusing. The best way to remember is by using the DOPE pneumonic.
When dealing with an intubated patient, when something suddenly changes, that change is usually not a positive change. When the intubated patient suddenly becomes worse, we need to identify and correct the problem quickly. The patient is somehow suffocating.

==================================================

Question: THERAPEUTIC COMMUNICATION

Answer: D - DISLODGED - the ET tube is not in the right place. Either the patient pulled the tube or somehow it became dislodged. In this case you would immediately use bag valve mask and call a rapid response
O - OBSTRUCTED - the tube is either kinked or the patient has a mucus plug. Suction the patients ET tube.
P - PNEUMOTHORAX - this is scary because usually it is a tension pneumo. The patient will not have breath sounds on the affected side. If it is a tension, that means the air pressure is so high from the collapsed lung that it is pushing the heart, major blood vessels, and trachea to the other side. Call a doctor immediately. Treatment is needle decompression or chest tube.
E - EQUIPMENT - either the ventilator is not working or the ventilator unhooked from the patients tube. If the vent isn't working right or becomes unhooked, bag the patient with high flow oxygen.

==================================================

Question: What is the best nursing response?

Answer: This question is always asked
1. Reposition to left lateral position
2. Apply high flow O2 (NRB)
3. IV fluid bolus
4. Contact physican
5. If the patient is getting IV pitocin, you would first stop the infusion

==================================================

Question: CHEST TUBES

Answer: These questions are asked a lot and is also important because a patient with ruptured ectopic is probably one of the sickest patients you will ever see. They will decline fast.
Symptoms are severe, sharp abdominal pain out of nowhere. The problem is the patient is bleeding internally at a rapid rate.
1. Establish 2 large bore IV's and administer 0.9 NS
2. Apply high flow O2
3. Place in reverse trendelenberg
4. Confirmed by ultrasound
5. The patient will usually be rushed to surgery

==================================================

Question: VENTILATORS

Answer: Previa is painless bleeding and abrupto is associated with severe pain

==================================================

Question: DOPE

Answer: When was your last period. We ask this because we want to determine due date. If the child is premature, less than 22 weeks, steroids are given along with surfactant to help lung development - dexamethasone

==================================================

Question: OB -

Answer: used to induce labor and also used to stop uterine bleeding after labor because it contracts the uterus

==================================================

Question: Monitor is showing late decels, what do you do first

Answer: contraindicated in patients with high blood pressure, used to tx severe bleeding from the uterus after childbirth

==================================================

Question: Ruptured ectopic pregnancy

Answer: contraindicated in patients with tachycardia (elevates heart rate).. delays preterm labor

==================================================

Question: PLACENTA PREVIA VS ABRUPTO

Answer: used for mothers if they have both high blood pressure and tachycardia

==================================================

Question: PREMATURE LABOR:

Answer: -variable decelerations
-You may have to place your hand in the vagina and put your fingers between the cord and the fetus' neck. Have the woman lay on her back and bend her knees towards her chest

==================================================

Question: A patient comes in with contractions and her water broke. What question should the nurse ask?

Answer: Initiate seizure precautions. Low sodium levels commonly cause seizures
Most patients with low sodium have congestive heart failure. The patient retains a lot of fluid which dilutes the sodium. Correction of this is fluid restriction. Do not give more 0.9 normal saline, it will not help correct the electrolyte problem and will most likely make the patient worse due to more fluids in the lungs and rest of the body.

==================================================

Question: Know your drugs

Answer: severe dehydration (nursing home patients) 0.45 ns

==================================================

Question: Pitocin

Answer: Place on cardiac monitor. Patients with high or low potassium are at risk for cardiac arrhythmias.

==================================================

Question: Methergine

Answer: is insulin, glucose, calcium, kayexelate, and albuterol.

Insulin, calcium, and albuterol all cause the potassium to go into the cells.

The glucose is given to make sure the patient doesn't become hypoglycemic. Give the glucose first then the insulin.

Check the patients blood sugar 15 minutes after administration.

==================================================

Question: Terbutaline

Answer: When a patient has low calcium, we always give calcium gluconate IVPB over 2 hours. Calcium chloride is only used in cardiac arrest. If we give a patient calcium chloride fast IVP, we can cause asystole.

==================================================

Question: Magnesium Sulfate

Answer: Low magnesium levels can cause VTACH. If a patient has VTACH, always check a mag level.
Here is an example of a critical ICU patient. In this patient, we would give potassium (K-rider) 40meq IVPB over 4 hours and magnesium 4grams over 4 hours.

1.6 - 2.6 mg/dL

==================================================

Question: PROLAPSED CORD

Answer: The first symptom these patients present with is nausea/vomiting

==================================================

Question: FLUIDS/ELECTROLYTES:

Answer: Always questions on this

==================================================

Question: A patient has a sodium level of 117 (normal 135-145). What is the most appropriate nursing intervention?

Answer: nausea, vomiting, sweating, and tachycardia.
In the most severe form the patient will have altered mental status, lethargic, or possible coma.

==================================================

Question: High sodium level give?

Answer: is IV fluids (a lot, sometimes 3-4L or more) and IV insulin.
The problem with these patients are that there blood is very acidic.
An arterial blood gas will be done to determine the patients PH.

==================================================

Question: Potassium - always questions on potassium

Answer: The patient will have frequent potassium checks and will most likely need supplemental potassium

==================================================

Question: Your patient has a potassium of 3.1 (low), what is the first nursing intervention?

Answer: At highest risk for suicide, especially in men
Highest risk of falls
Highest risk of depression

UTI in older adult will - altered mental status

==================================================

Question: Remember, a patient with high potassium is very sick and can go into sudden cardiac arrest or asystole.

Answer: Obtain a sterile urine sample.

- UTI in older adult often presents with similar symptoms as a stroke.

==================================================

Question: Treatment of high potassium:

Answer: - Most common in older adults
- Patient will present with extreme pain
- Affected leg will be externally rotated and will be shorter than the other. Always check a pedal pulse and cap refill to make sure a major artery is not being obstructed by the fracture.
- Patients with hip fracture are at highest risk for fat embolism

==================================================

Question: With low potassium, we usually infuse potassium over 4 hours IVPB.

Answer: Know your stages of ulcers.

==================================================

Question: The rule is a patient can receive potassium 10meq/hr through a peripheral IV and 20meq/hr through a central line. It must always be on a pump.
Rapid infusion of potassium will cause asystole.

Answer: Skin is intact with non blanchable areas of redness, skin feels warm, firm, painful. This stage may be hard to see in patients with darker skin tones.

==================================================

Question: Never give potassium IV push - this will be a test question

Answer: Shallow open ulcer with red/pink wound bed. No slough is present. Stage 2 ulcers can also be seen as intact or open blisters, abrasions, or shallow craters

==================================================

Question: Calcium:

Answer: Full thickness skin loss with some subcutaneous fat may be visible but bone, tendon, or muscle tissue are not exposed. Stage 3 may include undermining and tunneling.

==================================================

Question: Magnesium

Answer: Full thickness loss with bone, tendon, muscle involvement. Slough/eschar may be present, tunneling is usually present.

==================================================

Question: LITHIUM/DILANTIN TOXICITY/DIGOXIN

Answer: Full thickness tissue loss where base of the ulcer is covered by slough or eschar. There is no way to tell what is underneath the covered surface.

==================================================

Question: DIABETIC KETOACIDOSIS

Answer: 9% front 4.5, back 4.5

==================================================

Question: Typical symptoms are:

Answer: 9% each (4.5% for anterior/4.5% for posterior)

==================================================

Question: Treatment:

Answer: 18% each, anterior leg 9, posterior leg 9

==================================================

Question: ***REMEMBER - a patient receiving IV insulin or on an insulin drip is at high risk for having a low potassium level (remember insulin is given for patients with high potassium).

Answer: 18% Chest 9, Abdomen 9

==================================================

Question: GERIATRIC/OLDER ADULTS:

Answer: Back trunk 18% upper back 9, lower back 9

==================================================

Question: A 85 year old male comes in with altered mental status, slurred speech, and muscle weakness. A stroke alert is called. After the patient is stable and the CT scan is complete, what is the next priority nursing intervention?

Answer: 1%

==================================================

Question: HIP FRACTURE:

Answer: - temperature control, infection prevention, and IV fluids
- The patient will be at high risk for infection so cover with sterile dry sheets
- 2 large bore IV's

==================================================

Question: PRESSURE ULCERS:

Answer: lactated ringers, NOT 0.9 NS

==================================================

Question: Stage 1-

Answer: pt must be intubated immediately even if they appear normal and are talking.

- These patients have laryngeal edema and there airway will close in a matter of minutes

==================================================

Question: Stage 2-

Answer: Common side effects are tachycardia and shakiness. Tell the patient this is expected

==================================================

Question: Stage 3-

Answer: NEBULIZER

have the mother assist in administering blow by oxygen

Do not stimulate the child, child may get worse

==================================================

Question: Stage 4-

Answer: poor skin turgor, sunken fontanels, less than 8 wet diapers/day

==================================================

Question: Unstageable

Answer: hydrocephalus or meningitis. This child should be seen right away.

==================================================

Question: BURNS: Know your rule of 9's - guaranteed question

Answer: Most commonly cause by ear infections. 50% of children will have a febrile seizure.
Tylenol/motrin for fever and remove childs clothes

- If the temp is very high, >104, ice packs may be applied

==================================================

Question: Head and Neck =

Answer: cause enamel hypoplasia and tooth discoloration in children under 8 years of age

==================================================

Question: Each arm =

Answer: of the person, the environment or situation, and health

==================================================

Question: Each leg =

Answer: Clients with parkinsons disease frequently experience difficulty in initiating, maintaining, and performing motor activities
They may even experience being rooted to one spot and unable to move

==================================================

Question: Front trunk =

Answer: The best indicator that the tubing is properly placed is a chest x-ray
Other methods include auscultation and aspiration of GI contents, but x-ray is the best

==================================================

Question: Back trunk =

Answer: Completes education and becomes self supporting

==================================================

Question: Genitalia =

Answer: - Demonstrate a conceptualization of death and dying

-Creates a new definition of self and roles with others

==================================================

Question: Priorty for burn patient:

Answer: Develops a strong need for parental support and approval

==================================================

Question: The IV fluid used for burns:

Answer: Always carry a medic alert card stating that he is a total neck breather
If they have cardiac arrest, mouth to neck breathing can be done

==================================================

Question: Remember that a patient with burns to the chest will shortly develop eschar and will not be able to breath.

Answer: The therapeutic level is 10-20 mcg/dl ***usually on NCLEX

==================================================

Question: Singed nasal hairs or soot in the mouth or nose :

Answer: Reposition the client, provide oxygen via face mask, increase IV fluid, call the healthcare provider
To stabilize the fetus, intrauterine resuscitation is the first priority, and to enhance fetal blood supply

==================================================

Question: Albuterol:

Answer: When working in group therapy, if there is a client who continually talks over the group
After several weeks, the group is in the working phase and the group members should be allowed to determine the direction of the group
The nurse should ignore the client's comments and allow the group to handle the situation
A good leader should not have separate meetings with group members
Remember, identify what phase the group is in (initial, working, or termination) and this will help determine communication style

==================================================

Question: PEDIATRICS:

Answer: The four elements of malpractice are breach of duty, failure to adhere to the recognized standard of care, direct causation of injury, and evidence of actual injury (you must have all 4)

==================================================

Question: Any child who has nasal flaring or grunting needs immediate attention

Answer: Beta 1 adrenergic agonist that is indicated for short term use in cardiac decompensation or heart failure
Reduced cardiac contractility due to organic heart disease or cardiac surgical procedures
Alpha and beta agonists, such as epinephrine and dopamine

==================================================

Question: Any child who is drooling needs immediate attention (epiglotitis)

Answer: A - notfity physican immedately
B- take patient for stat cat scan
C- start IV fluids at 150/hr
D- Give pt PRN tylenol

==================================================

Question: Racemic epinephrine -

Answer: Never give to someone with low blood pressure

left sided heart failure - SOB, pulm edema, rales/crackles (VERY HIGH BLOOD PRESSURE)

give nitro tabs or nitro drip and possibly DOBUTAMINE

ride sided heart failure - edema in feet/lets, JVD, swollen liver,

never give nitro to a right sided MI → give IV fluids possibly dopamine

==================================================

Question: Common signs of dehydration:

Answer: wear gloves when touching a patient, use this for everyone

==================================================

Question: Bulging fontanels:

Answer: gown and gloves - > used for patients with MRSA or other skin infections.

==================================================

Question: Febrile seizures :

Answer: CDIFF

==================================================

Question: PRIORITY QUESTIONS (WHO NEEDS TO BE SEEN FIRST)

These are always on NCLEX

Always go with airway first

The answer is usually the person with difficulty breathing or the patient with chest pain.

Answer: influenza and RSV

==================================================

Question: ADDITIONAL SUBJECTS AND IMPORTANT INFO:

Answer: chicken pox, active shingles or TB or ebola

==================================================

Question: Tetracycline:

Answer: - clot buster

A patient comes in who is unresponsive with left facial droop and left sided paralysis. When asking the family when the patient was last seen normal, they respond before going to bed 4-5 hours ago.

==================================================

Question: Nursing theories serve to describe, explain, predict, or prescribe nursing care measures.

Answer: reasons for immediate cat scan and stopping TPA
headache
change in mental status for the worse
change in pupil size
any other negative changes in neuro exam
neuro assessments Q 15

==================================================

Question: The domain of nursing theory consists of :

Answer: NORMAL PH - 7.35-7.45
O2 - 80-100
CO2 - 35-45
HCO3 - 22-28

==================================================

Question: Delusions often generate fear and isolation, so the nurse should help the client participate in activities that avoid focusing on the false belief and encourage interaction with others

he nurse should reassure the client that he/she is in a safe place

So, if the client is delusional, try to encourage him/her to go to occupational therapy and start a project

Answer: COPD patients drive to breathe is low oxygen, so if you give more, they will stop

==================================================

Question: PARKINSONS:

Answer: NUCHAL RIGIDITY, FEVER, BLURRED VISION, ALTERED MENTAL STATUS

==================================================

Question: NG TUBE:

Answer: BACTERIAL IS VERY CONTAGIOUS, SYMPTOMS HAPPEN VERY FAST, RAPID DETERIATION, WIDE SPREAD ABX

==================================================

Nclex Isolation Precautions

Question: Standard Precautions:

Answer: 1. wash your hands (*Most important step in infection control, prevents nosocomial infections)
2. DON gloves (before coming in contact with anything wet, i.e. broken skin, mucous membranes, blood, body fluids, soiled instruments, contaminated waste materials.
3. wash hands again upon removal of gloves and between patients

==================================================

Question: Contact Precautions:

Answer: Before entering:
1.wash hands
2. DON gown then gloves

Upon entering:
1. use disposable equipment when possible
2. when not available clean and disinfect all equipment before removing from room

Before leaving the patient's room:
1. Remove gloves then gown
2. Wash hands

==================================================

Question: Contact Precautions Transporting patient:

Answer: 1. PT should perform hand hygeine and wear a clean gown
2. For direct contact with pt, nurse or care provider should wear a gown and gloves.
3. Notify receiving area

==================================================

Question: Common Microorganisms which require Contact Precautions :

Answer: 1. Antibiotic Resistant Organisms (Methicillin resistant Staphylococcus aureu (MRSA), Extended spectrum beta-lactamase (ESBL), Penicillin resistant Streptococcus pneumoniae (PRSP), Multi-drug resistant Pseudomonas aeruginosa (MDRP))
2. Scabies
3. Herpes Zoster (Shingles) localized
4. Diarrhea, Clostrididum difficile

==================================================

Question: Airborne Contact Precautions:

Answer: Before entering:
1.Wash hands
2. Don N95 Respirator (Mask)
3. Don gown then gloves

*Negative Pressure Isolation Room KEEP DOOR CLOSED

Before leaving pt's room:
1. Remove gloves then gown NOT N95 mask
2. Wash hands

After leaving pt's room:
1. Shut door
2. Wash hands
3. Remove N95 mask
4. Wash hands

==================================================

Question: Airborne Contact Precautions Transporting patient:

Answer: 1. Patient must wear a surgical or procedure mask and a clean gown
2. Patient must wash hands
3. For direct contact with pt, nurse or care provider should wear a gown and gloves.
4. Notify receiving area

==================================================

Question: Common Microorganisms which require Airborne Contact Precautions :

Answer: 1. Measles (Rubeola)
2. Tuberculosis (TB)
3. Chicken Pox (Varicella-Zoster virus)
4. Herpes Zoster (Shingles) disseminated

==================================================

Question: Droplet Contact Precautions:

Answer: Before Entering:
1. Wash Hands
2. DON Mask and Eye Protection
3. DON Gown then Gloves

Before Leaving Pt's Room:
1. Remove gloves then gown
2. Wash Hands
3. Remove eye protection and mask
4. Wash Hands

==================================================

Question: Droplet Contact Precautions Transporting patient:

Answer: 1. Pt must perform hand hygeine
2. Pt must wear a surgical or procedure mask and clean gown
3. For direct contact with pt, nurse or care provider should wear a gown and gloves.
4. Notify receiving area

==================================================

Question: Common Microorganisms which require Droplet Contact Precautions :

Answer: 1. Influenza (Flu)
2. Viral Respiratory tract infections (adenovirus, parainfluenza, rhinovirus, RSV)
3. Streptococcus group A pharyngitis, pneumonia, scarlet fever
4. Neisseria meningitidis invasive infections
5. H. Influenzae type b invasive infections
6. Pertussis
7. Rubella
8. Mumps

==================================================

Question: Strict isolation is:
Rules:

Answer: used to prevent the transmission of all highly communicable diseases that are spread by both, contact or airborne routes of transmission. Examples of such diseases are chickenpox and rabies

(1) Visitors must report to the nurses' station before entering the room.
(2) Door must be kept closed.
(3) Gowns must be worn by all persons entering the room.
(4) Masks must be worn by all persons entering the room.
(5) Hands must be washed on entering and leaving the room.
(6) Gloves must be worn by all persons entering the room.
(7) Articles must be discarded or wrapped before being sent to Central Supply for disinfection or sterilization.

==================================================

Question: Respiratory isolation is:
Rules

Answer: used to prevent transmission of organisms by means of droplets that are sneezed or breathed into the environment. Examples of such diseases are influenza and tuberculosis

(1) Visitors must report to the nurses' station before entering the room.
(2) Door must be kept closed.
(3) Gowns are not necessary.
(4) Masks must be worn by any person entering the room unless that person is not susceptible to the disease.
(5) Hands must be washed on entering and leaving the room.
(6) Gloves are not necessary.
(7) Articles contaminated with secretions must be disinfected.

==================================================

Question: Protective isolation is:
Rules:

Answer: used to prevent contact between potentially pathogenic microorganisms and uninfected persons who have seriously impaired resistance. Patients with certain diseases, such as leukemia, who are on certain therapeutic regimens are significantly more susceptible to infections

(1) Visitors must report to the nurses' station before entering the room.
(2) Door must be kept closed.
(3) Gowns must be worn by all persons entering the room.
(4) Masks must be worn by all persons entering the room.
(5) Hands must be washed on entering and leaving the room.
(6) Gloves must be worn by all persons having direct contact with the patient.
(7) Articles must be handled according to local SOP.

==================================================

Question: Enteric precautions are:
Rules:

Answer: used to control diseases that can be transmitted through direct or indirect oral contact with infected feces or contaminated articles. Transmission of infection depends on ingestion of the pathogen. Examples of diseases requiring enteric precautions are dysentery and hepatitis.

(1) Visitors must report to the nurses' station before entering the room.
(2) Gowns must be worn by all persons having direct contact with the patient.
(3) Masks are not necessary.
(4) Gloves must be worn by all persons having direct contact with the patient or with articles contaminated with fecal material.
(5) Special precautions are necessary for articles contaminated with urine and feces. Articles must be disinfected or discarded

==================================================

Question: Wound and skin precautions are:
Rules:

Answer: used to prevent the spread of microorganisms found in infected wounds (including burns and open sores) and contact with wounds and heavily contaminated articles. Conditions requiring these precautions include infected burns, infected wounds, and infections with large amounts of purulent discharge. Diseases that may require wound and skin precautions include herpes, impetigo, and ringworm.

(1) Visitors must report to the nurses' station before entering the room.
(2) Gowns must be worn by all persons having direct contact with the infected wound.
(3) Masks are not necessary except during dressing changes.
(4) Gloves must be worn by all persons having direct contact with the infected area.
(5) Special precautions are necessary for instruments, dressings, and linens.
CAUTION: Only hospital personnel who have been vaccinated with poliomyelitis vaccine should have direct contact with patients who have active poliomyelitis.

==================================================

Question: Blood precautions are:
Rules:

Answer: used to prevent acquisition of infection by patients and personnel from contact with blood or items contaminated with blood. Examples of diseases that require blood precautions (refer to Lesson 1) are HBV and HIV/AIDS

==================================================

Question: Secretion precautions-lesions are:

Answer: used to prevent acquisition of infection by personnel and patients from direct contact with wounds and secretion-contaminated articles. Some examples of diseases requiring these precautions are conjunctivitis, gonorrhea, and syphilis

==================================================

Question: Secretion precautions-oral are:

Answer: used to prevent acquisition of infection by personnel from direct contact with oral secretions. Some examples of diseases requiring these precautions are herpes areolas and scarlet fever.

==================================================

Question: Excretion precautions are:

Answer: used to prevent acquisition of infection by personnel and patients from direct contact with fecal excretions. Some examples of diseases requiring these precautions are poliomyelitis and staphylococcal food poisoning.

==================================================

Question: If it's not one of these 4 diseases and you know it's HIGHLY contagious, it's probably droplet, so know the 4 airborne are!

Answer: (think: My Chicken Has TB)
measles
chicken pox
Herpes zoster aka shingles
TB

==================================================

Question: airborne precautions, room considerations

Answer: Remember: Private room or cohort w/ negative pressure w/ 6 - 12 air exchange/hr AND MASK. N95 mask for TB.

==================================================

Question: Droplet precautions

Answer: SSS PPP IDER MMM AN

S = Sepsis
S = Scarlet Fever
S = Streptococcal Pharyngitis
P = Paravirus B19
P = Pneumonia
P = Pertussis
I = Influenza
D = Diptheria (Pharyngeal)
E = Epiglottitis
R = Rubella
M = Mumps
M = Meningitis
M = Mycoplasma or Meningeal Pneumonia
AN = Adenovirus

==================================================

Question: CONTACT precautions

Answer: MRS WEE

M = Multidrug Resistant Organism
R = Respiratory Infection
S = Skin Infections
W = Wound Infections
E = Enteric Infection (Clostridium Difficile)
E = Eye Infections (Conjunctivitis)

==================================================

Question: SKIN Infections:

Answer: V CHIPS

V-CHIPS
V = Varicella
C = Cutaneous Diptheria
H = Herepes Zoster
I = Impetigo
P = Pediculosis
S = Scabies

==================================================

Question: AIRBORNE PRECAUTION:
<5 microns
Private room with closed door
negative pressure room/ 6-12 exchange/hr
PPE: N95, Hepa Filter
DSE: (Remember MTV Hz)
Measle
TB/ TB suspect
Varicella
Herpes Zoster
SARS

DROPLET PRECAUTION:
>5 microns
Private room
YES to cohort, 3ft separation
PPE: Surgical Mask
DSE: (Remember "Dropletism")
Diptheria
Rubella
Oral Pharyngitis
Pertussis/ Pneumonia
Legionnaire's Disease
Erythema infectiosum (5th disease) -contagious when no rash
Tonsilitis
Influenza
Scarlet Fever
Mumps/ Meningitis

CONTACT PRECAUTION:
Private room
YES to cohort, 3ft separation
Secretions: skin wounds, eyes, nose, ears
Patient has own BP/Stet equipment
PPE: Gloves and Gown
DSE: RSV, MRSA, VRSA, VRE, CDAD, STD, Conjunctivitis, Impetigo

ENTERIC PRECAUTION:
Private room with bathroom facility
YES to cohort
PPE: Gloves and Gown
DSE: Hep A, Shigella, Salmonella, Norwalk virus (improper/ contaminated food)
Crytosporidiosis, Gardiasis (common in day care/ children)

NEUTROPENIC PRECAUTION:
Private room
PPE: Mask
DSE: Chemo/ Cancer patients, AIDS/HIV, Major burns/ surgery,
SLE, Transplant patient taking 4C's (Cellcept, Corticoidsteroids, Cytoxan, Cyclosporin)

Answer: note it's in alpha order

==================================================

Question: order in which the nurse should remove personal protective equipment(ppe)

gloves,
goggles
gown
mask

Answer: C-diff
Dysentery from infectious cause
Hepatitis A

==================================================

Nclex Leadership Questions

Question: A registered nurse reviews a plan of care developed by a nursing student for a client with depression and notes a nursing diagnosis of impaired nutrition: less than body requirements. The registered nurse asks the student to revise the plan if which incorrect intervention is documented?

a) offer small, high-calorie, high protein snacks frequently throughout the day and evening
b) offer high protein, high-calorie fluids frequently throughout the day and evening
c) remain with the client during meals
d) complete the food menu for the client during the depressed period

Answer: D
- The client should be asked which foods or drinks she likes, and consultation with a dietitian also may be done. The client is more likely to eat if the client has selected the foods and is given foods that she likes. Options A, B, and C are appropriate interventions for the client with depression with this nursing diagnosis.

==================================================

Question: A registered nurse reviews a plan of care developed by a nursing student for client with paranoia and notes a nursing diagnosis of Disturbed thought process. The registered nurse asks the nursing student to revise the plan if which incorrect intervention is documented?

a) sit with the client and hold the client's hand
b) avoid a warm approach when working with the client
c) use simple and clear language when speaking to the client
d) diffuse angry and hostile verbal attacks with a nondefensive stand

Answer: A
- When caring for a paranoid client, the nurse must avoid any physical contact and should not touch the client. The nurse should ask the client's permission if touch is necessary because touch may be interpreted as a physical or sexual assault. The nurse would use simple and clear language when speaking to the client to prevent misinterpretation and to clarify the nurse's intent and actions. A warm approach is avoided because it can be frightening to a person who needs emotional distance. A matter-of-fact consistency is nonthreatening. Any anger and hostile verbal attacks need to be diffused with a nondefensive stand. The anger that a paranoid client expresses is often displaced, and when the staff becomes defensive, anger of both the client and staff escalates. A nondefensive and nonjudgmental attitude provides an attitude in which feelings can be explored more easily.

==================================================

Question: A registered nurse is discussing the characteristics of anorexia nervosa with a nursing student. The registered nurse determines that the nursing student needs to further research this disorder if the student states that which of the following is a characteristic of anorexia nervosa?

a) personal relationships tend to become more superficial and distant
b) social contacts are avoided because of the fear of being invited to eat and being discovered
c) the client is being preoccupied with food and meal planning, especially for others
d) the client will usually keep her weight near normal

Answer: D
- As anorexia nervosa develops, personal relationships tend to become more superficial and distant. Social contacts are avoided because of the fear of being invited to eat and being discovered. The client is preoccupied with food and meal planning (especially for others), personal caloric intake throughout the day, and methods to avoid eating. Anorexic persons are likely to become very emaciated and will not maintain their near-normal body weight.

==================================================

Question: An experienced emergency department nurse observes a new nurse employed in the emergency department obtain the equipment needed to draw a blood sample for a blood alcohol level on a client. The experienced emergency department nurse intervenes if the new nurse plans to use which item?

a) tourniquet
b) alcohol swabs
c) a blood-draw needle
d) a blood tube

Answer: B
- Isopropyl alcohol or any antiseptic solution containing alcohol must not be used as a skin preparation before a blood alcohol specimen is drawn. These agents may falsely elevate the blood alcohol level and render the test invalid. Option A, C and D identify items needed to obtain the blood specimen.

==================================================

Question: A nurse administers digoxin (Lanoxin) 0.25 mg instead of the prescribed order of 0.125 mg. The nurse discovers the error while charting the medication. The nurse completes an incident report and notifies the physician of the incident. The nurse takes which additional action?

a) gives the client a copy of the incident report
b) makes a copy of the incident report and sends it to the physician's office
c) documents the incident in the client's record
d) places the incident report in the client's record

Answer: C
- The incident report is confidential and privileged information. It should not be copied or placed in the chart or have any reference made to it in the client's record. It is the physician's responsibility to sign the incident report before it is sent to the risk-management department. A copy should not be made or sent to the physician's office. The incident report is not a substitute for a complete entry in the client's record concerning the incident. A copy of the incident report is not given to the client; however, the client should be informed of the error, and this is usually done by the client's physician.

==================================================

Question: A registered nurse is supervising a new nursing graduate who is performing an irrigation on an assigned client with a buildup of cerumen in the left ear. Which of the following observations if made by the registered nurse would indicate that the nursing graduate is performing the procedure correctly?

a) the client is positioned with the ear to be irrigated facing upward
b) the irrigating solution is warmed to 100F
c) a direct and slow steady stream of irrigation solution is directed toward the eardrum
d) the client is positioned with the affected ear up following the irrigation

Answer: B
- Irrigation solutions that are not close to the client's body temperature can be uncomfortable and may cause injury, nausea, and vertigo. The client is positioned so that the ear to be irrigated is facing downward because this allows gravity to assist in the removal of the cerumen and solution. Following the irrigation, the client is to lie on the affected side for a period to finish the drainage of the irrigating solution. A slow, steady stream of solution should be directed toward the upper wall of the ear canal and not toward the tympanic membrane. Too much force could cause the tympanic membrane to rupture.

==================================================

Question: A nurse is performing a sterile wound irrigation on an assigned client. A nursing assistant enters the client's room and tells the nurse that a physician has telephoned and requests to speak to the nurse. The appropriate nursing action is which of the following?

a) finish the wound irrigation while the physician waits on the telephone
b) cover the client and answer the telephone call
c) ask the nursing assistant to obtain a telephone number from the physician so that the call can be returned after the wound irrigation
d) ask the nursing assistant to take a message

Answer: C
- Because wound irrigation is a sterile procedure and a risk for infection exists with an open wound, it is most appropriate to ask the nursing assistant to obtain a telephone number from the physician so that the call can be returned. It is not appropriate to ask a physician to wait while a procedure is being completed. It is best to return the call. Option D is not a responsibility of the nursing assistant.

==================================================

Question: A case manager is reviewing the records of the clients in the nursing unit. Which of the following documentation, if noted in a client's record, would the nurse indicate as a positive variance?

a) a client in skeletal traction has a temperature of 98.6F and the pin sites are clean and dry
b) a postoperative client is performing coughing and deep-breathing exercises every hour
c) a client with congestive heart failure has clear breath sounds
d) a client with pneumonia is discharged to home 1 day earlier than expected

Answer: D
- Variances are actual deviations or detours from the critical path. Variances are either positive or negative and avoidable or unavoidable, and may be caused by a variety of things. A positive variance occurs when the client has achieved maximum benefits and is discharged earlier than anticipated on her critical path. Option 4 is the only option that specifically identifies a positive variance. Options A, B, and C demonstrate progression on a critical path, but they are not specifically associated with the definition of a positive variance.

==================================================

Question: A nurse is a member of a community task force on violence. The task force recognizes that it has insufficient data to make decisions about specific interventions. Using the nursing process, the first activity that the nurse would suggest to the task force is to:

a) call other communities similar in size to determine what they do
b) develop a general educational program related to violence
c) conduct a community survey to assess community perceptions regarding violence
d) develop a pamphlet on violence to be distributed to the community

Answer: C
- An assessment activity is always the first step in the nursing process. Option C addresses assessment of community perceptions. Option A is a part of analysis from a variety of assessment data, but is not specific to the subject of the question. Options B and D are implementation measures.

==================================================

Question: A community health nurse has been assigned to be the leader of a task force to identify interventions for teenagers from a local community who are abusing drugs. At the first meeting of the task force, the members express concern that more information is needed to determine appropriate measures for the target teenagers. The nurse would direct the group effectively by suggesting which of the following?

a) preparing a survey that can be distributed to community members to determine their understanding of the drug abuse problem
b) initiating a drug abuse program in all of the schools
c) seeking out the teenage drug abusers and referring them to drug abuse centers
d) preparing posters that can be distributed to the schools

Answer: A
- Option A is the only option that addresses the subject of the question and will identify the additional information required by the task force. Options B, C, and D do not provide the additional information required in order for the task force to proceed with the necessary task of the group.

==================================================

Question: 1. A registered nurse is discussing treatment for a client who is hospitalized with acute systemic lupus erythematosus (SLE) with a nursing student assigned to the client. The registered nurse realized that the nursing student needs to research information about the disease if the student states that which of the following is a clinical manifestation of SLE?

a) fever
b) bradycardia
c) butterfly rash on the face
d) muscular aches and pains

Answer: B
- Manifestations of acute SLE may include fever, musculoskeletal aches and pains, butterfly rash on the face, pleural effusion, basilar pneumonia, generalized lymphadenopathy, pericarditis, tachycardia, hepatosplenomegaly, nephritis, delirium, convulsions, psychosis, and coma.

==================================================

Question: A nurse administers a fatal dose if a cardiac medication to a client. During the subsequent investigation, it was determined that the nurse did not check the client's vital signs before administering the medication. This failure to complete an appropriate assessment is addressed under which function on the Nurse Practice Act?

a) defining the specific educational requirements for licensure
b) describing the scope of practice of licensed and unlicensed care providers
c) recommending disciplinary action for nurses who violate the law
d) identifying the process for disciplinary action if standards of care are not met

Answer: D
In the situation described in the question, acceptable standards of care were not met (the nurse failed to adequately assess the client before administering a medication). Option D refers specifically to the situation described. Options A, B, and C do not relate to standards of care.

==================================================

Question: A registered nurse is observing a nursing student prepare a client for a renal angiography. The registered nurse would intervene if the nursing student:

a) checked circulation to the client's legs
b) checked for client's allergies
c) ensured that an informed consent for the diagnostic procedure was signed
d) ensured that an anesthesia consent was signed

Answer: D
- Renal angiography involves injection of a contrast medium. Therefore, the procedure is invasive, and an informed consent for the diagnostic procedure needs to be signed. A local anesthetic may be used at the needle insertion site, but an anesthesia consent form is not necessary. The nurse checks for client allergies to determine whether the client has an allergy to the contrast medium. A baseline assessment of circulation to the legs is important to assist in monitoring for complications in the postprocedure period.

==================================================

Question: A nursing student is assigned to care for a child who has been placed in Crutchfield tongs to stabilize a fracture in the cervical area. The registered nurse reviews the plan of care developed by the student and discusses revising the plan if which incorrect intervention is documented?

a) logroll the child when positioning
b) check the tongs every 24 hours for displacement and looseness
c) monitor neurological status
d) perform pin care every shift

Answer: B
- The purpose of Crutchfield tongs is to stabilize fractures or displaced vertebrae in the cervical and thoracic areas. Tongs are inserted on the sides of the scalp through drill holes. Traction pull is always along the axis of the spine. The nurse should check the tongs at least every 8 hours and as needed (PRN) for displacement and looseness. The child can be repositioned by logrolling or turned as a unit. Neurological status should be checked frequently because spinal cord injury frequently accompanies a cervical injury. Pin care is done every shift.

==================================================

Question: A nurse is performing an admission assessment on a client admitted to the hospital with a diagnosis of fever of unknown origin. The nurse performs interventions based on the nurse practice act when the nurse:

a) enters the information on the client's record
b) writes the information on a worksheet
c) informs the supervisor of the client's vital signs
d) tells another nurse that the client has a high fever

Answer: A
- Recording assessment data reflects the requirement of the nurse practice act to maintain adequate records. Verbal information and notes on worksheets are not part of the client's permanent record.

==================================================

Question: A nurse educator at the local community hospital is conducting an orientation session for nurses that are newly employed at the hospital. The nurse educator informs the new nurses that the policy of the hospital requires that nurses "float" to other nursing departments when client census is high on other units. The nurse educator advises the new nurses that if this situation arises and if the nurse is unfamiliar with the unit in which the nurse must "float" to:

a) refuse to float
b) call the nurse educator
c) report to the unit and identify tasks that can be safely performed
d) call the nursing supervisor

Answer: C
- Floating is an acceptable legal practice used by hospitals to solve their understaffing problems. Legally a nurse cannot refuse to float unless union contract guarantees that nurses can only work in a specified area or the nurse can prove the lack of knowledge for the performance of assigned tasks. When encountered with this situation, nurses should report to the unit and identify tasks that can be safely performed. The nursing supervisor and the nurse educator may need to become involved in the situation at some point if the nurse requires assistance or education regarding a new skill, but the action that the nurse must take is identified in option C.

==================================================

Question: A staff nurse makes negative comments about a unit manager's leadership style, and the unit manager overhears the staff nurse. Which action by the unit manager would be appropriate?

a) tell the staff nurse to stop making the comments
b) propose a tentative solution regarding the comments, and discuss it with the staff nurse
c) encourage the staff nurse to discuss the comments
d) persuade the staff nurse to stop being so critical

Answer: C
- Encouraging the staff nurse to discuss the comments will assist in identifying the concerns in a democratic way. Options A and D are autocratic. Option B does not provide the opportunity for the staff nurse to directly share concerns.

==================================================

Question: A registered nurse is reviewing a plan of care developed by a nursing student for a client scheduled for keratoplasty. The registered nurse tells the nursing student that which intervention written in the plan needs to be deleted because it is incorrect?

a) obtain a specimen for culture and sensitivity of the eye with a conjunctival swab
b) instill antibiotic ophthalmic medication as prescribed
c) cut the client's eyelashes
d) administer medications that will dilate the pupil

Answer: D
- Keratoplasty is done by removing damaged corneal tissue and replacing it with corneal tissue from a human donor (live or cadaver). Preoperative preparation of the recipient's eye may include obtaining a culture and sensitivity with conjunctival swabs, instilling antibiotic ophthalmic medication, and cutting the eyelashes. Some ophthalmologists order a medication such as 2% pilocarpine to constrict the pupil (not dilate the pupil) before surgery.

==================================================

Question: A registered nurse is discussing the clinical manifestations of Meniere's disease with a nursing student. The registered nurse determines that the nursing student needs to further research this disorder if the student states that which of the following is a manifestation of Meniere's disease?

a) tinnitus
b) sensorineural hearing loss on the involved side
c) conductive hearing loss on the involved side
d) vertigo accompanied by nausea and vomiting

Answer: C
- The three characteristic symptoms of Ménière's disease are tinnitus, sensorineural hearing loss on the involved side, and severe vertigo accompanied by nausea and vomiting. Option C is not associated with Ménière's disease.

==================================================

Question: A registered nurse reviews a plan of care developed by a nursing student for a client who will be returning from the operating room following a mastoidectomy. The registered nurse informs the student that which intervention is incorrect?

a) monitor the client for pain, dizziness, or nausea
b) keep the head of the bed elevated
c) instruct the client to lie on the affected side
d) monitor for signs of injury to cranial nerve VII

Answer: C
- Following mastoidectomy, the nurse should monitor vital signs and inspect the dressing for drainage or bleeding. The nurse should monitor for signs of facial nerve injury to cranial nerve VII. The nurse should also monitor the client for pain, dizziness, or nausea. The head of the bed should be elevated, and the client should lie on the unaffected side. The client will probably have sutures, an outer ear packing, and a bulky dressing that is removed on approximately the sixth postoperative day

==================================================

Question: A community health nurse is working with a group of clients at risk for hypertension. In implementing interventions by levels of prevention, which of the following would be a primary prevention intervention that the nurse would use with this group?

a) Encouraging the clients to attend hypertension screening clinics
b) encouraging clients to visit their physician regularly
c) providing information regarding the decreased use of salt in the diet
d) conducting a community-wide screening to detect individuals with hypertension

Answer: C
- Primary prevention interventions are those measures that keep illness, injury, or potential problems from occurring; therefore, option C is correct. Options A,B, and D are secondary prevention measures that seek to detect existing health problems or trends.

==================================================

Question: A clinical nurse educator is conducting an educational session for new nursing graduates and is discussing standards of care. The nurse educator determines that a graduate understands the purpose of standards of care when the graduate states that standards of care:

a) identify methods of treatment based on the most current technology
b) provide excellent care based on current medical research
c) include providing competent levels of care based on current practice
d) include providing care based on specialty guidelines for the client's condition

Answer: C
- The purpose of standards of care is to provide a broad direction for the overall practice of nursing that applies to all nursing situations, across specialty areas, across the country. Standards of care include providing competent levels of care based on current practice. Options A, B, and D do not specifically describe standards of care. Option A is specific to technology. Option B addresses medical research. Option D addresses specialty guidelines.

==================================================

Question: A registered nurse (RN) is observing a new licensed practical nurse (LPN) suctioning a client who has a diagnosis of acquired immunodeficiency syndrome (AIDS). The RN would determine that the LPN was performing the procedure safely if the RN observed that the LPN was wearing which of the following protective devices?

a) gloves, mask, and protective eyewear
b) gloves, gown, and mask
c) gown, mask, and protective eyewear
d) gloves, gown, and protective eyewear

Answer: A
- The RN is responsible for supervising procedures performed by a new LPN to ensure that client safety is maintained and that policies and procedural guidelines are adhered to. Standard precautions include use of gloves whenever there is actual or potential contact with blood or body fluids. During suctioning the nurse wears gloves, a mask, and protective eyewear or a face shield. Impervious gowns are worn in those instances when it is anticipated that there will be contact with a large amount of body fluid or blood.

==================================================

Question: A registered nurse (RN) is observing a nursing assistant ambulating a client with right-sided weakness. The RN would determine that the nursing assistant was performing the procedure safely if the nurse observed the nursing assistant:

a) standing behind the client
b) standing in front of the client
c) standing on the left side of the client
d) standing on the right side of the client

Answer: D
- When walking with clients, the nurse should stand on the affected side and grasp the security belt in the midspine area of the small of the client's back. The nurse should position the free hand at the shoulder area so that the client can be pulled toward the nurse in the event that there is a forward fall. The client is instructed to look up and outward rather than at the feet. Options A,B, and C are incorrect.

==================================================

Question: A registered nurse (RN) is observing a new licensed practical nurse (LPN) administer a deep intramuscular injection in the dorsogluteal site of a client. The RN determines that the LPN is performing the procedure correctly if the LPN:

a) administers the injection 2 inches below the acromial process
b) administers the injection in the thigh
c) positions the client in a prone toe-in position
d) positions the client in a Sims' position

Answer: C
- The RN is responsible for supervising certain procedures performed by an LPN to ensure that client safety is maintained. The dorsogluteal site or gluteus medius muscle is the desired site for deep intramuscular injections. A prone toe-in position will promote internal rotation of the hips, which will relax the muscle and make the injection less painful. Option D is incorrect and will not relax the muscle. Option A describes the administration of an injection into the deltoid muscle. Option B describes an injection into the vastus lateralis or rectus femoris muscle.

==================================================

Nclex Maternity Practice Questions

Question: Prior to discharging a 24-hour-old newborn, the nurse assesses her respiratory status. Which of the following would the nurse expect to assess?

A) Respiratory rate 45, irregular
B) Costal breathing pattern
C) Nasal flaring, rate 65
D) Crackles on auscultation

Answer: A
Typically, respirations in a 24-hour-old newborn are symmetric, slightly irregular, shallow, and unlabored at a rate of 30 to 60 breaths/minute. The breathing pattern is primarily diaphragmatic. Nasal flaring, rates above 60 breaths per minute, and crackles suggest a problem.

==================================================

Question: The nurse encourages the mother of a healthy newborn to put the newborn to the breast immediately after birth for which reason?

A) To aid in maturing the newborn's sucking
reflex
B) To encourage the development of maternal antibodies
C) To facilitate maternal-infant bonding
D) To enhance the clearing of the newborn's respiratory passages

Answer: C
Breast-feeding can be initiated immediately after birth. This immediate mother-newborn contact takes advantage of the newborn's natural alertness and fosters bonding. This contact also reduces maternal bleeding and stabilizes the newborn's temperature, blood glucose level, and respiratory rate. It is not associated with maturing the sucking reflex, encouraging the development of maternal antibodies, or aiding in clearing of the newborn's respiratory passages.

==================================================

Question: When making a home visit, the nurse observes a newborn sleeping on his back in a bassinet. In one corner of the bassinet is a soft stuffed animal and at the other end is a bulb syringe. The nurse determines that the mother needs additional teaching because of which of the following?

A) The newborn should not be sleeping on his back.
B) Stuffed animals should not be in areas where infants sleep.
C) The bulb syringe should not be kept in the bassinet.
D) This newborn should be sleeping in a crib.

Answer: B
The nurse should instruct the mother to remove all fluffy bedding, quilts, stuffed animals, and pillows from the crib to prevent suffocation. Newborns and infants should be placed on their backs to sleep. Having the bulb syringe nearby in the bassinet is appropriate. Although a crib is the safest sleeping location, a bassinet is appropriate initially.

==================================================

Question: Assessment of a newborn reveals a heart rate of 180 beats/minute. To determine whether this finding is a common variation rather than a sign of distress, what else does the nurse need to know?

A) How many hours old is this newborn?
B) How long ago did this newborn eat?
C) What was the newborn's birthweight?
D) Is acrocyanosis present?

Answer: A
The typical heart rate of a newborn ranges from 120 to 160 beats per minute with wide fluctuation during activity and sleep. Typically heart rate is assessed every 30 minutes until stable for 2 hours after birth. The time of the newborn's last feeding and his birthweight would have no effect on his heart rate. Acrocyanosis is a common normal finding in newborns.

==================================================

Question: Just after delivery, a newborn's axillary temperature is 94 degrees F. What action would be most appropriate?

A) Assess the newborn's gestational age.
B) Rewarm the newborn gradually.
C) Observe the newborn every hour.
D) Notify the physician if the temperature goes lower.

Answer: B
A newborn's temperature is typically maintained at 36.5 to 37.5 degrees C (97.7 to 99.7 degrees F). Since this newborn's temperature is significantly lower, the nurse should institute measures to rewarm the newborn gradually. Assessment of gestational age is completed regardless of the newborn's temperature. Observation would be inappropriate because lack of action may lead to a further lowering of the temperature. The nurse should notify the physician of the newborn's current temperature since it is outside normal parameters.

==================================================

Question: The parents of a newborn become concerned when they notice that their baby seems to stop breathing for a few seconds. After confirming the parents' findings by observing the newborn, which of the following actions would be most appropriate?

A) Notify the health care provider immediately.
B) Assess the newborn for signs of respiratory distress.
C) Reassure the parents that this is an expected pattern.
D) Tell the parents not to worry since his color is fine.

Answer: B
Although periods of apnea of less than 20 seconds can occur, the nurse needs to gather additional information about the newborn's respiratory status to determine if this finding is indicative of a developing problem. Therefore, the nurse would need to assess for signs of respiratory distress. Once this information is obtained, then the nurse can notify the health care provider or explain that this finding is an expected one. However, it would be inappropriate to tell the parents not to worry, because additional information is needed. Also, telling them not to worry ignores their feelings and is not therapeutic.

==================================================

Question: When assessing a newborn 1 hour after birth, the nurse measures an axillary temperature of 95.8 degrees F, an apical pulse of 114 beats/minute, and a respiratory rate of 60 breaths/minute. Which nursing diagnosis takes highest priority?

A) Hypothermia related to heat loss during birthing process
B) Impaired parenting related to addition of new family member
C) Risk for deficient fluid volume related to insensible fluid loss
D) Risk for infection related to transition to extrauterine environment

Answer: A
The newborn's heart rate is slightly below the accepted range of 120 to 160 beats/minute; the respiratory rate is at the high end of the accepted range of 30 to 60 breaths per minute. However, the newborn's temperature is significantly below the accepted range of 97.7 to 99.7 degrees F. Therefore, the priority nursing diagnosis is hypothermia. There is no information to suggest impaired parenting. Additional information is needed to determine if there is a risk for deficient fluid volume or a risk for infection.

==================================================

Question: The nurse places a newborn with jaundice under the phototherapy lights in the nursery to achieve which goal?

A) Prevent cold stress
B) Increase surfactant levels in the lungs
C) Promote respiratory stability
D) Decrease the serum bilirubin level

Answer: D
Jaundice reflects elevated serum bilirubin levels; phototherapy helps to break down the bilirubin for excretion. Phototherapy has no effect on body temperature, surfactant levels, or respiratory stability.

==================================================

Question: The nurse assesses a 1-day-old newborn. Which finding indicates that the newborn's oxygen needs aren't being met?

A) Respiratory rate of 54 breaths/minute
B) Abdominal breathing
C) Nasal flaring
D) Acrocyanosis

Answer: C
Nasal flaring is a sign of respiratory difficulty in the newborn. A rate of 54 breaths/minute, diaphragmatic/abdominal breathing, and acrocyanosis are normal findings.

==================================================

Question: During a physical assessment of a newborn, the nurse observes bluish markings across the newborn's lower back. The nurse interprets this finding as:

A) Milia
B) Mongolian spots
C) Stork bites
D) Birth trauma

Answer: B
Mongolian spots are blue or purple splotches that appear on the lower back and buttocks of newborns. Milia are unopened sebaceous glands frequently found on a newborn's nose. Stork bites are superficial vascular areas found on the nape of the neck and eyelids and between the eyes and upper lip. Birth trauma would be manifested by bruising, swelling, and possible deformity.

==================================================

Question: While making rounds in the nursery, the nurse sees a 6-hour-old baby girl gagging and turning bluish. What would the nurse do first?

A) Alert the physician stat and turn the newborn to her right side.
B) Administer oxygen via facial mask by positive pressure.
C) Lower the newborn's head to stimulate crying.
D) Aspirate the oral and nasal pharynx with a bulb syringe.

Answer: D
The nurse's first action would be to suction the oral and nasal pharynx with a bulb syringe to maintain airway patency. Turning the newborn to her right side will not alleviate the blockage due to secretions. Administering oxygen via positive pressure is not indicated at this time. Lowering the newborn's head would be inappropriate.

==================================================

Question: While performing a physical assessment of a newborn boy, the nurse notes diffuse edema of the soft tissues of his scalp that crosses suture lines. The nurse documents this finding as:

A) Molding
B) Microcephaly
C) Caput succedaneum
D) Cephalhematoma

Answer: C
Caput succedaneum is localized edema on the scalp, a poorly demarcated soft tissue swelling that crosses the suture lines. Molding refers to the elongated shape of the fetal head as it accommodates to the passage through the birth canal. Microcephaly refers to a head circumference that is 2 standard deviations below average or less than 10% of normal parameters for gestational age. Cephalhematoma is a localized effusion of blood beneath the periosteum of the skull.

==================================================

Question: Assessment of a newborn reveals uneven gluteal (buttocks) skin creases and a "clunk" when Ortolani's maneuver is performed. Which of the following would the nurse suspect?

A) Slipping of the periosteal joint
B) Developmental hip dysplasia
C) Normal newborn variation
D) Overriding of the pelvic bone

Answer: B
A "clunk" indicates the femoral head hitting the acetabulum as the head reenters the area. This, along with uneven gluteal creases, suggests developmental hip dysplasia. These findings are not a normal variation and are not associated with slipping of the periosteal joint or overriding of the pelvic bone.

==================================================

Question: The nurse strokes the lateral sole of the newborn's foot from the heel to the ball of the foot when evaluating which reflex?

A) Babinski
B) Tonic neck
C) Stepping
D) Plantar grasp

Answer: A
The Babinski reflex is elicited by stroking the lateral sole of the newborn's foot from the heel toward and across the ball of the foot. The tonic neck reflex is tested by having the newborn lie on his back and then turn his head to one side. The stepping reflex is elicited by holding the newborn upright and inclined forward with the soles of the feet on a flat surface. The plantar grasp reflex is elicited by placing a finger against the area just below the newborn's toes.

==================================================

Question: The nurse administers vitamin K intramuscularly to the newborn based on which of the following rationales?

A) Stop Rh sensitization
B) Increase erythopoiesis
C) Enhance bilirubin breakdown
D) Promote blood clotting

Answer: D
Vitamin K promotes blood clotting by increasing the synthesis of prothrombin by the liver. RhoGAM prevents Rh sensitization. Erythropoietin stimulates erythropoiesis. Phototherapy enhances bilirubin breakdown.

==================================================

Question: When explaining how a newborn adapts to extrauterine life, the nurse would describe which body systems as undergoing the most rapid changes?

A) Gastrointestinal and hepatic
B) Urinary and hematologic
C) Respiratory and cardiovascular
D) Neurological and integumentary

Answer: C
Although all the body systems of the newborn undergo changes, respiratory gas exchange along with circulatory modifications must occur immediately to sustain extrauterine life.

==================================================

Question: A new mother reports that her newborn often spits up after feeding. Assessment reveals regurgitation. The nurse responds based on the understanding that this most likely is due to which of the following?

A) Placing the newborn prone after feeding
B) Limited ability of digestive enzymes
C) Underdeveloped pyloric sphincter
D) Relaxed cardiac sphincter

Answer: D
The cardiac sphincter and nervous control of the stomach is immature, which may lead to uncoordinated peristaltic activity and frequent regurgitation. Placement of the newborn is unrelated to regurgitation. Most digestive enzymes are available at birth, but they are limited in their ability to digest complex carbohydrates and fats; this results in fatty stools, not regurgitation. Immaturity of the pharyngoesophageal sphincter and absence of lower esophageal peristaltic waves, not an underdeveloped pyloric sphincter, also contribute to the reflux of gastric contents.

==================================================

Question: After teaching a class about hepatic system adaptations after birth, the instructor determines that the teaching was successful when the class identifies which of the following as the process of changing bilirubin from a fat-soluble product to a water-soluble product?

A) Hemolysis
B) Conjugation
C) Jaundice
D) Hyperbilirubinemia

Answer: B
The process in which bilirubin is changed from a fat-soluble product to a water-soluble product is called conjugation. Hemolysis involves the breakdown of blood cells. In the newborn, hemolysis of the red blood cells is the principal source of bilirubin. Jaundice is the manifestation of increased bilirubin in the bloodstream. Hyperbilirubinemia refers to the increased level of bilirubin in the blood.

==================================================

Question: Twenty minutes after birth, a baby begins to move his head from side to side, making eye contact with the mother, and pushes his tongue out several times. The nurse interprets this as:

A) A good time to initiate breast-feeding
B) The period of decreased responsiveness preceding sleep
C) The need to be alert for gagging and vomiting
D) Evidence that the newborn is becoming chilled

Answer: A
The newborn is demonstrating behaviors indicating the first period of reactivity, which usually begins at birth and lasts for the first 30 minutes. This is a good time to initiate breast-feeding. Decreased responsiveness occurs from 30 to 120 minutes of age and is characterized by muscle relaxation and diminished responsiveness to outside stimuli. There is no indication that the newborn may experience gagging or vomiting. Chilling would be evidenced by tachypnea, decreased activity, and hypotonia.

==================================================

Question: The nurse institutes measure to maintain thermoregulation based on the understanding that newborns have limited ability to regulate body temperature because they:

A) Have a smaller body surface compared to body mass
B) Lose more body heat when they sweat than adults
C) Have an abundant amount of subcutaneous fat all over
D) Are unable to shiver effectively to increase heat production

Answer: D
Newborns have difficulty maintaining their body heat through shivering and other mechanisms. They have a large body surface area relative to body weight and have limited sweating ability. Additionally, newborns lack subcutaneous fat to provide insulation.

==================================================

Question: A new mother is changing the diaper of her 20-hour-old newborn and asks why the stool is almost black. Which response by the nurse would be most appropriate?

A) "You probably took iron during your pregnancy."
B) "This is meconium stool, normal for a newborn."
C) "I'll take a sample and check it for possible bleeding."
D) "This is unusual and I need to report this."

Answer: B
Meconium is greenish-black and tarry and usually passed within 12 to 24 hours of birth. This is a normal finding. Iron can cause stool to turn black, but this would not be the case here. The stool is a normal occurrence and does not need to be checked for blood or reported.

==================================================

Question: A client expresses concern that her 2-hour-old newborn is sleepy and difficult to awaken. The nurse explains that this behavior indicates which of the following?

A) Normal progression of behavior
B) Probable hypoglycemia
C) Physiological abnormality
D) Inadequate oxygenation

Answer: A
From 30 to 120 minutes of age, the newborn enters the second stage of transition, that of sleep or a decrease in activity. More information would be needed to determine if hypoglycemia, a physiologic abnormality, or inadequate oxygenation was present.

==================================================

Question: After the birth of a newborn, which of the following would the nurse do first to assist in thermoregulation?

A) Dry the newborn thoroughly.
B) Put a hat on the newborn's head.
C) Check the newborn's temperature.
D) Wrap the newborn in a blanket.

Answer: A
Drying the newborn immediately after birth using warmed blankets is essential to prevent heat loss through evaporation. Then the nurse would place a cap on the baby's head and wrap the newborn. Assessing the newborn's temperature would occur once these measures were initiated to prevent heat loss.

==================================================

Question: Assessment of a newborn reveals rhythmic spontaneous movements. The nurse interprets this as indicating:

A) Habituation
B) Motor maturity
C) Orientation
D) Social behaviors

Answer: B
Motor maturity is evidenced by rhythmic, spontaneous movements. Habituation is manifested by the newborn's ability to respond to the environment appropriately. Orientation involves the newborn's response to new stimuli, such as turning the head to a sound. Social behaviors involve cuddling and snuggling into the arms of a parent.

==================================================

Question: When teaching new parents about the sensory capabilities of their newborn, which sense would the nurse identify as being the least mature?

A) Hearing
B) Touch
C) Taste
D) Vision

Answer: D
Vision is the least mature sense at birth. Hearing is well developed at birth, evidenced by the newborn's response to noise by turning. Touch is evidenced by the newborn's ability to respond to tactile stimuli and pain. A newborn can distinguish between sweet and sour by 72 hours of age.

==================================================

Question: The nurse places a warmed blanket on the scale when weighing a newborn to minimize heat loss via which mechanism?

A) Evaporation
B) Conduction
C) Convection
D) Radiation

Answer: B
Using a warmed cloth diaper or blanket to cover any cold surface, such as a scale, that touches a newborn directly helps to prevent heat loss through conduction. Drying a newborn and promptly changing wet linens, clothes, or diapers help reduce heat loss via evaporation. Keeping the newborn out of a direct cool draft, working inside an isolette as much as possible, and minimizing the opening of portholes help prevent heat loss via convection. Keeping cribs and isolettes away from outside walls, cold windows, and air conditioners and using radiant warmers while transporting newborns and performing procedures will help reduce heat loss via radiation.

==================================================

Question: Which of the following would alert the nurse to the possibility of respiratory distress in a newborn?

A) Symmetrical chest movements
B) Periodic breathing
C) Respirations of 40 breaths/minute
D) Sternal retractions

Answer: D
Sternal retractions, cyanosis, tachypnea, expiratory grunting, and nasal flaring are signs of respiratory distress in a newborn. Symmetrical chest movements and a respiratory rate between 30 to 60 breaths/minute are typical newborn findings. Some newborns may demonstrate periodic breathing (cessation of breathing lasting 5 to 10 seconds without changes in color or heart rate) in the first few days of life.

==================================================

Question: When counseling a mother about the immunologic properties of breast milk, the nurse would emphasize breast milk as a major source of which immunoglobulin?

A) IgA
B) IgG
C) IgM
D) IgE

Answer: A
A major source of IgA is human breast milk. IgG, found in serum and interstitial fluid, crosses the placenta beginning at approximately 20 to 22 weeks' gestation. IgM is found in blood and lymph fluid and levels are generally low at birth unless there is a congenital intrauterine infection. IgE is not found in breast milk and does not play a major role in defense in the newborn.

==================================================

Question: The nurse is teaching a group of students about the similarities and differences between newborn skin and adult skin. Which statement by the group indicates that additional teaching is needed?

A) The newborn's skin and that of an adult are similar in thickness.
B) The newborn's sweat glands function fully, just like those of an adult.
C) Skin development in the newborn is not complete at birth.
D) The newborn has fewer fibrils connecting the dermis and epidermis

Answer: B
The newborn has sweat glands, like an adult, but full adult functioning is not present until the second or third year of life. The newborn and adult epidermis is similar in thickness and lipid composition, but skin development is not complete at birth. Fewer fibrils connect the dermis and epidermis in the newborn when compared with the adult.

==================================================

Question: When describing the neurologic development of a newborn to his parents, the nurse would explain that it occurs in which fashion?

A) Head-to-toe
B) Lateral-to-medial
C) Outward-to-inward
D) Distal-caudal

Answer: A
Neurologic development follows a cephalocaudal (head-to-toe) and proximal-distal (center to outside) pattern.

==================================================

Question: A client at 28 weeks gestation is admitted to the labor and birth unit. Which test would most likely be used to assess the client's comprehensive fetal status?

a) Ultrasound for physical structure
b) Nonstress test (NST)
c) Biophysical profile (BPP)
d) Amniocentesis

Answer: C.

Biophysical profile is a comprehensive test that would be used to assess the client's fetal status at 28 weeks gestation. Ultrasound for physical structure is limited to identifying the growth and development of the fetus, and does not assess for other parameters of fetal well-being. Women with a high-risk factor will probably begin having NSTs at 30-32 weeks gestation and at frequent intervals for the remainder of the pregnancy. Amniocentesis late in pregnancy is used to test for lung maturity, not overall fetal status in labor, and when performed earlier it is used to test for specific disorders.

==================================================

Question: A prenatal client in her second trimester is admitted to the maternity unit with painless, bright red vaginal bleeding. What test might the physician order?

a)Alpha-fetoprotein (AFP)
b)Contraction stress test (CST)
c)Amniocentesis
d)Ultrasound

Answer: D.

An ultrasound for placenta location to rule out placenta previa would be ordered for a client who presents with painless, bright red vaginal bleeding. The ability to see the lower portion of the uterus and cervix with ultrasound is particularly important when vaginal bleeding is noted and placenta previa is the suspected cause. Alpha-fetoprotein (AFP) is a test used to screen for neural tube defects. A contraction stress test is ordered in the third trimester to evaluate the respiratory function of the placenta. Amniocentesis is a procedure used for genetic diagnosis or, in later pregnancy, for lung maturity studies.

==================================================

Question: The nurse is preparing a prenatal client for a transvaginal ultrasound. What nursing action should the nurse include in the preparations? Select all that apply.

a)Advise the client to empty her bladder.
b)Encourage the client to drink 1.5 quarts of fluid.
c)Apply transmission gel over the client's abdomen.
d)Place client in lithotomy position.

Answer: D

After having the client void, assist her to a lithotomy position for a transvaginal ultrasound. Preparation for a transabdominal ultrasound includes encouraging the client to drink 1.5 quarts of fluid, maintaining a full bladder, and applying transmission gel over the client's abdomen.

==================================================

Question: A pregnant client asks why ultrasound is used so frequently during pregnancy. The nurse's response is based on her knowledge that the advantages of ultrasound include which of the following? Select all that apply.

a)"It is noninvasive and painless."
b)"It can be used to estimate gestational age."
c)"Results are immediate."
d)"The ultrasound is the only test to determine gender."

Answer: A, B, C

The ability to establish fetal age accurately by ultrasound is lost in the third trimester because fetal growth is not as uniform as it is in the first two trimesters; however, ultrasound can be used to approximate gestational age within 1-3 weeks' accuracy during the third trimester. A comprehensive ultrasound is used to detect anatomical defects, not gestational age. Ultrasound is not used to determine gender.

==================================================

Question: The physician orders an ultrasound for a prenatal client prior to an amniocentesis. The nurse explains to the client that the purpose of the ultrasound is to:

a)Determine the gestational sac volume.
b)Measure the fetus's crown-rump length.
c)Locate the placenta.
d)Measure the fetus's biparietal diameter.

Answer: C

During an amniocentesis, the physician scans the uterus using ultrasound to identify the fetal and placental positions and to identify adequate pockets of amniotic fluid. Determination of the gestational sac volume, measuring the crown-rump length, and measuring the biparietal diameter are aspects of assessing fetal well-being (biophysical profile, or BPP), and may or may not be done prior to the amniocentesis, depending on gestational age.

==================================================

Question: The nurse is reviewing four prenatal charts. Which client would be an appropriate candidate for a contraction stress test (CST)?

a)A client with intrauterine growth retardation
b)A client with multiple gestation
c)A client with an incompetent cervix
d)A client with placenta previa

Answer: A

A contraction stress test (CST) is indicated for a client with intrauterine growth retardation (IUGR), because it will assess the respiratory function of the placenta, which may be adversely affected by the conditions causing IUGR. The CST is contraindicated in third-trimester bleeding from placenta previa or marginal abruptio placentae, previous cesarean with classical incision (vertical incision in the fundus of the uterus), premature rupture of the membranes, incompetent cervix, anomalies of the maternal reproductive organs, history of preterm labor (if being done prior to term), or multiple gestation.

==================================================

Question: A prenatal client at 22 weeks gestation is scheduled for an amniocentesis. Which nursing action would apply to any client undergoing this procedure? Select all that apply.

a)Assess for bleeding.
b)Administer Rh immune globulin to the client.
c)Cleanse skin with alcohol.
d)Assess vital signs and fetal heart rate.

Answer: A, D

The skin is cleaned with a betadine solution. The use of a local anesthesia at the needle insertion site is optional. A 22-gauge needle is then inserted into the uterine cavity and amniotic fluid is withdrawn. After 15-20 mL of fluid has been removed, the needle is withdrawn and the site is assessed for streaming (movement of fluid), which is an indication of bleeding. The fetal heart rate and maternal vital signs are then assessed. Rh immune globulin is given only to all Rh-negative women.

==================================================

Question: A prenatal client at 30 weeks gestation is scheduled for a nonstress test (NST) and asks the nurse, "What is this test for?" The nurse correctly responds that the test is used to determine which of the following? Select all that apply.

a)Fetal lung maturity
b)Adequate fetal oxygenation
c)Accelerations of fetal heart rate
d)Fetal well-being

Answer: B, C, D

An NST documents fetal well-being by measuring fetal oxygenation and fetal heart rate accelerations, but not fetal lung maturity.

==================================================

Question: The nurse is teaching a prenatal client about chorionic villus sampling (CVS). The nurse correctly teaches the client that risks related to CVS include which of the following? Select all that apply.

a)Intrauterine infection
b)Rupture of membranes
c)Maternal hypertension
d)Spontaneous abortion

Answer: A, B, D

Risks of CVS include failure to obtain tissue, rupture of membranes, leakage of amniotic fluid, bleeding, intrauterine infection, maternal tissue contamination of the specimen, and Rh alloimmunization. CVS testing has a higher rate of spontaneous abortion than amniocentesis. Other complications include fetal limb defects and abnormalities of the fetal face and jaw.

==================================================

Question: A prenatal client at 35 weeks gestation is scheduled for an amniocentesis to determine fetal lung maturity. The nurse expects the lecithin/sphingomyelin (L/S) ratio to be:

a)0.5:1
b)1:1
c)2:1
d)3:1

Answer: C

Early in pregnancy, the sphingomyelin concentration in amniotic fluid is greater than the concentration of lecithin, and so the L/S ratio is low (lecithin levels are low and sphingomyelin levels are high). At about 32 weeks gestation, sphingomyelin levels begin to fall and the amount of lecithin begins to increase. By 35 weeks gestation, an L/S ratio of 2:1 (also reported as 2.0) is usually achieved in the normal fetus.

==================================================

Question: A pregnant client is concerned about a blow to the abdomen if she continues to play basketball during her pregnancy. The nurse's response is based upon her knowledge of which of the following facts concerning amniotic fluid?

a) The total amount of amniotic fluid during pregnancy is 300 mL.
b) Amniotic fluid functions as a cushion to protect against mechanical injury.
c) The fetus does not contribute to the production of amniotic fluid.
d) Amniotic fluid is slightly acidic.

Answer: B

During pregnancy, the amniotic fluid protects against injury. After 20 weeks of pregnancy, fluid volume ranges from 700-1000 mL. Some of the amniotic fluid is contributed by the fetus's excreting urine. Amniotic fluid is slightly alkaline.

==================================================

Question: A client states that she had a spontaneous abortion 12 months ago. The client asks if her hormones may have contributed to the loss of the pregnancy. The nurse's response is based upon her knowledge of which of the following facts?

a) Implantation occurs when progesterone levels are low.
b) hCG reaches a maximum level at 4 weeks gestation.
c) Progesterone decreases the contractility of the uterus.
d) Progesterone is only produced by the corpus luteum during pregnancy.

Answer: C

Progesterone decreases the contractility of the uterus, thus preventing uterine contractions that might cause spontaneous abortion. Progesterone must be present in high levels for implantation to occur. After 10 weeks, the placenta takes over the production of progesterone. hCG reaches its maximum level at 50-70 days gestation.

==================================================

Question: A nurse is teaching a group of student nurses about amniotic fluid. Which of the following statements by the student nurse reflects an understanding of the fetus's contribution to the quality of amniotic fluid? Select all that apply.

a) "The fetus contributes to the volume of amniotic fluid by excreting urine."
b) "Approximately 400 mL of amniotic fluid flows out of the fetal lungs each day."
c) "The fetus swallows about 600 mL of the fluid in 24 hours."
d) "A fetus can move freely and develop normally, even if there is no amniotic fluid."

Answer: A, B, C

"The fetus contributes to the volume of amniotic fluid by excreting urine." Approximately 400 mL of amniotic fluid flows out of the fetal lungs each day. The fetus swallows about 600 mL of the fluid in 24 hours. A normal volume of amniotic fluid is necessary for good fetal movement. Normal movement is necessary for good musculoskeletal development.

==================================================

Question: The nurse is preparing an educational workshop on fetal development. Which statement by the student would require the nurse to explain further?

a) "True knots are usually associated with a cord that is too long."
b) "The average cord length at term is 22 inches."
c) "The umbilical cord normally contains two veins and one artery."
d) "The high blood volume and Wharton's jelly content of the umbilical cord prevents compression of the cord."

Answer: C

Umbilical cords appear twisted or spiraled. This is most likely caused by fetal movement. A true knot in the umbilical cord rarely occurs; if it does, the cord is usually long. More common are so-called false knots, caused by the folding of cord vessels. A nuchal cord is said to exist when the umbilical cord encircles the fetal neck. A normal umbilical cord contains one large vein and two smaller arteries.
A specialized connective tissue known as Wharton's jelly surrounds the blood vessels in the umbilical cord. This tissue, plus the high blood volume pulsating through the vessels, prevents compression of the umbilical cord in utero.

==================================================

Question: At 17 weeks pregnant, a mother asks the nurse questions about the development of her baby. The mother states that it may be too early to visualize any body structures via ultrasound. The nurse's best response in relation to fetal development at 17 weeks is:

a) Myelination of the spinal cord has occurred.
b) Differentiation of hard and soft palate can be seen.
c) The earlobes are soft with little cartilage.
d) Hard tissue (enamel) for teeth has developed.

Answer: B

Differentiation of hard and soft palate are structures developed by 16 weeks gestation. Myelination of the spinal cord begins at 20 weeks gestation. Soft earlobes with little cartilage develop at 36 weeks gestation. Teeth form hard tissue (enamel) at 18 weeks gestation.

==================================================

Question: A nurse is teaching a group of first-trimester prenatal clients about the discomforts of pregnancy. A client asks the nurse, "What causes my nausea and vomiting?" The nurse responds indicating which of the following as being contributing factors to first-trimester emesis? Select all that apply.

a) Human chorionic gonadotropin
b) Estrogen
c) Alterations in carbohydrate metabolism
d) Prostaglandins

Answer: A, C

Nausea and vomiting are common during the first trimester because of elevated human chorionic gonadotropin levels and changed carbohydrate metabolism. Estrogen stimulates the growth of the uterus and breast tissue. Prostaglandins stimulate uterine contractions.

==================================================

Question: The nurse is taking an initial history of a prenatal client. Which of the following, if detected by the nurse practitioner, would indicate a positive, or diagnostic sign of pregnancy?

a) Positive pregnancy test
b) Goodell's sign
c) Uterine enlargement and amenorrhea
d) Fetal heartbeat with at Doppler at 11 weeks gestation

Answer: D

The positive signs of pregnancy are completely objective, cannot be confused with a pathologic state, and offer conclusive proof of pregnancy.
The fetal heartbeat can be detected with an electronic Doppler device as early as weeks 10-12 of pregnancy.
Pregnancy tests detect the presence of hCG in the maternal blood or urine. These are not considered a positive sign of pregnancy because other conditions can cause elevated hCG levels. Physical changes, like Godell's sign and uterine enlargement, can also have other causes and do not confirm pregnancy. The subjective changes of pregnancy, like amenorrhea, are the symptoms the woman experiences and reports. Because they can be caused by other conditions, they cannot be considered proof of pregnancy.

==================================================

Question: The nurse in the prenatal clinic assesses a 26-year-old client at 13 weeks gestation. Which presumptive (subjective) signs and symptoms of pregnancy should the nurse anticipate?

a) Hegar's sign and quickening
b) Ballottement and positive pregnancy test
c) Chadwick's sign and uterine souffle
d) Excessive fatigue and urinary frequency

Answer: D

Excessive fatigue and urinary frequency both are presumptive (subjective) signs and symptoms of pregnancy. Hegar's sign, ballottement, a positive pregnancy test, Chadwick's sign, and uterine souffle are probable (objective) signs or symptoms of pregnancy.

==================================================

Question: The nurse is teaching a group of students about the differences between a full-term newborn and a preterm newborn. The nurse determines that the teaching is effective when the students state that the preterm newborn has:

A) Fewer visible blood vessels through the skin
B) More subcutaneous fat in the neck and abdomen
C) Well-developed flexor muscles in the extremities
D) Greater surface area in proportion to weight

Answer: D
Preterm newborns have large body surface areas compared to weight, which allows an increased transfer of heat from their bodies to the environment. Preterm newborns often have thin transparent skin with numerous visible veins, minimal subcutaneous fat, and poor muscle tone.

==================================================

Question: When assessing a postterm newborn, which of the following would the nurse correlate with this gestational age variation?

A) Moist, supple, plum skin appearance
B) Abundant lanugo and vernix
C) Thin umbilical cord
D) Absence of sole creases

Answer: C
A postterm newborn typically exhibits a thin umbilical cord; dry, cracked, wrinkled skin; limited vernix and lanugo; and creases covering the entire soles of the feet.

==================================================

Question: The parents of a preterm newborn being cared for in the neonatal intensive care unit (NICU) are coming to visit for the first time. The newborn is receiving mechanical ventilation and intravenous fluids and medications and is being monitored electronically by various devices. Which action by the nurse would be most appropriate?

A) Suggest that the parents stay for just a few minutes to reduce their anxiety.
B) Reassure them that their newborn is progressing well.
C) Encourage the parents to touch their preterm newborn.
D) Discuss the care they will be giving the newborn upon discharge.

Answer: C
The NICU environment can be overwhelming. Therefore, the nurse should address their reactions and explain all the equipment being used. On entering the NICU, the nurse should encourage the parents to touch, interact, and hold their newborn. Doing so helps to acquaint the parents with their newborn, promotes self-confidence, and fosters parent-newborn attachment. The parents should be allowed to stay for as long as they feel comfortable. Reassurance, although helpful, may be false reassurance at this time. Discussing discharge care can be done later once the newborn's status improves and plans for discharge are initiated.

==================================================

Question: When performing newborn resuscitation, which action would the nurse do first?

A) Intubate with an appropriate-sized endotracheal tube.
B) Give chest compressions at a rate of 80 times per minute.
C) Administer epinephrine intravenously.
D) Suction the mouth and then the nose.

Answer: D
After placing the newborn's head in a neutral position, the nurse would suction the mouth and then the nose. This is followed by ventilation, circulation (chest compressions), and administration of epinephrine.

==================================================

Question: The nurse frequently assesses the respiratory status of a preterm newborn based on the understanding that the newborn is at increased risk for respiratory distress syndrome because of which of the following?

A) Inability to clear fluids
B) Immature respiratory control center
C) Deficiency of surfactant
D) Smaller respiratory passages

Answer: C
A preterm newborn is at increased risk for respiratory distress syndrome (RDS) because of a surfactant deficiency. Surfactant helps to keep the alveoli open and maintain lung expansion. With a deficiency, the alveoli collapse, predisposing the newborn to RDS. An inability to clear fluids can lead to transient tachypnea. Immature respiratory control centers lead to an increased risk for apnea. Smaller respiratory passages led to an increased risk for obstruction.

==================================================

Question: The nurse prepares to assess a newborn who is considered to be large for gestational age (LGA). Which of the following would the nurse correlate with this gestational age variation?

A) Strong, brisk motor skills
B) Difficulty in arousing to a quiet alert state
C) Birthweight of 7 lb 14 oz
D) Wasted appearance of extremities

Answer: B
LGA newborns typically are more difficult to arouse to a quiet alert state. They have poor motor skills, have a large body that appears plump and full-sized, and usually weigh more than 8 lb 13 oz at term.

==================================================

Question: An LGA newborn has a blood glucose level of 23 mg/dL. Which of the following would the nurse do next?

A) Administer intravenous glucose immediately.
B) Feed the newborn 2 ounces of formula.
C) Initiate blow-by oxygen therapy.
D) Place the newborn under a radiant warmer.

Answer: A
If an LGA newborn's blood glucose level is below 25 mg/dL, the nurse should institute immediate treatment with intravenous glucose regardless of the clinical symptoms. Oral feedings would be used to maintain the newborn's glucose level above 40 mg/dL. Blow-by oxygen would have no effect on glucose levels; it may be helpful in promoting oxygenation. Placing the newborn under a radiant warmer would be a more appropriate measure for cold stress.

==================================================

Question: When describing the complications that can occur in newborns to a group of pregnant women, which would the nurse include as being at lowest risk?

A) Small-for-gestational-age newborns
B) Large-for-gestational-age newborns
C) Appropriate-for-gestational-age newborns
D) Low-birthweight newborns

Answer: C
Appropriate-for-gestational-age newborns are at the lowest risk for any problems. The other categories all have an increased risk of complications.

==================================================

Question: While caring for a preterm newborn receiving oxygen therapy, the nurse monitors the oxygen level and duration closely based on the understanding that the newborn is at risk for which of the following?

A) Retinopathy of prematurity
B) Metabolic acidosis
C) Infection
D) Cold stress

Answer: A
Oxygen therapy has been implicated in the pathogenesis of retinopathy of prematurity (ROP). Therefore, the nurse monitors the newborn's oxygen therapy closely. Metabolic acidosis may occur due to anaerobic metabolism used for heat production. Infection may occur for numerous reasons, but they are unrelated to oxygen therapy. Cold stress results from problems due to the preterm newborn's inadequate supply of brown fat, decreased muscle tone, and large body surface area.

==================================================

Question: When planning the care for a SGA newborn, which action would the nurse determine as a priority?

A) Preventing hypoglycemia with early feedings
B) Observing for respiratory distress syndrome
C) Promoting bonding between the parents and the newborn
D) Monitoring vital signs every 2 hours

Answer: A
With the loss of the placenta at birth, the newborn must now assume control of glucose homeostasis. This is achieved by early oral intermittent feedings. Observing for respiratory distress, promoting bonding, and monitoring vital signs, although important, are not the priority for this newborn.

==================================================

Question: A woman gives birth to a newborn at 36 weeks' gestation. She tells the nurse, "I'm so glad that my baby isn't premature." Which response by the nurse would be most appropriate?

A) "You are lucky to have given birth to a term newborn."
B) "We still need to monitor him closely for problems."
C) "How do you feel about delivering your baby at 36 weeks?"
D) "Your baby is premature and needs monitoring in the NICU."

Answer: B
A baby born at 36 weeks' gestation is considered a late-preterm newborn. These newborns face similar challenges as those of preterm newborns and require similar care. Telling the mother that close monitoring is necessary can prevent any misconceptions that she might have and prepare her for what might arise. The baby is not considered a term newborn, nor is the baby considered premature. The decision for care in the NICU would depend on the newborn's status. Asking the woman how she feels about the delivery demonstrates caring but does not address the woman's lack of understanding about her newborn.

==================================================

Question: Which of the following would be most appropriate for the nurse to do when assisting parents who have experienced the loss of their preterm newborn?

A) Avoid using the terms "death" or "dying."
B) Provide opportunities for them to hold the newborn.
C) Refrain from initiating conversations with the parents.
D) Quickly refocus the parents to a more pleasant topic.

Answer: B
When dealing with grieving parents, nurses should provide them with opportunities to hold the newborn if they desire. In addition, the nurse should provide the parents with as many memories as possible, encouraging them to see, touch, dress, and take pictures of the newborn. These interventions help to validate the parents' sense of loss, relive the experience, and attach significance to the meaning of loss. The nurse should use appropriate terminology, such as "dying," "died," and "death," to help the parents accept the reality of the death. Nurses need to demonstrate empathy and to respect the parents' feelings, responding to them in helpful and supportive ways. Active listening and allowing the parents to vent their frustrations and anger help validate the parents' feelings and facilitate the grieving process.

==================================================

Question: Which of the following, if noted in the maternal history, would the nurse identify as possibly contributing to the birth of a LGA newborn?

A) Drug abuse
B) Diabetes
C) Preeclampsia
D) Infection

Answer: B
Maternal factors that increase the chance of having an LGA newborn include maternal diabetes mellitus or glucose intolerance, multiparity, prior history of a macrosomic infant, postdates gestation, maternal obesity, male fetus, and genetics. Drug abuse is associated with SGA newborns and preterm newborns. A maternal history of preeclampsia and infection would be associated with preterm birth.

==================================================

Question: Which of the following would alert the nurse to suspect that a preterm newborn is in pain?

A) Bradycardia
B) Oxygen saturation level of 94%
C) Decreased muscle tone
D) Sudden high-pitched cry

Answer: D
The nurse should suspect pain if the newborn exhibits a sudden high-pitched cry, oxygen desaturation, tachycardia, and increased muscle tone.

==================================================

Question: What factors influence the outcomes of the at-risk newborn? Select all that apply.

a)Birth weight
b)Gestational age
c)Type and length of newborn illness
d)Environmental factors
e)Maternal factors

Answer: All are correct. Maternal factors such as age and parity, newborn weight, and gestational age also influence outcomes, as do environmental factors such as exposure to environmental dangers (toxic chemicals and illicit drugs).
Evaluation; Physiological Integrity; Analysis

==================================================

Question: Identify a complication of the small-for-gestational-age newborn.

a)Hyperglycemia
b)Cognitive difficulties
c)Leukocytosis
d)Hyperthermia

Answer: B

SGA newborns often subsequently exhibit learning disabilities. The disabilities are characterized by hyperactivity, short attention span, and poor fine motor coordination. Some hearing loss and speech defects also occur. The SGA newborn does not exhibit symptoms of high blood sugars, increased temperatures, and high white blood cell counts.

==================================================

Question: Understanding the transition from intrauterine to extrauterine life, what intervention is most appropriate when working with an infant of a diabetic mother?

a)Frequent blood glucose checks
b)Obtain lab work to look for infection.
c)Administer IV fluids.
d)Place under radiant warmer bed immediately.

Answer: A

Lab work, IV fluids, and the radiant warmer bed may all be required for interventions for the infant of a diabetic mother, if the infant is experiencing signs of respiratory distress or sepsis. Frequent blood glucose checks need to be completed to ensure that blood glucose levels are being maintained.

==================================================

Question: A 28-week-gestation newborn experienced birth asphyxia at the time of delivery. What is a long-term complication of birth asphyxia?

a)Necrotizing enterocolitis
b)Retinopathy of prematurity
c)Intraventricular hemorrhage
d)Anemia of prematurity

Answer: C

Birth asphyxia will cause an insult to the brain, and more often than not will cause a bleed or intraventricular hemorrhage. Birth asphyxia is not directly correlated with NEC, retinopathy of prematurity, or anemia of prematurity. These are common for the premature infant, but not necessarily birth asphyxia.

==================================================

Question: What is the best intervention a nurse can utilize to promote parent-infant attachment?

a)Allow for privacy.
b)Contact support families that have been through the same diagnosis with their own child and allow time to discuss the situation.
c)Provide an extensive handbook with information related to the preterm newborn.
d)Encourage rooming in.

Answer: D

All will help strengthen the attachment bond, but the best answer would be to encourage rooming in. Rooming in can provide a great opportunity for the stable preterm infant and family to get acquainted; it offers both privacy and readily available help.

==================================================

Question: The preterm newborn is experiencing vomiting, diarrhea, weight loss, irritability, tremors, and tachypnea. What is the best explanation for these symptoms?

a)Traumatic birth
b)Maternal substance abuse
c)Sepsis
d)Gestational diabetes

Answer: B

The severity of withdrawal that an infant experiences can be assessed by using a scoring system such as the Finnegan scale. This scale is based on observations and measurement of the responses to neonatal abstinence from substances. It evaluates the infant on potentially life-threatening signs such as vomiting, diarrhea, weight loss, irritability, tremors, and tachypnea.

==================================================

Nclex Medical Surgical Questions

Question: Upon entering the room of a patient who has just returned from surgery for total laryngectomy and radical neck dissection, a nurse should recognize a need for intervention when finding

A. a gastrostomy tube that is clamped.
B. the patient coughing blood-tinged secretions from the tracheostomy.
C. the patient positioned in a lateral position with the head of the bed flat.
D. 200 ml of serosanguineous drainage in the patient's portable drainage device.

Answer: C. the patient positioned in a lateral position with the head of the bed flat. After total laryngectomy and radical neck dissection, a patient should be placed in a semi-Fowler's position to decrease edema and limit tension on the suture line.

==================================================

Question: A 78-year-old does not want to eat lunch and complains that the food that is serve does not taste good. Consistent with knowledge about age-related changes to taste, the nurse may find that the client is more willing to eat.

A) Greasy foods
B) Sour foods
C) Sweet foods
D) Salty foods.

Answer: C = the older adults' taste buds retain their sensitivity to carbohydrates. In addition, carbohydrates. Tend to be food items that are easy to chew. Older adults lose their sensitivity to sour and salty foods. Older adults may find greasy foods harder to digest and therefore may avoid them; however, preference for greasy foods is not related to changes in taste associated with age.

==================================================

Question: The nurse is preparing a discharge plan to a female client with peptic ulcer for the dietary modification she will need to follow at home. Which of the following statements indicates that the client understands the instruction of the nurse?

A) "I should not drink alcohol and caffeine."
B) "I should eat a bland, soft diet."
C) "It is important to eat six small meals a day."
D) "I should drink several glasses of milk a day."

Answer: A = caffeinated beverages and alcohol should be avoided because they stimulate gastric acid production and irritate gastric mucosa.

The client should avoid foods that cause discomfort; however, there is no need to follow a soft, bland diet.

Eating six small meals daily is no longer a common treatment for peptic ulcer disease.

Milk in large quantities is not recommended because it actually stimulates further production of gastric acids.

==================================================

Question: A community health nurse is teaching smoking cessation program to a group of healthy adult smokers. What type of prevention activity is this?

A) Primary
B) Secondary
C) Tertiary
D) None of the above

Answer: A = primary cancer prevention targets healthy individuals and includes steps to avoid factors that might lead to the development of diseases.

==================================================

Question: A female client with breast cancer is currently receiving radiation therapy for treatment. The client is complaining of apathy, hard to concentrate on something, and feeling tired despite of having time to rest and more sleep. These complains suggest symptoms of:

A) Hypocalcemia
B) radiation pneumonitis
C) advanced breast cancer
D) fatigue

Answer: D = Fatigue is a common complaint of individuals receiving medication therapy.

==================================================

Question: The nurse is removing the client's staples from an abdominal when the client cough continuously and the incision splits open exposing the intestines. Which of the following is the immediate nursing action of the nurse?

A) Call the surgeon to come to the client's room immediately
B) Have all visitors and family member leave the room
C) Press the emergency alarm to call the resuscitation team
D) Cover the abdominal organs with sterile dressing moistened with sterile normal saline.

Answer: D = When a wound eviscerates, the nurse should cover the open area with sterile dressing moistened with sterile normal saline and then cover it with a dry dressing.

The surgeon should then be notified to take the client back to the operating room to close the incision under general anesthesia.

==================================================

Question: A male client is receiving chemotherapy for lung cancer. He asks the nurse how the drug will work. Which of the following is the correct response of the nurse?

A) "Chemotherapy affects all rapidly dividing cells."
B) "Structure of the DNA is altered."
C) "Chemotherapy encourages cancer cells to divide."
D) "Cancer cells have susceptible drug toxins."

Answer: A = There are many mechanisms of action for chemotherapeutic agents, but most affect the rapidly dividing cells-both cancerous and noncancerous. Cancer cells are characterized by rapid cell division. Chemotherapy slows cell division

==================================================

Question: A client will be receiving general anesthesia. The nurse reviews the laboratory result of the client and found out that the serum potassium level is 5.8 mEq/L. What should be the nurse's initial response?

A) Send the client to surgery
B) Notify the anesthesiologist
C) Call the surgeon
D) Send the client to surgery

Answer: B = the nurse should notify the anesthesiologist because a serum potassium level of 5.8 mEq/L places the client at risk for dysrhythmias when under general anesthesia.

==================================================

Question: The nurse is instructing the unlicensed assistant on how to care for a client with chest tubes that are connected to water seal drainage. Which of the following instruction would be appropriate for the nurse to give the unlicensed assistant?

A) Mark the time and amount of drainage collected in the container
B) Raise the collection apparatus to the height of the bed to measure the fluid level.
C) Milk the test tubes every 4 hours
D) Attach the chest tubes to bed linen to avoid tension of the tubing

Answer: A = It is appropriate for an unlicensed assistant to mark the time of measurement and fluid level in the collection container.

==================================================

Question: The nurse is caring for a client after a lung lobectomy. The nurse notes fluctuating water levels in the water-seal chamber of the client's chest tube. What action should the nurse take?

A. Do nothing, but continue to monitor the client.
B. Call the physician immediately.
C. Check the chest tube for a loose connection.
D. Add more water to the water-seal chamber

Answer: Correct answer: A

Fluctuation in the water-seal chamber is a normal finding that occurs as the client breathes. No action is required except for continued monitoring of the client. The nurse doesn't need to notify the physician. Continuous bubbling in the water-seal chamber indicates an air leak in the chest tube system, such as from a loose connection in the chest tube tubing. The water-seal chamber should be filled initially to the 2 cm line, and no more water should be added.

==================================================

Question: A client with type 2 diabetes has a hemoglobin A1C level of 8.8 after 6 months of oral therapy with metformin (Glucophage®). The client tells the nurse that she often forgets to take her medication and doesn't really follow her diet. Which of the following is the nurse's best first response?

A. "If you don't get control of your blood sugar, you'll need to take insulin."
B. "It can be hard to get used to having a disease like diabetes. What are some of the things you find challenging about it?"
C. "Uncontrolled diabetes can lead to eye problems and kidneys problems."
D. "Many people have diabetes."

Answer: Correct answer: B

Acknowledging that the client is going through changes and allowing her to express her concerns will help the nurse assess her needs. Hemoglobin AIC shows the average blood glucose levels over a 3-month period. Diabetes should maintain the AIC <7%. Lecturing, threatening and comparing the clients to others belittles the client and discourages discussion, but the patient must be provided adequate information in order to make informed decisions about self-care.

==================================================

Question: The nurse is teaching a client newly diagnosed with type 1 diabetes how to self-administer subcutaneous insulin injections. How does the nurse best evaluate the effectiveness of her teaching?

A. Have the client repeat the steps back to the nurse.
B. Give the client a written test on self-administration of insulin.
C. Ask the client to write out the steps for self-administration of insulin injections.
D. Ask the client to give a return demonstration of self-administration of insulin.

Answer: Correct answer: D

Asking the client to give a return demonstration of his injection technique is the best way to assess whether the client can perform the procedure. It also gives the nurse the opportunity to provide feedback. Asking the client to recite the steps, pass a written test, or write out the steps shows the nurse whether the client is able to recall the steps but doesn't show that he has the necessary motor skills or the ability to perform the procedure.

==================================================

Question: The nurse is writing the teaching plan for a client undergoing a radioactive iodine uptake test to study thyroid function. Which of the following instructions should the nurse include?

A. "You need to stay at least 4 feet (1.2 m) away from other people after the test because you'll be radioactive."
B. "You need to lie very still on a stretcher that is placed in a long tube for the scan"
C. "Don't take any iodine or thyroid medication before the test."
D. "Schedule the bone scans before your radioactive iodine uptake test."

Answer: Correct answer: C

Medications such as iodine, contrast media, and antithyroid and thyroid drugs can affect the test results and should be withheld by the client for a week or longer, as directed by the physician. During a radioactive iodine uptake test, the client receives radioactive iodine by mouth or I.V. in small doses and doesn't require isolation. During magnetic resonance imaging--not radioactive iodine uptake testing--a client needs to lie still inside a long tube. Any test, such as a bone scan, that requires iodine contrast media should be scheduled after the radioactive iodine uptake test because the iodinated contrast medium can decrease uptake.

==================================================

Question: 31. A 64-year-old patient with newly diagnosed acute myelogenous leukemia (AML) who is undergoing induction therapy with chemotherapeutic agents tells the nurse, "I feel so sick that I don't know if the treatment is worth completing." The nurse's best response to the patient is

a. "I know you feel really ill right now, but after this therapy your disease will go into a remission and you will feel normal again."
b. "Induction therapy is very aggressive and causes the most side effects, so when this phase is completed you won't feel so ill."
c. "Your type of leukemia has an 80% survival rate if aggressive therapy is started, so the effects of treatment will be worth it to you."
d. "The chemotherapy is difficult, but it is necessary to put the disease into remission and give you time to make choices about your life.

Answer: D
Rationale: AML is very aggressive, and survival after diagnosis is short without treatment. Induction therapy is followed by more chemotherapy, so the nurse should not tell the patient that he or she will feel normal or not so ill. The survival with AML is not 80%.

==================================================

Question: The nurse is assessing a patient with gastroesophageal reflux disease (GERD) who is experiencing increasing discomfort. Which patient statement indicates that additional patient education about GERD is needed?

a. "I take antacids between meals and at bedtime each night."
b. "I quit smoking several years ago, but I still chew a lot of gum."
c. "I sleep with the head of the bed elevated on 4-inch blocks."
d. "I eat small meals throughout the day and have a bedtime snack."

Answer: D

Rationale: GERD is exacerbated by eating late at night, and the nurse should plan to teach the patient to avoid eating at bedtime. The other patient actions are appropriate to control symptoms of GERD.

==================================================

Question: A patient with recurring heartburn receives a new prescription for esomeprazole (Nexium). In teaching the patient about this medication, the nurse explains that this drug

a. reduces the reflux of gastric acid by increasing the rate of gastric emptying.
b. coats and protects the lining of the stomach and esophagus from gastric acid.
c. treats gastroesophageal reflux disease by decreasing stomach acid production.
d. neutralizes stomach acid and provides relief of symptoms in a few minutes.

Answer: C
Rationale: The proton pump inhibitors decrease the rate of gastric acid secretion. Promotility drugs such as metoclopramide (Reglan) increase the rate of gastric emptying. Cryoprotective medications such as sucralfate (Carafate) protect the stomach. Antacids neutralize stomach acid and work rapidly.

==================================================

Question: A nurse is performing an initial post op assessment on a client following upper GI surgery. The client has a NG tube to low, intermittent suction. To best assess the client for the presence of bowel sounds, the nurse should:

A. place the stethoscope to the left of the umbilicus.
B. turn off the nasogastric suction.
C. use the bell of the stethoscope.
D. turn the suction on the NG tube to continuous.

Answer: B. turn off the nasogastric suction.

==================================================

Question: A nurse is caring for a client diagnosed with Chron's disease, who has undergone a barium enema that demonstrated the presence of strictures in the ileum. Based on this finding, the nurse should monitor the client closely for signs of:

A. peritonitis
B. obstruction
C. malaborsorption.
D. fluid imbalance.

Answer: B. obstruction

==================================================

Question: While conducting a home visit with a client who had a partial resection of the ileum for Chron's Disease 4 weeks previously, a nurse becomes concerned when the client states:

A. My stools float and seem to have fat in them.
B. I have gaiend 5 pounds since I left the hospital.
C. I am still avoiding milk products.
D. I only have 2 formed stools per day.

Answer: A. My stools float and seem to have fat in them.

==================================================

Question: A nurse is reviewing the history and physical of a teenager admitted to a hospital with a diagnosis of ulcerative colitis. Based on this diagnosis, which information should the nurse expect to see on this client's medical record?

A. Abdominal pain and bloody diarrhea.
B. Weight gain and elevated blood glucose.
C. Abdominal distention and hypoactive bowel sounds.
D. Heartburn and regurgitation.

Answer: A. Abdominal pain and bloody diarrhea

==================================================

Question: A RN overhears a LPN talking with a client who is being prepared for a total colectomy with the creation of an ileoanal reservoir for ulcerative colitis. To decrease the client's anxiety, the RN should intervene to clarify the information given by the LPN when the LPN is heard saying:

A. this surgery will prevent you from developing colon cancer.
B. after this surgery you will no longer have ulcerative colitis.
C. when you return from surgery you will not be able to eat solid food for several days.
D. you will have an ileostomy when you return from the surgery.

Answer: D. you will have an ileostomy when you return from the surgery.

==================================================

Question: The nurse is assessing a client 24 hours following a cholecystectomy. The nurse noted that the T tube has drained 750 mL of green-brown drainage since the surgery. Which nursing intervention is appropriate?

A. Clamp the T tube
B. Irrigate the T tube
C. Notify the physician
D. Document the findings

Answer: D. Document the findings

==================================================

Question: .The nurse is caring for a hospitalized client with a diagnosis of ulcerative colitis. Which finding, if noted on assessment of the client, would the nurse report to the physician?

A. Hypotension
B. Bloody diarrhea
C. Rebound tenderness
D. A hemoglobin level of 12 mg/dL

Answer: C. Rebound tenderness

==================================================

Question: A nurse is preparing to remove a nasogartric tube from a client. The nurse should instruct the client to do which of the following just before the nurse removes the tube?

A. Exhale
B. Inhale and exhale quickly
C. Take and hold a deep breath
D. Perform a Valsalva maneuver

Answer: C. Take and hold a deep breath

==================================================

Nclex Musculoskeletal Questions

Question: A nurse is performing a musculoskeletal assessment on an older adult living independently. What normal physiologic changes of aging does the nurse expect? (Select all that apply.)
a. Muscle atrophy
b. Slowed movement
c. Scoliosis
d. Arthritis
e. Widened gait

Answer: A,B,D,E

==================================================

Question: A client returns to the postanesthesia care unit (PACU) after an arthroscopy to prepare a knee injury. What is the nurse's priority when caring for this client?

a. Perform passive range-of-motion exercises.
b. Keep the affected leg immobilized.
c. Ensure that the patient uses the patient-controlled analgesia (PCA) pump.
d. Check the neurovascular status of the affected leg and foot.

Answer: D

==================================================

Question: A client who has a plaster leg splint reports a painful pressure sensation under the elastic wrap that is holding the splint in place. What is the nurse's best initial action?

a. Remove the splint to reduce skin pressure.
b. Perform a neurovascular assessment.
c. Report the client's concern to the primary health care provider.
d. Inspect the skin under the elastic bandage.

Answer: B.

==================================================

Question: A client has a synthetic cast placed for a right wrist fracture in the emergency room. What priority health teaching is important for the nurse to provide for this client before returning home? (Select all that apply.)

a. "Keep your right arm below the level of your heart as often as possible."
b. "Use an ice pack for the first 24 hours to decrease tissue swelling."
c. "Move the fingers of the right hand frequently to promote blood flow."
d. "Report coolness or discoloration of your right hand to your doctor."
e. "Don't place any device under the case to scratch the skin if it itches."

Answer: B,C,D,E

==================================================

Question: A client who had an elective below-the-knee amputation (BKA) reports pain in the foot that was amputated last week. What is the nurse's most appropriate response to the client's pain?

a. "The pain will go away after the swelling decreases."
b. "That's phantom limb pain and every amputee has that."
c. "Your foot has been amputated, so it's in your head."
d. "On a scale of 0 to 10, how would you rate your pain?"

Answer: D.

==================================================

Question: What is the nurse's priority when doing an admission for a client who returned directly from the operating suite after a carpal tunnel repair?

a. Monitor vital signs, including pulse oximetry.
b. Check the surgical dressing to ensure it is intact.
c. Assess neurovascular assessment in the affected arm.
d. Monitor intake and output.

Answer: A.

==================================================

Question: The nurse plans to use which tool to measure joint range of motion (ROM)?
A. Doppler device
B. Goniometer
C.Reflex hammer
D. Tonometer

Answer: B.

==================================================

Question: A nursing student is studying the skeletal system. Which statement best indicates to the nursing instructor that the student understands a normal physiologic function of the skeletal system?
A."Volkmann's canals connect osteoblasts and osteoclasts."
B. "In the deepest layer of the periosteum is the cortex, which consists of dense, compact bone tissue."
C. "The matrix of the bone is where deposits of calcium and magnesium are present."
D. "Hematopoiesis occurs in the red marrow, which is where blood cells are produced."

Answer: D.

==================================================

Question: The nurse is completing an admission assessment on a client scheduled for arthroscopic knee surgery. Which information will be most essential for the nurse to report to the health care provider?
A. Knee pain at a level of 9 (0-to-10 scale)
B. Warm, red, and swollen knee
C. Allergy to shellfish and iodine
D. Previous surgery on the other knee

Answer: B.

==================================================

Question: Which client information is most essential for the nurse to report to the health care provider before a client with knee pain undergoes magnetic resonance imaging (MRI)?
A. Daily use of aspirin
B. Swollen and tender knee
C. Presence of a permanent pacemaker
D. History of claustrophobia

Answer: C.

==================================================

Question: When assessing a female client, the nurse learns that the client has several risk factors for osteoporosis. Which risk factor will be the priority for client teaching?
A. Low calcium and vitamin D intake
B. Postmenopausal status
C. Positive family history
D. Previous use of steroids

Answer: A.

==================================================

Question: Which aspect of postoperative management will the nurse plan to discuss with a client about to undergo an arthroscopic repair of the knee?
A. Physical therapy for exercises
B. Pharmacy for client medications
C. Dietitian for nutrition
D. Social work for care coordination

Answer: A.

==================================================

Question: An older adult client with diabetes who had arthroscopic surgery on the right knee the previous day has a red, swollen, and painful right knee. The nurse anticipates that the health care provider will request which type of medication?
A.Antibiotic
B. Anticoagulant
C. Opioid analgesic
D. Corticosteroid

Answer: A.

==================================================

Question: A client is suspected of having muscular dystrophy (MD). Which laboratory test result does the nurse anticipate with this disease?
A. Decreased serum creatine kinase (CK) level
B. Moderately elevated aspartate aminotransferase (AST)
C. Decreased alkaline phosphatase (ALP)
D. Decreased skeletal muscle creatine kinase (CK-MM) level

Answer: B.

==================================================

Question: The charge nurse in the hospital-based day surgery center is making client assignments for the staff. Which client is most appropriate to assign to a nurse who has floated from the general surgical unit?
A. Young adult who has just been admitted for surgery after sustaining an ankle fracture
B. Adult who needs teaching about quadriceps-setting exercises after knee arthroscopy
C. Middle-aged adult who will require a pneumatic tourniquet applied before knee surgery
D. Older adult who has undergone arthroscopic surgery of the shoulder under local anesthesia

Answer: D.

==================================================

Question: A client is scheduled to undergo closed magnetic resonance imaging (MRI) without contrast medium. Which information does the nurse give to the client before the test?
A. "It will be important to lie still in a reclined position for 20 minutes."
B. "Do not eat or drink for 8 hours before the test."
C. "You can have the MRI if you have an internal pacemaker."
D. "All jewelry and clothing with zippers or metal fasteners must be removed."

Answer: D.

==================================================

Question: The nurse is conducting a musculoskeletal history in an older adult client who requires a caregiver to perform all activities of daily living (ADLs). Which level of functioning does the nurse record in the client's history using Gordon's Functional Health Patterns?
A.Level 0
B.Level II
C. Level III
D. Level IV

Answer: D.

==================================================

Question: The nurse is conducting a musculoskeletal history in an older adult client who requires a caregiver to perform all activities of daily living (ADLs). Which level of functioning does the nurse record in the client's history using Gordon's Functional Health Patterns?
A. Level
B. Level II
C. Level III
D. Level IV

Answer: D.

==================================================

Question: A 65-year-old female client has chronic hip pain and muscle atrophy from an arthritic disorder. Which musculoskeletal assessment finding does the nurse expect to see in the client?
A. Antalgic gait
B. Midswing gait
C. Narrow-based stance
D. No lurch in gait

Answer: A.

==================================================

Question: Care of the older adult may be affected by which physiologic change in the musculoskeletal system?
A. Regeneration of cartilage
B. Decreased range of motion (ROM)
C. Increased bone density
D. Narrower gait

Answer: B.

==================================================

Question: The nurse is attempting to perform a quick assessment of a client's hip discomfort. The client is sitting upright in a wheelchair. What is the nurses initial action?
A.Have the client flex and extend the foot on the affected side.
B.Flex and extend the client's knee to assess for discomfort.
C.Ask the client to stand from the wheelchair and transfer to the bed.
D. Perform passive abduction and adduction of the client's hips.

Answer: B.

==================================================

Question: The nurse is reviewing the medication history for a client scheduled for a left total hip replacement. The nurse plans to contact the health care provider if the client is taking which medication?
A. Acetaminophen (Tylenol) for pain relief
B. Bupropion (Wellbutrin) for smoking cessation
C. Magnesium hydroxide (Milk of Magnesia) to treat heartburn
D. Prednisone (Deltasone) to treat asthma

Answer: D.

==================================================

Question: Which diagnostic test requires the nurse to know whether the client is allergic to iodine-based contrast?
A.Arthroscopy
B.Computed tomography (CT)
C. Electromyography (EMG)
D. Tomography

Answer: B.

==================================================

Question: The nurse is using a common scale to grade a client's muscle strength. The client is able to complete range of motion (ROM) only with gravity eliminated. Which grade does the nurse document in this client's record?
A. 0
B. 1
C. 2
D. 3

Answer: C.

==================================================

Question: The ambulatory surgery post anesthesia care unit (PACU) nurse has just received report about clients who had arthroscopic surgery. Which client will the nurse plan to assess first?
A. Young adult client who has been in the PACU for 30 minutes after left knee arthroscopy under local anesthesia
B. Adult client who had a synovial biopsy of the right knee under local anesthesia and has been in the PACU for 20 minutes
C. Adult client who has multiple right knee incisions for repair of torn cartilage and arrived in the PACU an hour ago
D. Middle-aged adult client who returned to the PACU 25 minutes ago after left knee arthroscopic surgery under epidural anesthesia

Answer: D.

==================================================

Question: Which information about a client who was admitted with a pelvic fracture after being crushed by a tractor is most important for the nurse to assess to monitor for serious complications from this type of injury?
A. Skin to evaluate lacerations and abrasions.
B. Lungs for bilateral normal breath sounds
C. Pain score and level of alertness
D. Urine dipstick for the presence of red blood cells.

Answer: D.

==================================================

Question: A client with peripheral vascular disease will undergo a Syme amputation. What will the nurse teach this patient when providing education about this procedure?
A. "You will be able to bear weight without needing a prosthesis."
B. "This type of procedure results in more pain than others."
C. "The surgeon will remove both the foot and ankle."
D. "This is an above-the-knee type of amputation."

Answer: A.

==================================================

Question: The client has sustained a traumatic amputation of the left arm after a machine accident. In what order should the following nursing actions be taken? 1. Apply direct pressure to the amputated site. 2. Elevate the extremity above the client's heart. 3. Assess the client for breathing problems. 4. Examine the amputation site.
A. 2, 4, 3, 1
B. 3, 4, 1, 2
C. 1, 4, 3, 2
D. 4, 1, 2, 3

Answer: B.

==================================================

Question: A client sustains a fracture of one arm and the provider applies a plaster cast to the extremity. What will the nurse teach the client to do during the first 24 hours after discharge from the emergency department?
A. Monitor neuromuscular status for decreased circulation and sensation in the extremity.
B. Apply a heating pad for 15 to 20 minutes four times daily to help with pain.
C. Check the fit of the cast by inserting a tongue blade between the cast and the skin.
D. Keep the cast covered with a soft towel to help it to dry quickly.

Answer: A.

==================================================

Question: A client sustains a fracture of one arm and the provider applies a plaster cast to the extremity. What will the nurse teach the client to do during the first 24 hours after discharge from the emergency department?
A. Monitor neuromuscular status for decreased circulation and sensation in the extremity.
B. Apply a heating pad for 15 to 20 minutes four times daily to help with pain.
C. Check the fit of the cast by inserting a tongue blade between the cast and the skin.
D. Keep the cast covered with a soft towel to help it to dry quickly.

Answer: A.

==================================================

Question: An older adult client has had an open reduction and internal fixation of a fractured right hip. Which intervention does the nurse implement for this client?
A. Keep the client's heels off the bed at all times.
B. Reposition the client every 3 to 4 hours.
C. Administer preventive pain medication before deep-breathing exercises.
D. Prohibit the use of antiembolic stockings.

Answer: A.

==================================================

Question: A client has a grade III open fracture of the right tibia. To prevent infection, which intervention does the nurse implement?
A. Apply bacitracin (Neosporin) ointment to the site daily with a sterile cotton swab.
B. Use strict aseptic technique when cleaning the site.
C. Leave the site open to the air to keep it dry.
D. Assist the client to shower daily and pat the wound site dry.

Answer: B.

==================================================

Question: A client is in skeletal traction. Which nursing intervention ensures proper care of this client?
A. Ensure that weights are placed on the floor.
B. Ensure that pins are not loose and tighten as needed.
C. Inspect the skin at least every 8 hours.
D. Remove the traction weights only for bathing.

Answer: C.

==================================================

Question: An older adult client has multiple tibia and fibula fractures of the left extremity after a motor vehicle crash. Which pain medication does the nurse anticipate will be requested for this client?
A. Cyclobenzaprine (Flexeril)
B. Ibuprofen (Advil)
C. Meperidine (Demerol)
D. Patient-controlled analgesia (PCA) with morphine

Answer: D.

==================================================

Question: Which statement indicates to the nursing instructor that the nursing student understands the normal healing process of bone after a fracture?
A. "A callus is quickly deposited and transformed into bone."
B. "A hematoma forms at the site of the fracture."
C. "Cellular and vascular proliferation surround the fracture site."
D. "Granulation tissue reabsorbs the hematoma and deposits new bone."

Answer: B.

==================================================

Question: A rock climber has sustained an open fracture of the right tibia after a 20-foot (6 meter) fall. The nurse plans to assess the client for which potential complications?
Select all that apply.
A. Acute compartment syndrome (ACS)
B. Fat embolism syndrome (FES)
C. Congestive heart failure
D. Urinary tract infection (UTI)
E. Osteomyelitis

Answer: A,B, E

==================================================

Question: The nurse performs a neurovascular assessment on a client with closed multiple fractures of the right humerus who is experiencing increased pain even with maximum ordered doses of morphine. The nurse notes distal capillary refill of 3 seconds and coolness of the hand and fingers. The client reports numbness of the hand and is unable to wiggle the thumb. Which nursing action is indicated?
A. Elevate the extremity.
B. Apply an ice pack to the extremity.
C. Reposition the extremity and recheck in 15-20 minutes.
D. Notify the provider of these findings.

Answer: D.

==================================================

Question: A client undergoes a surgical amputation of a lower extremity after a motor vehicle crash. The client's vital signs are stable. What is a priority nursing action in the early postoperative period to help prevent complications in this client?
A. Fitting the client with a prosthetic device
B. Inspecting the limb stump daily for signs of skin breakdown
C. Positioning and range-of-motion of the affected extremity
D. Teaching the client and family how to apply shrinker stockings

Answer: C.

==================================================

Question: Which nursing action does the nurse on the orthopedic unit plan to delegate to unlicensed assistive personnel (UAP)?
A. Removing the wound drain for a client who had an open reduction of a hip fracture 3 days ago.
B. Assessing for bruising on a client who is receiving warfarin (Coumadin) to prevent deep vein thrombosis.
C. Teaching a client with a right ankle fracture how to use crutches when transferring and ambulating.
D. Checking the vital signs for a client who was admitted after a total knee replacement 3 hours ago.

Answer: D.

==================================================

Question: The nurse anticipates providing collaborative care for a client with a traumatic amputation of the right hand with which health care team members?
Select all that apply.
A. Occupational therapist
B. Physical therapist
C. Psychologist
D. Respiratory therapist
E. Speech therapist

Answer: A,B,C

==================================================

Question: Which intervention does the nurse suggest to a client who has undergone a leg amputation to help cope with loss of the limb?
A. Talking with an amputee close to the client's age who has a similar amputation
B. Drawing a picture of how the client sees him- or herself
C. Talking with a psychiatrist about the amputation
D. Engaging in diversional activities to avoid focusing on the amputation

Answer: A.

==================================================

Question: A client is recovering from an above-the-knee amputation resulting from peripheral vascular disease. Which statement indicates that the client is coping well after the procedure?
A. "My spouse will be the only person to change my dressing."
B. "I can't believe that this has happened to me. I can't stand to look at it."
C. "I do not want any visitors while I'm in the hospital."
D. "It will take me some time to get used to this."

Answer: D.

==================================================

Question: The nurse admits an older adult client who sustained a left hip fracture and is in considerable pain. The nurse anticipates that the client will be placed in which type of traction prior to surgical repair?
A. Balanced skin traction
B. Buck's traction
C. Overhead traction
D. Plaster traction

Answer: B.

==================================================

Question: A client has undergone an elective below-the-knee amputation of the right leg as a result of severe peripheral vascular disease. In postoperative care teaching, the nurse instructs the client to notify the health care provider immediately if which change occurs?
A. Observation of a large amount of serosanguineous or bloody drainage
B. Mild to moderate pain controlled with prescribed analgesics
C. Absence of erythema and tenderness at the surgical site
D. Ability to flex and extend the right knee

Answer: A.

==================================================

Question: A client with a fracture asks the nurse about the difference between an open fracture and a simple fracture. Which statement by the nurse is correct?
A. "Simple fracture involves a break in the bone, with skin contusions."
B. "An open fracture does not extend through the skin."
C. "Simple fracture has an increased risk for infection and emboli."
D. "An open fracture involves a break in the bone, with damage to the skin."

Answer: D.

==================================================

Question: A client has sustained a fracture of the left tibia. The extremity is immobilized using an external fixation device. Which postoperative instruction does the nurse include in this client's teaching plan?
A. "Use pain medication as prescribed to control pain."
B. "Clean the pin site when any drainage is noticed."
C. "Wear the same clothing that is normally worn."
D. "Apply bacitracin (Neosporin) if signs or symptoms of infection develop around pin sites."

Answer: A.

==================================================

Question: The nurse is instructing a local community group about ways to reduce the risk for musculoskeletal injury. What information does the nurse include in the teaching plan?
A. "Avoid contact sports."
B. "Avoid rigorous exercise."
C. "Wear helmets when riding a motorcycle."
D. "Avoid driving in inclement weather."

Answer: C.

==================================================

Question: A client has sustained a rotator cuff tear while playing baseball. The nurse anticipates that the client will receive which immediate conservative treatment?
A. Surgical repair of the rotator cuff
B. Prescribed exercises of the affected arm
C. Activity limitations for the affected arm
D. Patient-controlled analgesia with morphine

Answer: A.

==================================================

Question: Which signs and symptoms would be expected in a patient who has a first-degree ankle sprain?
Select all that apply.
A. Tenderness
B. Severe pain
C. Loss of function
D. Minimal swelling
E. Diminished pulses

Answer: A & D.

==================================================

Question: Which factors could increase the risk for developing compartment syndrome?
Select all that apply.
A. Edema
B. Active bleeding
C. Application of a half cast
D. Restrictive arm splint
E. Intravenous (IV) infiltration

Answer: A, B, D,E

==================================================

Question: What are the phases of compartment syndrome?

Answer: Swelling of the arm
Occlusion of the arm arteries and veins
Death of the arm tissue
Muscle and nerve damage
Eruption of fibrotic tissue

==================================================

Question: The nurse is caring for a patient who has swelling on the right forearm after a fall. Which order by the health care provider should the nurse anticipate next?
A. Obtain STAT X-ray
B. Administer intravenous morphine
C. Apply warm compress on the right forearm
D. Start intravenous (IV) infusion with normal saline

Answer: A.

==================================================

Question: When caring for a patient with right ankle swelling, the nurse notes edema, bruising, and decreased movement of the foot. Which acute actions should the nurse take?
Select all that apply.
A. Elevate the extremity
B. Assess the patient's urine output
C. Perform passive range of motion (ROM)
D. Conduct a baseline neurovascular examination
E. Educate the patient and family about strengthening exercises

Answer: A &D

==================================================

Question: A patient diagnosed with compartment syndrome begins to complain of left flank pain. The nurse notes dark red urine. Which action would the nurse take to manage this patient's symptoms?
A. Increase fluid intake
B. Provide a cold compress
C. Elevate the affected limb above the head
D. Increase traction weight on the affected limb

Answer: A.

==================================================

Question: Which signs and symptoms would the nurse find in a patient with compartment syndrome?
Select all that apply.
A. Erythema
B. 1+ pulses
C. Warmth at the affected site
D. Pain not relieved by morphine
E. Tingling sensation of the fingers

Answer: B, D, E

==================================================

Question: Which manifestation can be seen in a patient with a third-degree back strain?
A. Mild edema
B. Pain and pallor
C. Active bleeding
D. Complete loss of function

Answer: D.

==================================================

Question: Which injuries are susceptible to compartment syndrome?
Select all that apply.
A. Fractured rib
B. Orbital fracture
C. Upper arm fracture
D. Basilar skull fracture
E Third-degree ankle strain

Answer: C & E.

==================================================

Question: When caring for a patient with a right leg fracture, the patient reports pain not relieved by analgesic and tingling of the toes. The nurse also notes diminished pedal pulses and swelling above the dressing. Which action should the nurse take?
A. Loosen the dressing
B. Maintain the traction weight
C. Prepare for emergency decompression
D. Immediately inform the health care provider

Answer: A.

==================================================

Question: A patient with an upper arm fracture is diagnosed with compartment syndrome. After the health care provider cuts the cast, the nurse notes continued swelling, absent radial pulse, pallor, and cool skin. Which initial action should the nurse take?
A. Begin preoperative checklist
B. Raise the fractured arm above the head
C. Obtain samples for urinalysis and serum creatinine
D. Continue doing the neurovascular examination until patient is stable

Answer: A.

==================================================

Question: A patient presents with bilateral leg pain, abdominal pain, and difficulty breathing after a motor vehicle accident. The nurse notes obvious deformities to the bilateral lower legs, bruising to the chest, and abrasions on the chest, abdomen, and arms. Which actions should the nurse perform while assessing the severity of the patient's injuries?
Select all that apply.
A. Auscultate lung sounds
B. Assess peripheral pulses
C. Assess capillary refill time
D. Start intravenous (IV) infusion with dextrose in 5% water
E. Prepare patient for computed tomography (CT) scan

Answer: A,B,C

==================================================

Question: While caring for a patient after a third-degree ankle sprain, the nurse notes the patient can bear weight on the affected foot without discomfort. Which actions would the nurse consider?
Select all that apply.
A. Teach strengthening exercises for the ankle
B. Assess neurovascular status of affected extremity
C. Teach patient to continue applying ice packs at home
D. Check X-ray finding with the patient to see if there is no fracture
E. Perform passive range-of-motion (ROM) exercises on the affected foot

Answer: A & B

==================================================

Question: A patient presents at the emergency department and is found to have a closed fracture of the humerus. Which description is consistent with this type of fracture?
A.The skin remains intact.
B.The skin is broken while the bone is exposed.
C.The line of the fracture extends across the bone shaft.
D. The line of the fracture extends in a twisting direction.

Answer: A.

==================================================

Question: Which conditions can increase the risk for fractures?
Select all that apply.
A. Diabetes
B. Bone cyst
C.Hypertension
D.Osteoporosis
E. Metastatic cancer

Answer: B,D,E

==================================================

Question: The nurse is caring for a patient with swelling, discoloration, and obvious deformity to the right forearm after a fall. Which order should the nurse anticipate from the health care provider?
A.Administer cephalosporin
B.Prepare for STAT closed reduction
C.Obtain an X-ray of the right arm
D. Obtain a computed tomography (CT) scan of the right arm

Answer: C.

==================================================

Question: The nurse is caring for a patient with a right humeral fracture after a skating accident. Which type of splint would the nurse use for this patient?
A.Cylinder
B. Sugar tong
C. Long arm splint
D.Short arm splint

Answer: C.

==================================================

Question: After identifying Ms. Fielding's rib fractures, which potential signs and symptoms should the nurse monitor to detect related injuries?
Select all that apply.
A.Hemoptysis
B.High blood pressure
C.Low oxygen saturation
D. Right lower quadrant pain
E. Random blood sugar of 120

Answer: A & C

==================================================

Question: Ms. Fielding reports shortness of breath. The nurse notes shallow, labored breathing. Which intervention is priority for the nurse to take while Ms. Fielding is in the emergency unit?
A. Prepare patient for CT scan
B. Encourage increased oral fluids
C. Administer supplemental oxygen
D. Administer intravenous (IV) morphine

Answer: C.

==================================================

Question: The nurse is caring for a patient with a suspected arm fracture. The nurse notes an increased risk for fracture based on which patient statement?
A. "I am a strict vegetarian."
B. "I have two pet dogs at home."
C. "I use daily inhaled steroids for asthma."
D. "I worked out 3 or 4 times a week before the injury."

Answer: C.

==================================================

Question: After a motor vehicle accident, a patient presents with a deformity to the leg with decreased pedal pulses. The fibula protrudes from the lateral aspect of the leg. How should the nurse classify the fracture?
A. Open
B. Spiral
C. Closed
D. Displaced
E. Incomplete

Answer: A & D

==================================================

Question: A nurse is reviewing the medical history of a patient with a pathologic fracture of the hip. Which condition may be a cause for the fracture of this patient?
A. Breast cancer
B. Lactose intolerance
C. Rheumatoid arthritis
D. Systemic lupus erythematosus (SLE)

Answer: A.

==================================================

Question: A patient who fractured the humerus 5 weeks prior presents for a follow-up X-ray. Which result should the nurse expect to find on the X-ray?
A. Soft tissue damage
B. Fracture still evident
C. Joint union complete
D. Distance between bone fragments decreasing

Answer: B.

==================================================

Question: A patient is undergoing a noninvasive electrical bone growth stimulation on a nonunion fracture of the spine after a spinal fusion surgery that was performed 6 months ago. Which action should the nurse take?
A. Check if the electrode is implanted in the bone fragment.
B. Ensure that treatment is administered while patient is sleeping.
C. Assist patient in walking while in treatment for at least three times for 15 minutes.
D. Check if the external power supply is inserted properly through the skin and bone.

Answer: B.

==================================================

Question: The nurse is caring for a patient after an open reduction of a hip fracture that requires immobilization. Which form of immobilization would the nurse anticipate?
A. Skin traction
B. Skeletal traction
C. Internal fixation
D. External fixation

Answer: B.

==================================================

Question: The nurse is caring for a patient who reports having severe pain and decreased movement after an ankle dislocation. After verifying the prescription and patient allergies, which medication would the nurse safely administer to this patient?
A. Morphine
B. Cyclobenzaprine
C. Acetaminophen/codeine
D. Oxycodone/acetaminophen

Answer: A.

==================================================

Question: A patient with a pelvic fracture is non-weight bearing and is unable to ambulate. Which dietary modifications should the nurse anticipate for this patient?
Select all that apply.
A. Increase dietary fiber
B. Limit oral fluid intake
C. Increase iron-rich foods
D. Increase calcium-rich foods
E. Decrease protein-rich foods

Answer: A & D

==================================================

Question: The nurse is caring for a patient with a right leg deformity who reports numbness of the toes and plantar aspect of the foot. On further assessment, the nurse notes no pedal pulse and cool skin. Which action should the nurse take next after notifying the provider?
A. Prepare for reduction and splint
B. Start IV infusion with normal saline
C. Prepare the patient for a STAT X-ray
D. Elevate the right leg and apply ice pack

Answer: A.

==================================================

Question: The nurse is caring for a patient with a short leg cast who reports itching and burning inside the cast. Which actions would the nurse perform?
Select all that apply.
A. Assess capillary refill of the toes
B. Apply ice on the affected leg for 2 days
C. Immobilize joints above and below the cast
D. Report the symptoms to the health care provider
E. Teach the patient to use cool setting of hair dryer for itching

Answer: A, D, E

==================================================

Question: The nurse is caring for a patient who underwent an open hip fracture reduction. The patient reports having pain that radiates down the leg. Which actions should the nurse take?
Select all that apply.
A. Assess for muscle spasm
B. Administer pain medication
C. Massage the surrounding muscles
D. Assess vital signs and capillary refill
E. Report symptoms immediately to health care provider

Answer: A, B, D

==================================================

Question: The nurse is caring for a patient with an open arm fracture after a horse riding accident. Which actions are important for the nurse to take?
Select all that apply.
A. Splint the arm
B. Administer IV antibiotics
C. Assess capillary refill of fingers
D. Assess airway and cervical spine
E. Administer antitetanus as prescribed
F. Apply heating pads on the affected areas

Answer: B, C, D,E

==================================================

Nclex Ob

Question: A nursing student is preparing a prenatal class on the process of fetal circulation. The nursing instructor asks the student specifically to describe the process through the umbilical cord. Which of the following statements from the student is correct?

1. "The one artery carries freshly oxygenated blood and nutrient-rick blood back from the placenta to the fetus."
2. "The two arteries carry freshly oxygenated blood back from the placenta to the fetus."
3. "The two arteries in the umbilical cord carry deoxygenated blood and waste products away from the fetus to the placenta."
4. "The two veins in the umbilical cord carry blood that is high in carbon dioxide and other waste products away from the fetus to the placenta."

Answer: 3. "The two arteries in the umbilical cord carry deoxygenated blood and waste products away from the fetus to the placenta."

Rational: Blood pumped by the embryo's heart leaves the embryo through two umbilical arteries. When oxygenated, the blood is returned by one umbilical vein.

==================================================

Question: A pregnant client tells the clinic nurse that she wants to k now the sex of her baby as soon as it can be determined. The nurse understands that the client should be able to find out at 12 weeks' gestation because by the end of the twelfth week:

1. The sex of the fetus can be determined by the appearance of the external genitalia.
2. The sex of the fetus can be determined because the external genitalia begin to differentiate.
3. The sex of the fetus can be determined because the testes are descended into the scrotal sac.
4. The sex of the fetus can be determined because the internal differences in males and females becomes apparent.

Answer: 1. The sex of the fetus can be determined by the appearance of the external genitalia.

Rational: By the end of the 12th week, the external genitalia of the fetus have developed to such a degree that the sex of the fetus can be determined visually.

==================================================

Question: A nurse is performing an assessment on a client who is at 38 weeks gestation and notes that the FHR is 174/bpm. On the basis of this finding, the appropriate nursing action is to:

1. Notify the physician
2. Document the findings.
3. Check the mother's heart rate
4. Tell the client that the FHR is normal.

Answer: 1. Notify the physican

Rational: The FHR depends on gestational age and ranges from 160-170/bpm in the 1st trimester, but slows with fetal growth to 120-160/bpm near or at term. Because the FHR is increased from the reference range, the nurse should notify the physican.

==================================================

Question: A nurse is conducting a prenatal class on the female reproductive system. When a client in the class asks why the fertilized ovum stays in the Fallopian tube for 3 days, the nurse responds that the reason for this is that it:

1. Promotoes the fertilized ovum's changes of survival.
2. Promotes the fertilized ovum's exposure to estrogen & progesterone.
3. Promotes the fertilized ovum's normal implantation in the top portion of the uterus.
4. Promotes the fertilized ovum's exposure to LH and FSH

Answer: 3. Promotes the fertilized ovum's normal implantation in the top portion of the uterus.

==================================================

Question: A nursing instructor is reviewing the menstrual cycle with a nursing student who will be conducing a prenatal teaching session. The instructor asks the student to describe the FSH and LF. The student accurately responds by stating that:

1. FSH & LH are secreted by the adrenal glands.
2. FSH & LH are released from the anterior pituitary gland
3. FSH & LH are secreted by the corpus luteum of the ovary
4. FSH & LH stimulate the formation of the milk during pregnancy.

Answer: 2. FSH & LH are released from the anterior pituitary gland

==================================================

Question: A nurse explains some of the purposes of the placenta to a client during a prenatal visit. The nurse determins that the client understands some of these purposes when the client states that the placenta:

1. Cushions and protects the baby
2. Maintains the temperature of the baby
3. Is the way the baby gets food and oxygen
4. Prevents all antibodies and viruses from passing to the baby.

Answer: 3. Is the way the baby gets food and oxygen.

==================================================

Question: A nursing instructor asks a nursing student to list the functions of the amniotic fluid. The student responds correctly by stating that which of the following are functions of the amniotic fluid? SELECT ALL THAT APPLY.

1. Allows for fetal movement
2. Is a measure of kidney function
3. Surrounds, cushions, and protects the fetus
4. Maintains the body temperature of the fetus
5. Prevents large particles such as bacteria from passing to the fetus
6. Provides an exchange of nutrients and waste products between the mother and fetus

Answer: 1. Allows for fetal movement
2. Is a measure of kidney function
3. Surrounds, cushions, and protects the fetus
4. Maintains the body temperature of the fetus

==================================================

Question: A nurse is performing an assessment of a pregnant client who is at 28 wks gestation. The nurse measures the fundal height in centimeters and expects he finding to be which of the following?

1. 22 cm
2. 30 cm
3. 36 cm
4. 40 cm

Answer: 2. 30 cm

Rational: During the 2nd/3rd trimesters (18 wks-30wks), fundal height in cm approximately equals the fetus' age in weeks +/- 2cm. At 16 wks, the fundus can be located halfway between the symphysis pubis and the umbilicus. At 20-22 wks, the fundus is at the umbilicus. At 36 wks, the fundus is at the xiphoid process.

==================================================

Question: A nurse is collecting data during an admission assessment of a client who is pregnant with twins. The client has a healthy 5-year-old child who was delivered at 38 wks and tells the nurse that she does not have a history of any type of abortion or fetal demise. The nurse would document the GTPAL for this client as:

1. G3, T2, P0, A0, L1
2. G2, T1, P0, A0, L1
3. G1, T1, P1, A0, L1
4. G2, T0, P0, A0, L1

Answer: 2. G2, T1, P0, A0, L1

==================================================

Question: A pregnant client is seen in a health care clinic for a regular prenatal visit. The client tells the nurse that she is experiencing irregular contractions, and the nurse determines that she is experiencing Braxton Hicks contractions. Based on this finding, which nursing action is appropriate?

1. Contact the physician
2. Instruct the client to maintain bedrest for the remainder of the pregnancy.
3. Inform the client that these contractions are common and may occur throughout the pregnancy.
4. Call the maternity unit and inform them that the client will be admitted in a prelabor condition.

Answer: 3. Inform the client that these contractions are common and may occur throughout the pregnancy.

==================================================

Question: A nurse is providing instructions to a pregnant client with genital herpes about the measures that are needed to protect the fetus. The nurse tells the client that:

1. Total abstinence from sexual intercourse is necessary during the entire pregnancy.
2. Sitz baths need to be taken every 4 hours while awake if vaginal lesions are present.
3. Daily administration of acyclovir (Zovirax) is necessary during the entire pregnancy.
4. A cesarean section will be necessary if vaginal lesions are present at the time of labor.

Answer: 4. A cesarean section will be necessary if vaginal lesions are present at the time of labor.

==================================================

Question: A nurse is reviewing the record of a client who has just been told that a pregnancy test is positive. The physician has documented the presence of Goodell's sign. The nurse determines that this sign indicates:

1. A softening of the cervix.
2. The presence of fetal movement
3. The presence of HCG in the urine
4. A soft blowing sound that corresponds to the maternal pulse during auscultation of the uterus.

Answer: 1. A softening of the cervix.

==================================================

Question: A client arrives at the clinic for the first prenatal assessment. The client tells a nurse that the first day of her LMP was October 19, 2012. Using Nagele's rule, the nurse determines the EDD is:

1. July 12, 2012
2. July 26, 2013
3. August 12, 2013
4. August 26, 2013

Answer: 2. July 26, 2013

==================================================

Question: A nurse-midwife is assessing a pregnant client for the presence of ballottement. To make this determination, the nurse mid-wife does which of the follwoing?

1. Auscultates for fetal heart sounds
2. Assesses the cervix for compressibility
3. Palpates the abdomen for fetal movement
4. Initiates a gentle upward tap on the cervix.

Answer: 4. Initiates a gentle upward tap on the cervix.

==================================================

Question: A nurse is performing an assessment of a primigravida who is being evaluated in a clinic during her 2nd trimester of pregnancy. Which of the following indicates an ABNORMAL physical finding that necessitates further testing?

1. Quickening
2. Braxton Hicks Contractions
3. FHR of 180/bpm
4. Consistent increase in fundal height

Answer: 3. FHR of 180/bpm

==================================================

Question: A nurse is assisting in performing an assessment on a client who suspects that she is pregnant and is checking the client for PROBABLE signs of pregnancy. Which of the following are probably signs of pregnancy? SELECT ALL THAT APPLY.

1. Ballottement
2. Chadwick's Sign
3. Uterine enlargement
4. Braxton Hicks contractions
5. FHR detected by a non-electronic devise
6. Outline of fetus via radiography or ultrasonography

Answer: 1. Ballottement
2. Chadwick's Sign
3. Uterine enlargement
4. Braxton Hicks contractions

==================================================

Question: A nurse is providing instructions to a pregnant client who is scheduled for an amniocentesis. The nurse tells the client that:

1. Strict bed rest is required after the procedure.
2. An informed consent needs to be signed before the procedure
3. Hospitalization is necessary for 24 hours after the procedure
4. A fever is expected after the procedure because of the trauma to the abdomen.

Answer: 2. An informed consent needs to be signed before the procedure

==================================================

Question: A pregnant client in the 1st trimester calls a nurse at a health care clinic and reports that she has noticed a thin, colorless vaginal drainage. The nurse should make which statement to the client?

1. "Come to the clinic immediately."
2. "Report the ED at the maternity center immediately."
3. "The vaginal drainage may be bothersome, but is a normal occurrence."
4. "Use tampons if the discharge is bothersome, but to be sure to change the tampons ever 2 hours."

Answer: 3. "The vaginal drainage may be bothersome, but is a normal occurrence."

Rational: Leukorrhea begins during the 1st trimester. Many clients notice a thin, colorless or yellow vaginal discharge throughout pregnancy. Some clients become distressed about this condition, but it does not require that the client report to the clinic or ED.

==================================================

Question: A nurse has performed a nonstress test on a pregnant client and is reviewing the fetal monitor strip. The nurse interprets the test as reactive and understand that this indicates:

1. Normal findings
2. Abnormal findings
3. The need for further evaluation
4. That the findings on the monitor were difficult to interpreet

Answer: 1. Normal findings

Rational: A reactive NST is a normal result. To be considered reactive, the baseline FHR must be within normal range, 120-160/bpm, with good long-term variability. In addition, 2+ FHR accelerations of t least 15/bpm must occur, each with a duration of at least 15 sec, in a 20-minute interval.

==================================================

Question: A NST is performed on a client who is pregnant, and the results of the test indicate NONREACTIVE finding. The physician prescribes a CST, and the results are documented as NEGATIVE. A nurse interprets the finding of the CST as indicating:

1. A normal test result
2. An abnormal test result
3. A high risk for fetal demise
4. The need for a cesarean delivery

Answer: 1. A normal test result

Rational: A NEGATIVE test result indicates that no late decelerations occurred in the FHR, although the fetus was stressed by three contractions of at least 40 sec duration in a 10 minute period.

==================================================

Question: A pregnant client tells a nurse that she has been craving "unusual foods." The nurse gathers additional assessment data from the client and discovers that the client has been ingesting daily amounts of white clay dirt from her backyard. Lab studies are performed on the client. The nurse reviews the results and determines that which of the following indicates a physiological consequence of the client's practice?

1. Hematocrit 38%
2. Glucose 86 mg/dL
3. Hemoglobin 9.1 g/dL
4. WBC count 12,400/mm3

Answer: 3. Hemoglobin 9.1 g/dL

Rational: Pica cravings often lead to iron deficiency anemia, resulting in a decreased hemoglobin level.

==================================================

Question: A clinic nurse is providing instructions to pregnant client regarding measures that assist in alleviating heartburn. Which statement by the client indicates an understanding of the instructions?

1. "I should avoid between-meal snacks."
2. "I should lie down for an hour after meals."
3. "I should use spices for cooking rather than using salt."
4. I should avoid eating foods that produce gas, such as beans and some vegetables, and fatty foods such as deep-fried chicken."

Answer: 4. I should avoid eating foods that produce gas, such as beans and some vegetables, and fatty foods such as deep-fried chicken."

==================================================

Question: A home care nurse visits a pregnant client who ad a diagnosis of mild preeclampsia. Which assessment finding indicates a worsening of the preeclampsia and the need to notify the physician?

1. Urinary output as increased
2. Dependent edema has resolved
3. BP reading is at the prenatal baseline
4. The client complains of a headache and blurred vision.

Answer: 4. The client complains of a headache and blurred vision.

==================================================

Question: A nurse implements a teaching plan for a pregnant client who is newly diagnosed with GDM. Which statement made by the client indicates a need for further teaching?

1. "I should stay on the diabetic diet."
2. "I should perform glucose monitoring at home."
3. "I should avoid exercise because of the negative effects on insulin production."
4. "I should be aware of any infections and report signs of infection immediately to may HCP."

Answer: 3. "I should avoid exercise because of the negative effects on insulin production."

==================================================

Question: A pregnant client in the last trimester has been admitted to the hospital with a diagnosis of severe preeclampisa. A nurse monitors for complications associated with the diagnosis and assesses the client for.

1. Enlargement of the breast
2. Complaints of feeling hot when the room is cool
3. Periods of fetal movement followed by quiet periods
4. Evidence of bleeding, such as in the gums, petechiae, and purpura

Answer: 4. Evidence of bleeding, such as in the gums, petechiae, and purpura

Rational: Severe preeclampsia can trigger DIC because of the widespread damage to vascular integrity.

==================================================

Question: A nurse in a maternity unit is reviewing the records of the clients on the unit. Which client would the nurse identify as being at the greatest risk for developing DIC?

1. A primigravida with mild preeclampisa
2. A primigravida who delivered a 10lb infant 3 hours ago
3. A G2 who has just been diagnosed with dead futus syndrome
4. A G4 who delivered 8 hours ago and has lost 500mL of blood

Answer: 3. A G2 who has just been diagnosed with dead fetus syndrome

==================================================

Question: A client is in the 1st trimester of pregnancy arrives at a health care clinic and reports that she has been experiencing vaginal bleeding. A threatened abortion is suspected, and the nurse instructs the client regarding management of care. Which statement made by the client indicates a need for further instructions?

1. I will watch for the evidence of the passage of tissue.
2. I will maintain strict bedrest throughout the remainder of the pregnancy
3. I will count the number of perineal pads used on a daily basis and note the amount and color of blood on the pad.
4. I will avoid sexual intercourse until the bleeding as stopped, and for 2 wks following the last evidence of bleeding.

Answer: 1. I will watch for the evidence of the passage of tissue.

==================================================

Question: A home care nurse is monitoring a pregnant client with gestational HTN who is at risk for preeclampsia. At each home care visit, the nurse assess the client for which classic signs of preeclampsia? SELECT ALL THAT APPLY.

1. Proteinuria
2. HTN
3. Low grade fever
4. Generalized edema
5. Increased pulse rate
6. Increased respirator rate

Answer: 1. Proteinuria
2. HTN
4. Generalized edema

==================================================

Question: A nurse is assessing a pregant client in the 2nd trimester of pregnancy who was admitted to the maternity unit with a suspected diagnosis of abruptio placentae. Which of the following assessment findings would the nurse expect to note if this condition is present?

1. Soft abdomen
2. Uterine tenderness
3. Absence of abdominal pain
4. Painless, bright red vaginal bleeding

Answer: 2. Uterine tenderness

==================================================

Question: A maternity nurse is preparing for the admission of a client in the 3rd trimester of pregnancy who is experiencing vaginal bleeding and has a suspected diagnosis of placenta previa. The nurse review the physican's prescriptiosn and would question which prescription?

1. Prepare the client for an ultrasound
2. Obtain equipment for a manual pelvic examination
3. Prepare to draw a hemoglobin and hematocrit blood sample
4. Obtain equipment for external electronic FHR monitoring.

Answer: 2. Obtain equipment for a manual pelvic examination

==================================================

Question: An ultrasound is performed on a client at term gestation who is experiencing moderate vaginal bleeding. The results of the ultrasound indicate that abruptio placentae is present. Based on these findings, the nurse would prepare the client for:

1. Delivery of the fetus
2. Strict monitoring of I/O
3. Complete bedrest for the remainder of the pregnancy
4. The need for weekly monitoring of coagulation studies until the time of delivery

Answer: 1. Delivery of the fetus

==================================================

Question: A nurse in the postpartum unit is caring for a client who has just delivered a newborn infant following a placenta previa. The nurse reviews the plan of care and prepares to monitor the client for which risk associated with placenta previa?

1. Infection
2. Hemorrhage
3. Chronic HTN
4. DIC

Answer: 2. Hemorrhage

==================================================

Question: A nurse is performing an assessment on a client diagnosed with placenta previa. Which of these assessment findings would the nurse expect to note? SELECT ALL THAT APPLY.

1. Uterine rigidity
2. Uterine tenderness
3. Severe abdominal pain
4. Bright red vaginal bleeding
5. Soft, relaxed, nontender uterus
6. Fundal height may be greater than expected for gestational age.

Answer: 4. Bright red vaginal bleeding
5. Soft, relaxed, nontender uterus
6. Fundal height may be greater than expected for gestational age.

==================================================

Nclex Pn Readiness Test

Question: A client has arterial blood gas values: pH 7.30, PaO2 89, PaCO2 50 and HCO3 26 mEq/L. Based on these values, the nurse should suspect...

Answer: Respiratory Acidosis

==================================================

Question: A client admitted with deep vein thrombosis (DVT) is prescribed bed rest. Which action should the nurse take?

Answer: When the client has an urge to void, assist her to a sitting position.

==================================================

Question: When preparing client for bronchoscopy the nurse instructs client to avoid what?

Answer: Eating

==================================================

Question: Which of the following in an example of primary preventive measure?

Answer: Avoiding overexposure to the sun.

==================================================

Question: When caring for a client with acute respiratory failure the nurse should expect to focus on resolving which of the following problems?

Answer: Hypercapnia, hypoventilation, and hypoxemia.

==================================================

Question: What is the mechanism of action of ahydrous theophylline in treating a nonreversible obstructive airway disease such as COPD?

Answer: It makes the central respiratory center more sensitive to carbon dioxide and stimulates the respiratory drive.

==================================================

Question: A client with schizophrenia is admitted to the facility, when collecting data about the client, the nurse should document which symptoms as negative symptoms of schizophrenia?

Answer: Apathy, blunted affect, and lack of motivation.

==================================================

Question: What is the most appropriate nursing diagnosis for a client with acute pancreatitis?

Answer: Deficient fluid volume

==================================================

Question: A client has arterial blood gas values: pH 7.30; P2O2 89; PaCO2 50; and HCO3 26 mEq/L. Based on these values the nurse should suspect which?

Answer: Respiratory Acidosis

==================================================

Question: A client admitted with deep vein thrombosis is prescribed bed rest. bed rest. Which action should the nurse take?

Answer: When the client has urge to void, assist her to a sitting position.

==================================================

Question: Client with schizophrenia is taking the atypical antipsychotic medication clozapine (Clozaril). What are the signs and symptoms which indicate an adverse reaction to this med?

Answer: 1. Sore throat
2. Fever

==================================================

Question: A client with delusional thinking shows a lack of interest in eating at meal times. She states that she is unworthy of eating and that her children will die if she eats. Which nursing action would be appropriate for this client?

Answer: Restricting the client's access to food except at specified meal and snack times.

==================================================

Question: A client with constipation is prescribed an irrigating enema. What are the steps?

Answer: 1. Assist into sims
2. Lubricate the end of the catheter
3. Keep solution container below 18" above bed level

==================================================

Question: The nurse is caring for a client who had a subtotal gastrectomy 24 hours ago. The nurse should?

Answer: Irrigate the NG tube with normal saline solution.

==================================================

Question: DO NOT delegate what you can E.A.T.

Answer: E - evaluate
A - assess
T - teach

==================================================

Question: No Pee, no K

Answer: Do not give potassium without adequate urine output

==================================================

Question: The nurse is caring for a 40 yr old client admitted with an acute myocardial infarction. Which behavior by the client indicates adult cognitive development?

Answer: Generates new levels of awareness.

==================================================

Question: Addisons = down, down, down, up, down

Answer: hyponatremia, hypotension, decreased blood volume, hyperkalemia, hypoglycemia

==================================================

Question: Cushings disease = up, up, up, down, up

Answer: hypernatremia, hypertension, increased blood volume, hypokalemia, hyperglycemia

==================================================

Question: EleVate Veins; dAngle Arteries for better perfusion.

Answer: A= appearance
P= pulse (>100, <100, absent)
G= grimace (cough, grimace, no response)
A= activity (flexed, flaccid, limp)
R= respirations (strong cry, weak cry, absent)

==================================================

Question: For newborn babies APGAR

Answer: My - Measles
Chicken - Chicken Pox / Varicella
Hez - Herpes Zoster / Shingles
TB
or... MTV = Measles, TB, Varicella - chicken pox / herpes-zoster shingles
Private Room - neg pressure w 6-12 air exchanges/hr. N95 for TB

==================================================

Question: Transmission-Based Precautions: AIRBORNE

Answer: Think of SPIDERMAN
S - sepsis
S - scarlet fever
S - streptococcal pharyngitis
P - parvovirus B19
P - pneumonia
P - pertussis
I - influenza
D - diptheria
E - epiglottitis
R - rubella
M - mumps
M - meningitis
M - mycoplasma or meningeal pneumonia
An - Adenovirus Private Room or Cohort Mask

==================================================

Question: Transmission-Based Precautions: Droplet

Answer: MRS. WEE
M - multidrug resistant organism
R - respiratory infection
S - skin infections *
W - wound infxn
E - enteric infxn - clostridium difficile
E - eye infxn - conjuctivitis

==================================================

Question: CONTACT PRECAUTION

Answer: VCHIPS
V - varicella zoster
C - cutaneous diptheria
H - herpes simplex
I - impetigo
P - pediculosis
S - scabies

==================================================

Question: SKIN INFECTIONS: VCHIPS

Answer: Rectal Bleeding

==================================================

Question: Which of the following is a sign of colon cancer?

Answer: maintaining fluid balance

==================================================

Question: When planning care for a client with a small bowel obstruction the nurse should consider the primary goal to be...

Answer: 1. Explain the procedure
2. Make sure informed consent is obtained.
3. Instruct client to void.

==================================================

Question: Which nursing interventions should be performed before an abdominal paracentesis?

Answer: Administering IV fluids

==================================================

Question: The nurse caring for a client with a small-bowel obstruction would plan to implement which intervention?

Answer: Metabolism and excretion

==================================================

Question: Alterations in hepatic blood flow resulting from a drug interaction also can affect?

Answer: Atrophy of the gastric mucosa

==================================================

Question: For a client at the age of 82, which age-related change increases the risk of anemia?

Answer: Abduction

==================================================

Question: As the client moves the arm away from the midline, the nurse evaluates the ability to perform?

Answer: Store the drug in a tight light-resistant container

==================================================

Question: Which statement regarding usage of glutethimide is correct?

Answer: Joint stiffness that decreases with activity

==================================================

Question: Which of these findings best correlates with a diagnosis of osteoarthritis?

Answer: Acetylcholine

==================================================

Question: Which of the following would the nurse identify as a neurotransmitter?

Answer: 1. Right leg is shorter
2. Right leg is adducted
3. Right leg is externally rotated

==================================================

Question: A client with a suspected fracture of the right hip is in ER. Which data collection would the nurse expect to see in the clients right leg?

Answer: A safe environment

==================================================

Question: The nurse is caring for a client with delirium. Which of the following is most important for the nurse to provide the client?

Answer: ECT will induce a seizure.

==================================================

Question: Electroconvulsive therapy (ECT). When teaching the client and family about this treatment, the nurse should include which point?

Answer: Polyuria

==================================================

Question: A client with bipolar disorder is being treated with lithium for the first time. The nurse should observe the client for which early adverse effect of lithium?

Answer: Expression of fear of colorectal cancer following 3 days of constipation.

==================================================

Question: The nurse is caring for a client with hypochondriasis. Which behavior would the nurse be most likely to encounter?

Answer: Breath sounds clear on auscultation.

==================================================

Question: Which of the following outcome criteria would be most appropriate for the client with ineffective airway clearance?

Answer: Bronchospasm

==================================================

Question: Antiplatelet drugs most commonly produce which hypersensitivity reaction?

Answer: Apply acyclovir ointment every three hours, 6 times per day for 7 days

==================================================

Question: A client with primary herpes genitalis. What are the instructions?

Answer: Take on an empty stomach and maintain treatment.

==================================================

Question: What are the instructions for a 19 year old with acne vulgaris taking tretinoin (Retin-A), benzoyl peroxide and tetracycline (Achromycin)?

Answer: ...start after a known voiding that empties the bladder.

==================================================

Question: When initiating a 24 hour urine specimen, the collection time should...

Answer: 1. Smoking may lower drug level
2. Avoid prolonged sun exposure
3. Increase fluid & fiber to prevent constipation.

==================================================

Question: The nurse is teaching a client about the intidepressant amitriptyline (Elavil). Which points should the nurse be including in the teaching?

Answer: ...self-awareness and understanding.

==================================================

Question: The basis for building a strong therapeutic nurse-client relationship begins with the nurses...

Answer: Pathologic bone fractures

==================================================

Question: Clients with multiple myeloma are at risk for?

Answer: GI upset and a metallic taste

==================================================

Question: Early s/s of Vitamin D toxicity include?

Answer: ...tell the client to rest in a room set at a comfortable temperature.

==================================================

Nclex Practice Questions Cardiovascular System

Question: Which of the following actions is the first priority of care for a pt exhibiting signs & symptoms of coronary artery disease?
1. Decrease anxiety
2. Enhance myocardial oxygenation
3. Administer sublingual nitroglycerin
4. Educate the pt about his symptoms

Answer: 2. Enhancing myocardial oxygenation is always the first priority when a pt exhibits signs or symptoms of cardiac compromise. W/out adequate oxygenation, the myocardium suffers damage. Sublingual nitroglycerin is administered to treat acute angina, but administration isn't the first priority. Although educating the pt & decreasing anxiety are important in care delivery, neither are priorities when a pt is compromised.

==================================================

Question: Medical treatment of coronary artery disease includes which of the following procedures?
1. Cardiac catherization
2. Coronary artery bypass surgery
3. Oral med therapy
4. Percutaneous transluminal coronary angioplasty

Answer: 3. Oral med administration is a noninvasive, medical treatment for coronary artery disease. Cardiac catherization isn't a treatment, but a diagnostic tool. Coronary artery bypass surgery & percutaneous transluminal coronary angioplasty are invasive, surgical treatments.

==================================================

Question: Which of the following is the most common symptom of myocardial infarction (MI)?
1. Chest pain
2. Dyspnea
3. Edema
4. Palpitations

Answer: 1. The most common symptom of an MI is chest pain, resulting from deprivation of oxygen to the heart. Dyspnea is the second most common symptom, related to an increase in the metabolic needs of the body during an MI. Edema is a later sign of heart failure, often seen after an MI. Palpitations may result from reduced cardiac output, producing arrhythmias.

==================================================

Question: Which of the following symptoms is the most likely origin of pain the pt described as knifelike chest pain that increases in intensity with inspiration?
1. Cardiac
2. Gastrointestinal
3. Musculoskeletal
4. Pulmonary

Answer: 4. Pulmonary pain is generally described by these symptoms. Musculoskeletal pain only increases w/ movement. Cardiac & GI pains don't change w/ respiration.

==================================================

Question: Which of the following blood tests is most indicative of cardiac damage?
1. Lactate dehydrogenase
2. Complete blood count (CBC)
3. Troponin I
4. Creatine kinase (CK)

Answer: 3. Troponin I levels rise rapidly & are detectable w/in 1 hour of myocardial injury. Troponin I levels aren't detectable in people w/out cardiac injury. Lactate dehydrogenase (LDH) is present in almost all body tissues & not specific to heart muscle. LDH isoenzymes are useful in diagnosing cardiac injury. CBC is obtained to review blood counts, & a complete chemistry is obtained to review electrolytes. Because CK levels may rise w/ skeletal muscle injury, CK isoenzymes are required to detect cardiac injury

==================================================

Question: What is the primary reason for administering morphine to a pt with an MI?
1. To sedate the pt
2. To decrease the pt's pain
3. To decrease the pt's anxiety
4. To decrease oxygen demand on the pt's heart

Answer: 4. Morphine is administered because it decreases myocardial oxygen demand. Morphine will also decrease pain & anxiety while causing sedation, but it isn't primarily given for those reasons.

==================================================

Question: Which of the following conditions is most commonly responsible for myocardial infarction?
1. Aneurysm
2. Heart failure
3. Coronary artery thrombosis
4. Renal failure

Answer: 3. Coronary artery thrombosis causes an inclusion of the artery, leading to myocardial death. An aneurysm is an outpouching of a vessel & doesn't cause an MI. Renal failure can be associated w/ MI but isn't a direct cause. Heart failure is usually a result from an MI.

==================================================

Question: Which of the following complications is indicated by a third heart sound (S3)?
1. Ventricular dilation
2. Systemic hypertension
3. Aortic valve malfunction
4. Increased atrial contractions

Answer: 1. Rapid filling of the ventricle causes vasodilation that is auscultated as S3. Increased atrial contraction or systemic hypertension can result in a fourth heart sound. Aortic valve malfunction is heard as a murmur.

==================================================

Question: After an anterior wall myocardial infarction, which of the following problems is indicated by auscultation of crackles in the lungs?
1. Left-sided heart failure
2. Pulmonic valve malfunction
3. Right-sided heart failure
4. Tricupsid valve malfunction

Answer: 1. The left ventricle is responsible for most of the cardiac output. An anterior wall MI may result in a decrease in left ventricular function. When the left ventricle doesn't function properly, resulting in left-sided heart failure, fluid accumulates in the interstitial & alveolar spaces in the lungs & causes crackles. Pulmonic & tricuspid valve malfunction causes right sided heart failure.

==================================================

Question: What is the first intervention for a pt experiencing MI?
1. Administer morphine
2. Administer oxygen
3. Administer sublingual nitroglycerin
4. Obtain an ECG

Answer: 2. Administering supplemental oxygen to the pt is the first priority of care. The myocardium is deprived of oxygen during an infarction, so additional oxygen is administered to assist in oxygenation & prevent further damage. Morphine & nitro are also used to treat MI, but they're more commonly administered after the oxygen. An ECG is the most common diagnostic tool used to evaluate MI.

==================================================

Question: Which of the following classes of meds protects the ischemic myocardium by blocking catecholamines & sympathetic nerve stimulation?
1. Beta-adrenergic blockers
2. Calcium channel blockers
3. Narcotics
4. Nitrates

Answer: 1. Beta-adrenergic blockers work by blocking beta receptors in the myocardium, reducing the response to catecholamines & sympathetic nerve stimulation. They protect the myocardium, helping to reduce the risk of another infarction by decreasing myocardial oxygen demand. Calcium channel blockers reduce the workload of the heart by decreasing the heart rate. Narcotics reduce myocardial oxygen demand promote vasodilation, & decrease anxiety. Nitrates reduce myocardial oxygen consumption by decreasing left ventricular end-diastolic pressure (preload) & systemic vascular resistance (afterload).

==================================================

Question: What is the most common complication of an MI?
1. Cardiogenic shock
2. Heart failure
3. arrhythmias
4. Pericarditis

Answer: 3. Arrhythmias, caused by oxygen deprivation to the myocardium, are the most common complication of an MI. Cardiogenic shock, another complication of an MI, is defined as the end stage of left ventricular dysfunction. This condition occurs in approximately 15% of pts w/ MI. Because the pumping function of the heart is compromised by an MI, heart failure is the second most common complication. Pericarditis most commonly results from a bacterial or viral infection but may occur after the MI.

==================================================

Question: With which of the following disorders is jugular vein distention most prominent?
1. Abdominal aortic aneurysm
2. Heart failure
3. MI
4. Pneumothorax

Answer: 2. Elevated venous pressure, exhibited as jugular vein distention, indicates a failure of the heart to pump. JVD isn't a symptom of abdominal aortic aneurysm or pneumothorax. An MI, if severe enough, can progress to heart failure, however, in & of itself, an MI doesn't cause JVD.

==================================================

Question: Toxicity from which of the following meds may cause a pt to see a green-yellow halo around lights?
1. Digoxin
2. Furosemide (Lasix)
3. Metoprolol (Lopressor)
4. Enalapril (Vasotec)

Answer: 1. One of the most common signs of digoxin toxicity is the visual disturbance known as the "green-yellow halo sign." The other meds aren't associated w/ such an effect.

==================================================

Question: Which of the following symptoms is most commonly associated with left-sided heart failure?
1. Crackles
2. Arrhythmias
3. Hepatic engorgement
4. Hypotension

Answer: 1. Crackles in the lungs are a classic sign of left-sided heart failure. These sounds are caused by fluid backing up into the pulmonary system. Arrhythmias can be associated w/ both right- & left-sided heart failure. Left-sided heart failure causes hypertension secondary to an increased workload on the system.

==================================================

Question: In which of the following disorders would the nurse expect to assess sacral edema in a bedridden pt?
1. Diabetes
2. Pulmonary emboli
3. Renal failure
4. Right-sided heart failure

Answer: 4. The most accurate area on the body to assess dependent edema in a bed-ridden pt is the sacral area. Sacral, or dependent, edema is secondary to right-sided heart failure.

==================================================

Question: Which of the following symptoms might a pt with right-sided heart failure exhibit?
1. Adequate urine output
2. Polyuria
3. Oliguria
4. Polydipsia

Answer: 3. Inadequate deactivation of aldosterone by the liver after right-sided heart failure leads to fluid retention, which causes oliguria.

==================================================

Question: Which of the following classes of meds maximizes cardiac performance in pts with heart failure by increasing ventricular contractibility?
1. Beta-adrenergic blockers
2. Calcium channel blockers
3. Diuretics
4. Inotropic agents

Answer: 4. Inotropic agents are administered to increase the force of the heart's contractions, thereby increasing ventricular contractility & ultimately increasing cardiac output.

==================================================

Question: Stimulation of the sympathetic nervous system produces which of the following responses?
1. Bradycardia
2. Tachycardia
3. Hypotension
4. Decreased myocardial contractility

Answer: 2. Stimulation of the sympathetic nervous system causes tachycardia & increased contractility. The other symptoms listed are related to the parasympathetic nervous system, which is responsible for slowing the heart rate.

==================================================

Question: Which of the following conditions is most closely associated with weight gain, nausea, & a decrease in urine output?
1. Angina pectoris
2. Cardiomyopathy
3. Left-sided heart failure
4. Right-sided heart failure

Answer: 4. Weight gain, nausea, & a decrease in urine output are secondary effects of right-sided heart failure. Cardiomyopathy is usually identified as a symptom of left-sided heart failure. Left-sided heart failure causes primarily pulmonary symptoms rather than systemic ones. Angina pectoris doesn't cause weight gain, nausea, or a decrease in urine output.

==================================================

Question: Which of the following heart muscle diseases is unrelated to other cardiovascular disease?
1. Cardiomyopathy
2. Coronary artery disease
3. Myocardial infarction
4. Pericardial effusion

Answer: 1. Cardiomyopathy isn't usually related to an underlying heart disease such as atherosclerosis. The etiology in most cases is unknown. CAD & MI are directly related to atherosclerosis. Pericardial effusion is the escape of fluid into the pericardial sac, a condition associated w/ Pericarditis & advanced heart failure.

==================================================

Question: Which of the following types of cardiomyopathy can be associated with childbirth?
1. Dilated
2. Hypertrophic
3. Myocarditis
4. Restrictive

Answer: 1. Although the cause isn't entirely known, cardiac dilation & heart failure may develop during the last month of pregnancy or the first few months after birth. The condition may result from a preexisting cardiomyopathy not apparent prior to pregnancy. Hypertrophic cardiomyopathy is an abnormal symmetry of the ventricles that has an unknown etiology but a strong familial tendency. Myocarditis isn't specifically associated w/ childbirth. Restrictive cardiomyopathy indicates constrictive pericarditis; the underlying cause is usually myocardial.

==================================================

Question: Septal involvement occurs in which type of cardiomyopathy?
1. Congestive
2. Dilated
3. Hypertrophic
4. Restrictive

Answer: 3. In hypertrophic cardiomyopathy, hypertrophy of the ventricular septum—not the ventricle chambers—is apparent. This abnormality isn't seen in other types of cardiomyopathy.

==================================================

Question: Which of the following recurring conditions most commonly occurs in pts with cardiomyopathy?
1. Heart failure
2. Diabetes
3. MI
4. Pericardial effusion

Answer: 1. Because the structure & function of the heart muscle is affected, heart failure most commonly occurs in pts w/ cardiomyopathy. MI results from prolonged myocardial ischemia due to reduced blood flow through one of the coronary arteries. Pericardial effusion is most predominant in pts w/ pericarditis.

==================================================

Question: Dyspnea, cough, expectoration, weakness, & edema are classic signs & symptoms of which of the following conditions?
1. Pericarditis
2. Hypertension
3. MI
4. Heart failure

Answer: 4. These are the classic signs of failure. Pericarditis is exhibited by a feeling of fullness in the chest & auscultation of a pericardial friction rub. Hypertension is usually exhibited by headaches, visual disturbances, & a flushed face. MI causes heart failure but isn't related to these symptoms.

==================================================

Question: In which of the following types of cardiomyopathy does cardiac output remain normal?
1. Dilated
2. Hypertrophic
3. Obliterative
4. Restrictive

Answer: 2. Cardiac output isn't affected by hypertrophic cardiomyopathy because the size of the ventricle remains relatively unchanged. All of the rest decrease cardiac output.

==================================================

Question: Which of the following cardiac conditions does a fourth heart sound (S4) indicate?
1. Dilated aorta
2. Normally functioning heart
3. Decreased myocardial contractility
4. Failure of the ventricle to eject all of the blood during systole

Answer: 4. An S4 occurs as a result of increased resistance to ventricular filling after atrial contraction. The increased resistance is related to decreased compliance of the ventricle. A dilated aorta doesn't cause an extra heart sound, though it does cause a murmur. Decreased myocardial contractility is heard as a third heart sound. An S4 isn't heard in a normally functioning heart.

==================================================

Question: Which of the following classes of drugs is most widely used in the treatment of cardiomyopathy?
1. Antihypertensives
2. Beta-adrenergic blockers
3. Calcium channel blockers
4. Nitrates

Answer: 2. By decreasing the heart rate & contractility, beta-blockers improve myocardial filling & cardiac output, which are primary goals in the treatment of cardiomyopathy. Antihypertensives aren't usually indicated because they would decrease cardiac output in pts who are already hypotensive. Calcium channel blockers are sometimes used for the same reasons as beta-blockers; however, they aren't as effective as beta-blockers & cause increased hypotension. Nitrates aren't used because of their dilating effects, which would further compromise the myocardium.

==================================================

Question: If medical treatments fail, which of the following invasive procedures is necessary for treating cariomyopathy?
1. Cardiac catherization
2. Coronary artery bypass graft (CABG)
3. Heart transplantation
4. Intra-aortic balloon pump (IABP)

Answer: 3. The only definitive treatment for cardiomyopathy that can't be controlled medically is a heart transplant because the damage to the heart muscle is irreversible.

==================================================

Question: Which of the following conditions is associated with a predictable level of pain that occurs as a result of physical or emotional stress?
1. Anxiety
2. Stable angina
3. Unstable angina
4. Variant angina

Answer: 2. The pain of stable angina is predictable in nature, builds gradually, & quickly reaches maximum intensity. Unstable angina doesn't always need a trigger, is more intense, & lasts longer than stable angina. Variant angina usually occurs at rest—not as a result of exercise or stress.

==================================================

Question: Which of the following types of angina is most closely related with an impending MI?
1. Angina decubitus
2. Chronic stable angina
3. Noctural angina
4. Unstable angina

Answer: 4. Unstable angina progressively increases in frequency, intensity, & duration & is related to an increased risk of MI w/in 3 to 18 months.

==================================================

Question: Which of the following conditions is the predominant cause of angina?
1. Increased preload
2. Decreased afterload
3. Coronary artery spasm
4. Inadequate oxygen supply to the myocardium

Answer: 4. Inadequate oxygen supply to the myocardium is responsible for the pain accompanying angina. Increased preload would be responsible for right-sided heart failure. Decreased afterload causes increased cardiac output. Coronary artery spasm is responsible for variant angina.

==================================================

Question: Which of the following tests is used most often to diagnose angina?
1. Chest x-ray
2. Echocardiogram
3. Cardiac catherization
4. 12-lead electrocardiogram (ECG)

Answer: 4. The 12-lead ECG will indicate ischemia, showing T-wave inversion. In addition, w/ variant angina, the ECG shows ST-segment elevation. A chest x-ray will show heart enlargement or signs of heart failure, but isn't used to diagnose angina.

==================================================

Question: Which of the following results is the primary treatment goal for angina?
1. Reversal of ischemia
2. Reversal of infarction
3. Reduction of stress & anxiety
4. Reduction of associated risk factors

Answer: 1. Reversal of the ischemia is the primary goal, achieved by reducing oxygen consumption & increasing oxygen supply. An infarction is permanent & can't be reversed.

==================================================

Question: Which of the following interventions should be the first priority when treating a pt experiencing chest pain while walking?
1. Sit the pt down
2. Get the pt back to bed
3. Obtain an ECG
4. Administer sublingual nitroglycerin

Answer: 1. The initial priority is to decrease the oxygen consumption; this would be achieved by sitting the pt down. An ECG can be obtained after the pt is sitting down. After the ECGm sublingual nitro would be administered. When the pt's condition is stabilized, he can be returned to bed.

==================================================

Question: Myocardial oxygen consumption increases as which of the following parameters increase?
1. Preload, afterload, & cerebral blood flow
2. Preload, afterload, & renal blood flow
3. Preload, afterload, contractility, & heart rate.
4. Preload, afterload, cerebral blood flow, & heart rate.

Answer: 3. Myocardial oxygen consumption increases as preload, afterload, renal contractility, & heart rate increase. Cerebral blood flow doesn't directly affect myocardial oxygen consumption.

==================================================

Question: Which of the following positions would best aid breathing for a pt with acute pulmonary edema?
1. Lying flat in bed
2. Left side-lying
3. In high Fowler's position
4. In semi-Fowler's position

Answer: 3. A high Fowler's position promotes ventilation & facilitates breathing by reducing venous return. Lying flat & side-lying positions worsen the breathing & increase workload of the heart. Semi-Fowler's position won't reduce the workload of the heart as well as the Fowler's position will.

==================================================

Question: Which of the following blood gas abnormalities is initially most suggestive of pulmonary edema?
1. Anoxia
2. Hypercapnia
3. Hyperoxygenation
4. Hypocapnia

Answer: 4. In an attempt to compensate for increased work of breathing due to hyperventilation, carbon dioxide decreases, causing hypocapnea. If the condition persists, CO2 retention occurs & hypercapnia results.

==================================================

Question: Which of the following is a compensatory response to decreased cardiac output?
1. Decreased BP
2. Alteration in LOC
3. Decreased BP & diuresis
4. Increased BP & fluid retention

Answer: 4. The body compensates for a decrease in cardiac output w/ a rise in BP, due to the stimulation of the sympathetic NS & an increase in blood volume as the kidneys retain sodium & water. Blood pressure doesn't initially drop in response to the compensatory mechanism of the body. Alteration in LOC will occur only if the decreased cardiac output persists.

==================================================

Question: Which of the following actions is the appropriate initial response to a pt coughing up pink, frothy sputum?
1. Call for help
2. Call the physician
3. Start an I.V. line
4. Suction the pt

Answer: 1. Production of pink, frothy sputum is a classic sign of acute pulmonary edema. Because the pt is at high risk for decompensation, the nurse should call for help but not leave the room. The other three interventions would immediately follow.

==================================================

Question: Which of the following terms describes the force against which the ventricle must expel blood?
1. Afterload
2. Cardiac output
3. Overload
4. Preload

Answer: 1. Afterload refers to the resistance normally maintained by the aortic & pulmonic valves, the condition & tone of the aorta, & the resistance offered by the systemic & pulmonary arterioles. Cardiac output is the amount of blood expelled by the heart per minute. Overload refers to an abundance of circulating volume. Preload is the volume of blood in the ventricle at the end of diastole.

==================================================

Question: Acute pulmonary edema caused by heart failure is usually a result of damage to which of the following areas of the heart?
1. Left atrium
2. Right atrium
3. Left ventricle
4. Right ventricle

Answer: 3. The left ventricle is responsible for the majority of force for the cardiac output. If the left ventricle is damaged, the output decreases & fluid accumulates in the interstitial & alveolar spaces, causing pulmonary edema. Damage to the left atrium would contribute to heart failure but wouldn't affect cardiac output or, therefore, the onset of pulmonary edema. If the right atrium & right ventricle were damaged, right-sided heart failure would result.

==================================================

Question: An 18-year-old pt who recently had an URI is admitted with suspected rheumatic fever. Which assessment findings confirm this diagnosis?
1. Erythema marginatum, subcutaneous nodules, & fever
2. Tachycardia, finger clubbing, & a load S3
3. Dyspnea, cough, & palpitations
4. Dyspnea, fatigue, & synocope

Answer: 1. Diagnosis of rheumatic fever requires that the pt have either two major Jones criteria or one minor criterion plus evidence of a previous streptococcal infection. Major criteria include carditis, polyarthritis, Sydenham's chorea, subcutaneous nodules, & erythema maginatum (transient, nonprurtic macules on the trunk or inner aspects of the upper arms or thighs). Minor criteria include fever, arthralgia, elevated levels of acute phase reactants, & a prolonged PR-interval on ECG.

==================================================

Question: A pt admitted with angina compains of severe chest pain & suddenly becomes unresponsive. After establishing unresponsiveness, which of the following actions should the nurse take first?
1. Activate the resuscitation team
2. Open the pt's airway
3. Check for breathing
4. Check for signs of circulation

Answer: 1. Immediately after establishing unresponsiveness, the nurse should activate the resuscitation team. The next step is to open the airway using the head-tilt, chin-lift maneuver & check for breathing (looking, listening, & feeling for no more than 10-seconds). If the pt isn't breathing, give two slow breaths using a bag mask or pocket mask. Next, check for signs of circulation by palpating the carotid pulse.

==================================================

Question: A 55-year-old pt is admitted with an acute inferior-wall myocardial infarction. During the admission interview, he says he stopped taking his metoprolol (Lopressor) 5 days ago because he was feeling better. Which of the following nursing diagnoses takes priority for this pt?
1. Anxiety
2. Ineffective tissue perfusion; cardiopulmonary
3. Acute pain
4. Ineffective therapeutic regimen management

Answer: 2. MI results from prolonged myocardial ischemia caused by reduced blood flow through the coronary arteries. Therefore, the priority nursing diagnosis for this pt is Ineffective tissue perfusion (cardiopulmonary). Anxiety, acute pain, & ineffective therapeutic regimen management are appropriate but don't take priority.

==================================================

Question: A pt comes into the E.R. with acute shortness of breath & a cough that produces pink, frothy sputum. Admission assessment reveals crackles & wheezes, a BP of 85/46, a HR of 122 BPM, & a respiratory rate of 38 breaths/minute. The pt's medical history included DM, HTN, & heart failure. Which of the following disorders should the nurse suspect?
1. Pulmonary edema
2. Pneumothorax
3. Cardiac tamponade
4. Pulmonary embolus

Answer: 1. SOB, tachypnea, low BP, tachycardia, crackles, & a cough producing pink, frothy sputum are late signs of pulmonary edema.

==================================================

Question: The nurse coming on duty receives the report from the nurse going off duty. Which of the following pts should the on-duty nurse assess first?
1. The 58-year-old pt who was admitted 2 days ago with heart failure, BP of 126/76, & a RR 21 bpm
2. The 88-year-old pt with end-stage right-sided heart failure, BP of 78/50, & a DNR order.
3. The 62-year-old pt who was admitted one day ago with thrombophlebitis & receiving IV heparin.
4. A 76-year-old pt who was admitted 1 hour ago with new-onset atrial fibrillation & is receiving IV diltiazem (Cardizem).

Answer: 4. The pt w/ A-fib has the greatest potential to become unstable & is on IV med that requires close monitoring. After assessing this pt, the nurse should assess the pt w/ thrombophlebitis who is receiving a heparin infusion, & then go to the 58-year-old pt admitted 2-days ago w/ heart failure (her s/s are resolving & don't require immediate attention). The lowest priority is the 89-year-old w/ end stage right-sided heart failure, who requires time consuming supportive measures.

==================================================

Question: When developing a teaching plan for a pt with endocarditis, which of the following points is most essential for the nurse to include?
1. "Report fever, anorexia, & night sweats to the physician."
2. "Take prophylactic antibiotics after dental work & invasive procedures."
3. "Include potassium rich foods in your diet."
4. "Monitor your pulse regularly."

Answer: 1. The most essential teaching point is to report signs of relapse, such as fever, anorexia, & night sweats, to the physician. To prevent further endocarditis episodes, prophylactic antibiotics are taken before & sometimes after dental work, childbirth, or GU, GI, or gynecologic procedures. A potassium-rich diet & daily pulse monitoring aren't necessary for a pt w/ endocarditis.

==================================================

Question: A nurse is conducting a health history with a pt with a primary diagnosis of heart failure. Which of the following disorders reported by the pt is unlikely to play a role in exacerbating the heart failure?
1. Recent URI
2. Nutritional anemia
3. Peptic ulcer disease
4. A-Fib

Answer: 3. Heart failure is precipitated or exacerbated by physical or emotional stress, dysrhythmias, infections, anemia, thyroid disorders, pregnancy, Paget's disease, nutritional deficiencies (thiamine, alcoholism), pulmonary disease, & hypervolemia.

==================================================

Question: A nurse is preparing for the admission of a pt with heart failure who is being sent directly to the hospital from the physician's office. The nurse would plan on having which of the following meds readily available for use?
1. Diltiazem (Cardizem)
2. Digoxin (Lanoxin)
3. Propranolol (Inderal)
4. Metoprolol (Lopressor)

Answer: 2. Digoxin exerts a positive inotropic effect on the heart while slowing the overall rate through a variety of mechanisms. Digoxin is the med of choice to treat heart failure. Diltiazem (calcium channel blocker) & propranolol & metoprolol (beta blockers) have a negative inotropic effect & would worsen the failing heart.

==================================================

Question: A nurse caring for a pt in one room is told by another nurse that a second pt has developed severe pulmonary edema. On entering the 2nd pt's room, the nurse would expect the pt to be:
1. Slightly anxious
2. Mildly anxious
3. Moderately anxious
4. Extremely anxious

Answer: 4. Pulmonary edema causes the pt to be extremely agitated & anxious. The pt may complain of a sense of drowning, suffocation, or smothering.

==================================================

Question: A pt with pulmonary edema has been on diuretic therapy. The pt has an order for additional furosemide (Lasix) in the amount of 40 mg IV push. Knowing that the pt also will be started on Digoxin (Lanoxin), a nurse checks the pt's most recent:
1. Digoxin level
2. Sodium level
3. Potassium level
4. Creatinine level

Answer: 3. The serum potassium level is measured in the pt receiving digoxin & furosemide. Heightened digitalis effect leading to digoxin toxicity can occur in the pt w/ hypokalemia. Hypokalemia also predisposes the pt to ventricular dysrhythmias.

==================================================

Question: A pt who had cardiac surgery 24 hours ago has a urine output averaging 19 ml/hr for 2 hours. The pt received a single bolus of 500 ml of IV fluid. Urine output for the subsequent hour was 25 ml. Daily laboratory results indicate the blood urea nitrogen is 45 mg/dL & the serum creatinine is 2.2 mg/dL. A nurse interprets the pt is at risk for:
1. Hypovolemia
2. UTI
3. Glomerulonephritis
4. Acute renal failure

Answer: 4. The pt who undergoes cardiac surgery is at risk for renal injury from poor perfusion, hemolysis, low cardiac output, or vasopressor med therapy. Renal insult is signaled by decreased urine output, & increased BUN & creatinine levels. The pt may need meds such as dopamine (Intropin) to increase renal perfusion & possibly could need peritoneal dialysis or hemodialysis.

==================================================

Question: A nurse is preparing to ambulate a pt on the 3rd day after cardiac surgery. The nurse would plan to do which of the following to enable the pt to best tolerate the ambulation?
1. Encourage the pt to cough & deep breathe
2. Premedicate the pt with an analgesic
3. Provide the pt with a walker
4. Remove telemetry equipment because it weighs down the hospital gown.

Answer: 2. The nurse should encourage regular use of pain med for the first 48 to 72 hours after cardiac surgery because analgesia will promote rest, decrease myocardial oxygen consumption resulting from pain, & allow better participation in activities such as coughing, deep breathing, & ambulation. Options 1 & 3 will not help in tolerating ambulation. Removal of telemetry equipment is contraindicated unless prescribed.

==================================================

Question: A pt's electrocardiogram strip shows atrial & ventricular rates of 80 complexes per minute. The PR interval is 0.14 second, & the QRS complex measures 0.08 second. The nurse interprets this rhythm is:
1. Normal sinus rhythm
2. Sinus bradycardia
3. Sinus tachycardia
4. Sinus dysrhythmia

Answer: 1

==================================================

Question: A pt has frequent bursts of ventricular tachycardia on the cardiac monitor. A nurse is most concerned with this dysrhythmia because:
1. It is uncomfortable for the pt, giving a sense of impending doom.
2. It produces a high cardiac output that quickly leads to cerebral & myocardial ischemia.
3. It is almost impossible to convert to a normal sinus rhythm.
4. It can develop into ventricular fibrillation at any time.

Answer: 4. Ventricular tachycardia is a life-threatening dysrhythmia that results from an irritable ectopic focus that takes over as the pacemaker for the heart. The low cardiac output that results can lead quickly to cerebral & myocardial ischemia. Pt's frequently experience a feeling of impending death. Ventricular tachycardia is treated w/ antidysrhythmic meds or magnesium sulfate, cardioversion (pt awake), or defibrillation (loss of consciousness), Ventricular tachycardia can deteriorate into ventricular defibrillation at any time.

==================================================

Question: A home care nurse is making a routine visit to a pt receiving digoxin (Lanoxin) in the treatment of heart failure. The nurse would particularly assess the pt for:
1. Thrombocytopenia & weight gain
2. Anorexia, nausea, & visual disturbances
3. Diarrhea & hypotension
4. Fatigue & muscle twitching

Answer: 2. The first signs & symptoms of digoxin toxicity in adults include abdominal pain, N/V, visual disturbances (blurred, yellow, or green vision, halos around lights), bradycardia, & other dysrhythmias.

==================================================

Question: A pt with angina complains that the angina pain is prolonged & severe & occurs at the same time each day, most often in the morning, On further assessment a nurse notes that the pain occurs in the absence of precipitating factors. This type of anginal pain is best described as:
1. Stable angina
2. Unstable angina
3. Variant angina
4. Nonanginal pain

Answer: 3. Stable angina is induced by exercise & is relieved by rest or nitroglycerin tablets. Unstable angina occurs at lower & lower levels of activity & rest, is less predictable, & is often a precursor of myocardial infarction. Variant angina, or Prinzmetal's angina, is prolonged & severe & occurs at the same time each day, most often in the morning.

==================================================

Question: The physician orders continuous intravenous nitroglycerin infusion for the pt with MI. Essential nursing actions include which of the following?
1. Obtaining an infusion pump for the med
2. Monitoring BP q4h
3. Monitoring urine output hourly
4. Obtaining serum potassium levels daily

Answer: 1. IV nitro infusion requires an infusion pump for precise control of the med. BP monitoring would be done w/ a continuous system, & more frequently than every 4 hours. Hourly urine outputs are not always required. Obtaining serum potassium levels is not associated w/ nitroglycerin infusion.

==================================================

Question: Aspirin is administered to the pt experiencing an MI because of its:
1. Antipyrectic action
2. Antithrombotic action
3. Antiplatelet action
4. Analgesic action

Answer: 2. Aspirin does have antipyretic, antiplatelet, & analgesic actions, but the primary reason ASA is administered to the pt experiencing an MI is its antithrombotic action.

==================================================

Question: Which of the following is an expected outcome for a pt on the second day of hospitalization after an MI?
1. Has severe chest pain
2. Can identify risks factors for MI
3. Agrees to participate in a cardiac rehabilitation walking program
4. Can perform personal self-care activities without pain

Answer: 4. By day 2 of hospitalization after an MI, pts are expected to be able to perform personal care w/out chest pain. Day 2 hospitalization may be too soon for pts to be able to identify risk factors for MI or begin a walking program; however, the pt may be sitting up in a chair as part of the cardiac rehabilitation program. Severe chest pain should not be present.

==================================================

Question: Which of the following reflects the principle on which a pt's diet will most likely be based during the acute phase of MI?
1. Liquids as ordered
2. Small, easily digested meals
3. Three regular meals per day
4. NPO

Answer: 2. Recommended dietary principles in the acute phase of MI include avoiding large meals because small, easily digested foods are better digested foods are better tolerated. Fluids are given according to the pt's needs, & sodium restrictions may be prescribed, especially for pts w/ manifestations of heart failure. Cholesterol restrictions may be ordered as well. Pts are not prescribed a diet of liquids only or NPO unless their condition is very unstable.

==================================================

Question: An older, sedentary adult may not respond to emotional or physical stress as well as a younger individual because of:
1. Left ventricular atrophy
2. Irregular heartbeats
3. peripheral vascular occlusion
4. Pacemaker placement

Answer: 1. In older adults who are less active & do not exercise the heart muscle, atrophy can result. Disuse or deconditioning can lead to abnormal changes in the myocardium of the older adult. As a result, under sudden emotional or physical stress, the left ventricle is less able to respond to the increased dem&s on the myocardial muscle.

==================================================

Question: Which of the following nursing diagnoses would be appropriate for a pt with heart failure? Select all that apply.
1. Ineffective tissue perfusion R/T decreased peripheral blood flow secondary to decreased CI
2. Activity intolerance R/T increased cardiac output.
3. Decreased cardiac output R/T structural & functional changes.
4. Impaired gas exchange R/T decreased sympathetic nervous system activity.

Answer: 1 & 3. HF is a result of structural & functional abnormalities of the heart tissue muscle. The heart muscle becomes weak & does not adequately pump the blood out of the chambers. As a result, blood pools in the left ventricle & backs up into the left atrium, & eventually into the lungs. Therefore, greater amounts of blood remain in the ventricle after contraction thereby decreasing cardiac output. In addition, this pooling leads to thrombus formation & ineffective tissue perfusion because of the decrease in blood flow to the other organs & tissues of the body. Typically, these pts have an ejection fraction of less than 50% & poorly tolerate activity. Activity intolerance is related to a decrease, not increase, in cardiac output. Gas exchange is impaired. However, the decrease in cardiac output triggers compensatory mechanisms, such as an increase in sympathetic nervous system activity.

==================================================

Question: Which of the following would be a priority nursing diagnosis for the pt with heart failure & pulmonary edema?
1. Risk for infection related to stasis of alveolar secretions
2. Impaired skin integrity related to pressure
3. Activity intolerance related to pump failure
4. Constipation related to immobility

Answer: 3. Activity intolerance is a primary problem for pts w/ heart failure & pulmonary edema. The decreased cardiac output associated w/ heart failure leads to reduced oxygen & fatigue. Pts frequently complain of dyspnea & fatigue. The pt could be at risk for infection related to stasis of secretions or impaired skin integrity related to pressure. However, these are not the priority nursing diagnoses for the pt w/ HF & pulmonary edema, nor is constipation related to immobility.

==================================================

Question: Captopril may be administered to a pt with HF because it acts as a:
1. Vasopressor
2. Volume expander
3. Vasodilator
4. Potassium-sparing diuretic

Answer: 3. ACE inhibitors have become the vasodilators of choice in the pt w/ mild to severe HF. Vasodilator drugs are the only class of drugs clearly shown to improve survival in overt heart failure.

==================================================

Question: Furosemide is administered intravenously to a pt with HF. How soon after administration should the nurse begin to see evidence of the drugs desired effect?
1. 5 to 10 min
2. 30 to 60 min
3. 2 to 4 hours
4. 6 to 8 hours

Answer: 1. After IV injection of furosemide, diuresis normally begins in about 5 minutes & reaches its peak w/in about 30 minutes. Med effects last 2 - 4 hours.

==================================================

Question: Which of the following foods should the nurse teach a pt with heart failure to avoid or limit when following a 2-gram sodium diet?
1. Apples
2. Tomato juice
3. Whole wheat bread
4. Beef tenderloin

Answer: 2. Canned foods & juices, such as tomato juice, are typically high in sodium & should be avoided in a sodium-restricted diet. BRING ON THE STEAK!

==================================================

Question: The nurse finds the apical pulse below the 5th intercostal space. The nurse suspects:
1. Left atrial enlargement
2. Left ventricular enlargement
3. Right atrial enlargement
4. Right ventricular enlargement

Answer: 2. A normal apical impulse is found under over the apex of the heart & is typically located & auscultated in the left fifth intercostal space in the midclavicular line. An apical impulse located or auscultated below the fifth intercostal space or lateral to the midclavicular line may indicate left ventricular enlargement.

==================================================

Nclex Practice Questions For Fundamentals Of Nursing

Question: A 73-year-old patient who sustained a right hip fracture in a fall requests pain medication from the nurse. Based on his injury, which type of pain is this patient most likely experiencing?
1) Phantom
2) Visceral
3) Deep somatic
4) Referred

Answer: Answer:
3) Deep somatic

Rationale:
Deep somatic pain originates in ligaments, tendons, nerves, blood vessels, and bones. Therefore, a hip fracture causes deep somatic pain. Phantom pain is pain that is perceived to originate from a part that was removed during surgery. Visceral pain is caused by deep internal pain receptors and commonly occurs in the abdominal cavity, cranium, and thorax. Referred pain occurs in an area that is distant to the original site.

==================================================

Question: Which pain management task can the nurse safely delegate to nursing assistive personnel?
1) Asking about pain during vital signs
2) Evaluating the effectiveness of pain medication
3) Developing a plan of care involving nonpharmacologic interventions
4) Administering over-the-counter pain medications

Answer: Answer:
1) Asking about pain during vital signs

Rationale:
The nurse can delegate the task of asking about pain when nursing assistive personnel (NAP) obtain vital signs. The NAP must be instructed to report findings to the nurse without delay. The nurse should evaluate the effectiveness of pain medications and develop the plan of care. Administering over-the-counter and prescription medications is the responsibility of the registered nurse or licensed practical nurse.

==================================================

Question: Which factor in the patient's past medical history dictates that the nurse exercise caution when administering acetaminophen (Tylenol)?

1) Hepatitis B
2) Occasional alcohol use
3) Allergy to aspirin
4) Gastric irritation with bleeding

Answer: Answer:
1) Hepatitis B

Rationale:
Even in recommended doses, acetaminophen can cause severe hepatotoxicity in patients with liver disease, such as hepatitis B. Patients who consume alcohol regularly should also use acetaminophen cautiously. Those allergic to aspirin or other nonsteroidal anti-inflammatory drugs (NSAIDs) can use acetaminophen safely. Acetaminophen rarely causes gastrointestinal (GI) problems; therefore, it can be used for those with a history of gastric irritation and bleeding.

==================================================

Question: Which action should the nurse take before administering morphine 4.0 mg intravenously to a patient complaining of incisional pain?

1) Assess the patient's incision.
2) Clarify the order with the prescriber.
3) Assess the patient's respiratory status.
4) Monitor the patient's heart rate.

Answer: Answer:
3) Assess the patient's respiratory status.

Rationale:
Before administering an opioid analgesic, such as morphine, the nurse should assess the patient's respiratory status because opioid analgesics can cause respiratory depression. It is not necessary to clarify the order with the physician because morphine 4 mg IV is an appropriate dose. It is not necessary to monitor the patient's heart rate.

==================================================

Question: Which action should the nurse take when preparing patient-controlled analgesia for a postoperative patient?

1) Caution the patient to limit the number of times he presses the dosing button.
2) Ask another nurse to double-check the setup before patient use.
3) Instruct the patient to administer a dose only when experiencing pain.
4) Provide clear, simple instructions for dosing if the patient is cognitively impaired.

Answer: Answer:
2) Ask another nurse to double-check the setup before patient use.

Rationale:
As a safeguard to reduce the risk for dosing errors, the nurse should request another nurse to double-check the setup before patient use. The nurse should reassure the patient that the pump has a lockout feature that prevents him from overdosing even if he continues to push the dose administration button. The nurse should also instruct the patient to administer a dose before potentially painful activities, such as walking. Patient-controlled analgesia is contraindicated for those who are cognitively impaired.

==================================================

Question: The nurse administers codeine sulfate 30 mg orally to a patient who underwent craniotomy 3 days ago for a brain tumor. How soon after administration should the nurse reassess the patient's pain?

1) Immediately
2) In 10 minutes
3) In 15 minutes
4) In 60 minutes

Answer: Answer:
4) In 60 minutes

Rationale:
Codeine administered by the oral route reaches peak concentration in 60 minutes; therefore, the nurse should reassess the patient's pain 60 minutes after administration. The nurse should reassess pain after 10 minutes when administering codeine by the intramuscular or subcutaneous routes. Drugs administered by the intravenous (IV) route are effective almost immediately; however, codeine is not recommended for IV administration.

==================================================

Question: Which nonsteroidal anti-inflammatory drug might be administered to inhibit platelet aggregation in a patient at risk for thrombophlebitis?

1) Ibuprofen (Motrin)
2) Celecoxib (Celebrex)
3) Aspirin (Ecotrin)
4) Indomethacin (Indocin)

Answer: Answer:
3) Aspirin (Ecotrin)

Rationale:
Aspirin is a unique NSAID that inhibits platelet aggregation. Low-dose aspirin therapy is commonly administered to decrease the risk of thrombophlebitis, myocardial infarction, and stroke. Ibuprofen, celecoxib, and indomethacin are NSAIDs, but they do not inhibit platelet aggregation.

==================================================

Question: A client who is receiving epidural analgesia complains of nausea and loss of motor function in his legs. The nurse obtains his blood pressure and notes a drop in his blood pressure from the previous reading. Which complication is the patient most likely experiencing?

1) Infection at the catheter insertion site
2) Side effect of the epidural analgesic
3) Epidural catheter migration
4) Spinal cord damage

Answer: Answer:
3) Epidural catheter migration

Rationale:
The patient is exhibiting signs of epidural catheter migration, which include nausea, a decrease in blood pressure, and loss of motor function without an identifiable cause. Signs of infection at the catheter site include redness, swelling, and drainage. Loss of motor function is not a typical side effect associated with epidural analgesics. These are common signs of catheter migration, not spinal cord damage.

==================================================

Question: Which of the following clients is experiencing an abnormal change in vital signs? A client whose (select all that apply):

1) Blood pressure (BP) was 132/80 mm Hg sitting and is 120/60 mm Hg upon standing
2) Rectal temperature is 97.9°F in the morning and 99.2°F in the evening
3) Heart rate was 76 before eating and is 60 after eating
4) Respiratory rate was 14 when standing and is 22 after walking

Answer: Answer:
1) Blood pressure (BP) was 132/80 mm Hg sitting and is 120/60 mm Hg upon standing
3) Heart rate was 76 before eating and is 60 after eating

Rationale:
The BP change is abnormal; a BP change greater than 10 mm Hg may indicate postural hypotension. The change in heart rate is abnormal; heart rate usually increases slightly after eating rather than decreasing. The temperatures are within normal range for the rectal route, and temperature increases throughout the day. It is normal to have an increased respiratory rate after exercise.

==================================================

Question: The nurse assesses clients' breath sounds. Which one requires immediate medical attention? A client who has:

1) Crackles
2) Rhonchi
3) Stridor
4) Wheezes

Answer: Answer:
3) Stridor

Rationale:
Stridor is a sign of respiratory distress, possibly airway obstruction. Crackles and rhonchi indicate fluid in the lung; wheezes are caused by narrowing of the airway. Crackles, rhonchi, and wheezes indicate respiratory illness and are potentially serious but do not necessarily indicate respiratory distress that requires immediate medical attention.

==================================================

Question: The nurse assesses the client's pedal pulses as having a pulse volume of 1 on a scale of 0 to 3. Based on this assessment finding, it would be important for the nurse to also assess the:

1) Pulse deficit
2) Blood pressure
3) Apical pulse
4) Pulse pressure

Answer: Answer:
2) Blood pressure

Rationale:
If the leg pulses are weak, the nurse should assess the blood pressure in order to further explore the reason for the low pulse volume. If the blood pressure is low, then a low pulse volume would be expected. The pulse deficit is the difference between the apical and radial pulse. The apical pulse would not be helpful to assess peripheral circulation. The pulse pressure is the difference between the systolic and diastolic pressures.

==================================================

Question: Which of the following clients has indications of orthostatic hypotension? A client whose blood pressure is:

1) 118/68 when standing and 110/72 when lying down
2) 140/80, HR 82 bpm when sitting and 136/76, HR 98 bpm when standing
3) 126/72 lying down and 133/80 when sitting, and reports shortness of breath
4) 146/88 when lying down and 130/78 when standing, and reports feeling dizzy

Answer: Answer:
4) 146/88 when lying down and 130/78 when standing, and reports feeling dizzy

Rationale:
Orthostatic hypotension is a drop of 10 mm Hg or more in blood pressure upon moving to a standing position, with complaints of feeling dizzy and/or faint.

==================================================

Question: A client who has experienced prolonged exposure to the cold is admitted to the hospital. Which method of taking a temperature would be most appropriate for this client?

1) Axillary with an electronic thermometer
2) Oral with a glass thermometer
3) Rectal with an electronic thermometer
4) Tympanic with an infrared thermometer

Answer: Answer:
3) Rectal with an electronic thermometer

Rationale:
The rectal route is the most accurate for assessing core temperature, especially when it is critical to get an accurate temperature. Therefore, in this situation it is preferred. Temperature is a particularly relevant data point for this client with hypothermia as it indicates the patient's baseline status and response to treatment. The electronic thermometer is safer than glass and is relatively accurate. Mercury thermometers are no longer used in the hospital setting. The accuracy of tympanic thermometers is debatable.

==================================================

Question: Which of the following clients would have the most difficulty maintaining thermoregulation?

1) Young child playing soccer during the summer
2) Middle-aged adult snow skiing
3) Young adult playing golf on a hot day
4) Older adult raking leaves on a cold day

Answer: Answer:
4) Older adult raking leaves on a cold day

Rationale:
Older adults have more difficulty maintaining body heat because of their slower metabolism, loss of subcutaneous fat, and decreased vasomotor control.

==================================================

Question: Which of the following clients should have an apical pulse taken? A client who is:

1) Febrile and has a radial pulse of 100 bpm
2) A runner who has a radial pulse of 62 bpm
3) An infant with no history of cardiac defect
4) An elderly adult who is taking antianxiety medication

Answer: Answer:
3) An infant with no history of cardiac defect

Rationale:
An apical pulse should be taken if the radial pulse is weak and/or irregular, if the rate is <60 or >100, if the patient is on cardiac medications, or when assessing children up to 3 years. It is difficult to palpate a peripheral pulse on infants and young children.

==================================================

Question: Which situation requires intrapersonal communication?

1) Staff meetings
2) Positive self-talk
3) Shift report
4) Wound care committee meeting

Answer: Answer:
2) Positive self-talk

Rationale:
The nurse engaging in positive self-talk is using intrapersonal communication—conscious internal dialogue. Staff meetings, shift report, and a committee meeting are all examples of group or interpersonal communication.

==================================================

Question: The nurse suspects that a patient is being physically abused at home. What is the best environment in which to discuss the possibility of abusive events?

1) The patient's shared semiprivate room
2) The hallway outside the patient's room
3) An empty corner at the nurse's station
4) A conference room at the end of the hall

Answer: Answer:
4) A conference room at the end of the hall

Rationale:
The best environment in which to discuss sensitive matters is a quiet room where conversation can occur in private, particularly when the space is nonthreatening. The patient might be distracted if conversation takes place in a room where others (e.g., patients and visitors) are present. The hallway outside the patient's room and the nurses' station are public areas and should not be used for private conversation.

==================================================

Question: A patient is admitted to the medical surgical floor with a kidney infection. The nurse introduces herself to the patient and begins her admission assessment. Which goal is most appropriate for this phase of the nurse-patient relationship? The patient will be able to:

1) Describe how to operate the bed and call for the nurse.
2) Discuss communication patterns and roles within the family.
3) Openly express his concerns about the hospitalization.
4) State expectations related to discharge.

Answer: Answer:
1) Describe how to operate the bed and call for the nurse.

Rationale:
This is the orientation phase of the relationship. The orientation phase begins when the nurse introduces herself to the patient and begins to gather data. In this phase, the nurse and patient are getting to know each other. As part of the orientation phase, the nurse will orient the patient to the hospital room and routines. In the preinteraction phase, the nurse gathers information about the patient before she meets him. Discussion of personal information, particularly if sensitive or complex, is suitable for the working phase of the nurse-patient interaction. The patient expressing feelings and concerns also occurs during the working phase. During the working phase, care is communicated, thoughts and feelings are expressed, and honest verbal and nonverbal communication occurs. Stating expectations related to discharge is most appropriate for the termination phase—the conclusion of the relationship.

==================================================

Question: A local church organizes a group for people who are having difficulty coping with the death of a loved one. Which type of group has been organized?

1) Work-related social support group
2) Therapy group
3) Task group
4) Community committee

Answer: Answer:
2) Therapy group

Rationale:
Therapy groups are designed to help individual members cope with issues, such as the death of a spouse, divorce, or motherhood. Work-related social support groups help members of a profession cope with work-associated stress. Task groups meet to accomplish a specified task. Community-based committees meet to discuss community issues.

==================================================

Question: A mother comes to the emergency department after receiving a phone call informing her that her son was involved in a motor vehicle accident. When she approaches the triage desk, she frantically asks, "How is my son?" Which response by the nurse is best?

1) "He's being examined now; he's awake and talking. We'll take you to see him soon."
2) "Don't worry, I'm sure he'll be fine; we have an excellent trauma team caring for him."
3) "Everything will be okay; please take a seat and I'll check on him for you."
4) "Your son is strong and has youth on his side; I'm sure he'll be fine."

Answer: Answer:
1) "He's being examined now; he's awake and talking. We'll take you to see him soon."

Rationale:
By telling the mother that her son is awake and talking and being examined by the doctor, the nurse provides accurate information and helps reduce the mother's anxiety. Responses such as "Don't worry, everything will be okay" and "I'm sure he'll be fine" offer false reassurance and fail to respect the mother's concern.

==================================================

Question: During a presentation at a nursing staff meeting, the unit manager speaks very slowly with a monotone. She uses medical and technical terminology to convey her message. Dressed in business attire, the manager stands erect and smiles occasionally while speaking. Which elements of her approach are likely to cause the staff to lose interest in what she has to say? Select all answers that apply.

1) Slow speech
2) Monotone
3) Occasional smile
4) Formal dress

Answer: Answer:
1) Slow speech
2) Monotone

Rationale:
Speaking slowly with a monotone can contribute to reduced attention as the listener can think faster than the speaker is speaking, and the monotone voice has an almost hypnotizing effect. Smiling improves personal interest and connection between the speaker and listener so should not cause a loss of interest. Wearing formal business attire would not directly detract from listeners' engagement in the speaker's message unless it was unusual enough to distract listeners; nothing in the situation above indicates that is so.

==================================================

Question: Which factor(s) in the patient's past medical history place(s) him at risk for falling? Select all that apply.

1) Orthostatic hypotension
2) Appendectomy
3) Dizziness
4) Hyperthyroidism

Answer: Answer:
1) Orthostatic hypotension
3) Dizziness

Rationale:
Orthostatic hypotension, cognitive impairment, difficulty with walking or balance, weakness, dizziness, and drowsiness from certain medications place the patient at risk for falling. A history of right appendectomy and hyperthyroidism do not place that patient at risk for falling.

==================================================

Question: The nurse is teaching a child and family about firearm safety. The nurse should instruct the child to take which step first if he sees a gun at a friend's house?

1) Leave the area.
2) Do not touch the gun.
3) Stop where he is.
4) Tell an adult.

Answer: Answer:
3) Stop where he is.

Rationale:
The child should be instructed to stop where he is. This allows him to think about the next steps he has memorized. Next, he should avoid touching the gun, leave the area, and immediately go tell an adult.

==================================================

Question: A patient is agitated and continues to try to get out of bed. The nurse tries unsuccessfully to reorient him. What should the nurse do next?

1) Apply a vest restraint.
2) Move the patient to a quieter room.
3) Ask another nurse to care for the patient.
4) Provide comfort measures.

Answer: Answer:
4) Provide comfort measures.

Rationale:
Patients sometimes become agitated because they are uncomfortable or in pain. Providing comfort measures may decrease agitation. If the patient continues to be agitated, the nurse should encourage a family member or friend to sit with the patient. Applying a physical restraint should be kept as a last resort for use only when less restrictive measures fail. The patient should be placed in a room near the nurses' station so he can be checked frequently if there is no one available to provide one-on-one supervision. A quieter room would probably not help.

==================================================

Question: While teaching a health promotion group of adults, the nurse notices one person who is clutching his throat with both hands. What should the nurse do first?

1) Call 9-1-1.
2) Encourage the person to cough vigorously.
3) Ask, "Are you choking?"
4) Give five back blows.

Answer: Answer:
3) Ask, "Are you choking?"

Rationale:
Clutching the throat is the universal sign of choking. The first action when you suspect airway obstruction is to ask, "Are you choking?" If the person indicates "yes," or if the person cannot cough, speak, or breathe, that indicates choking. You must first be certain the person is choking because you can cause harm when you perform the choking maneuver. You would not call 9-1-1, encourage coughing, or give five back blows until you first establish that the person is choking. The client appears to be giving the universal sign for choking, but the nurse must validate the client's meaning before acting.

==================================================

Question: What should parents do to promote child safety in the home?

1) Attach the baby's pacifier to a ribbon so that it does not fall on the floor.
2) Give a 2-year-old whole grapes instead of popcorn for a snack.
3) Store firearms unloaded and out of sight in a location too high for the child to reach.
4) Install window guards; never leave a window wide open.

Answer: Answer:
4) Install window guards; never leave a window wide open.
Rationale:
To prevent falls, install window guards and never leave a window wide open. A ribbon can become entangled around a small child's neck, causing asphyxiation. Young children can easily choke on a grape. Firearms should be unloaded, but stored in a locked cabinet. Children are curious and like to explore and climb. It would not be too difficult for a child to find a firearm stored, for example, on a high closet shelf.

==================================================

Question: What is the leading cause of unintentional death for the entire U.S. population?

1) Motor vehicle accidents
2) Poisoning
3) Choking
4) Falls

Answer: Answer:
1) Motor vehicle accidents

Rationale:
The leading causes of unintentional death for the total population, in this order, are automobile accidents, poisoning, falls, and drowning.

==================================================

Question: Which change in hygiene practices may be necessary as the patient ages?

1) Brushing teeth twice a day
2) Bathing every other day
3) Decreasing moisturizer use
4) Increasing soap use

Answer: Answer:
2) Bathing every other day

Rationale:
As a person ages, sebaceous glands become less active, causing skin to dry. Older people may find it necessary to bathe every 2 days, increase the use of moisturizers, and decrease soap use to prevent further drying of skin. Older adults should brush their teeth after every meal and at bedtime to prevent tooth decay. It is recommended that people of all ages brush their teeth at least twice a day, so that option does not represent a change in an older adult's hygiene practices.

==================================================

Question: A woman of Orthodox Jewish faith who underwent a hysterectomy for cancer is being cared for on the surgical floor. Which healthcare team member(s) could be assigned to bathe this patient? Choose all correct answers.

1) Male nursing assistant
2) Male licensed practical nurse
3) Female graduate nurse
4) Female registered nurse

Answer: Answer:
3) Female graduate nurse
4) Female registered nurse

Rationale:
Orthodox Judaism prohibits personal care being provided by a member of the opposite sex. The patient who underwent a hysterectomy is female; therefore, out of respect for her religious beliefs, she should not be bathed by the male licensed practical nurse or nursing assistant.

==================================================

Question: A 75-year-old patient who is 5 feet 7 inches tall and weighs 170 pounds is admitted with dehydration. A nursing diagnosis of Risk for Impaired Skin Integrity is identified for this patient. Which factor places the client at Risk for Impaired Skin Integrity?

Answer: Answer:
Dehydration

Rationale:
Dehydration places the patient at risk for impaired skin integrity. Dehydration, caused by fluid volume deficit, causes the skin to become dry and crack easily, impairing skin integrity. People who are very thin or very obese are more likely to experience impaired skin integrity. This patient is of normal height and weight; therefore, his body stature does not place him at risk. There is nothing to suggest that this patient has an impaired nutritional status.

==================================================

Question: The nurse notes a lesion that appears to be caused by tissue compression on the right hip of a patient who suffered a stroke 5 days ago. How should the nurse document this finding?

1) Maceration
2) Abrasion
3) Excoriation
4) Pressure ulcer

Answer: Answer:
4) Pressure ulcer

Rationale:
The nurse should document a lesion caused by tissue compression and inadequate perfusion as a pressure ulcer. Abrasion, a rubbing away of the epidermal layer of skin, is commonly caused by shearing forces that occur when a patient moves or is moved in bed. Maceration is a softening of skin from prolonged moisture. Excoriation is a loss of the superficial layers of the skin caused by the digestive enzymes in feces.

==================================================

Question: The charge nurse asks the nursing assistive personnel (NAP) to give a bag bath to a patient with end-stage chronic obstructive pulmonary disease. How should the NAP proceed?

1) Bathe the patient's entire body using 8 to 10 washcloths.
2) Assist the patient to a chair and provide bathing supplies.
3) Saturate a towel and blanket in a plastic bag, and then bathe the patient.
4) Assist the patient to the bathtub and provide a bath chair.

Answer: Answer:
1) Bathe the patient's entire body using 8 to 10 washcloths.

Rationale:
A towel bath is a modification of the bed bath in which the NAP places a large towel and a bath blanket into a plastic bag, saturates them with a commercially prepared mixture of moisturizer, nonrinse cleaning agent, and water; warms in them in a microwave, and then uses them to bathe the patient. A bag bath is a modification of the towel bath, in which the NAP uses 8 to 10 washcloths instead of a towel or blanket. Each part of the patient's body is bathed with a fresh cloth. A bag bath is not given in a chair or in the tub.

==================================================

Question: For a morbidly obese patient, which intervention should the nurse choose to counteract the pressure created by the skin folds?

1) Cover the mattress with a sheepskin.
2) Keep the linens wrinkle free.
3) Separate the skin folds with towels.
4) Apply petrolatum barrier creams.

Answer: Answer:
2) Keep the linens wrinkle free.

Rationale:
Separating the skin folds with towels relieves the pressure of skin rubbing on skin. Sheepskins are not recommended for use at all. Petrolatum barrier creams are used to minimize moisture caused by incontinence.

==================================================

Question: A client exhibits all of the following during a physical assessment. Which of these is considered a primary defense against infection?

1) Fever
2) Intact skin
3) Inflammation
4) Lethargy

Answer: Answer:
2) Intact skin

Rationale:
Intact skin is considered a primary defense against infection. Fever, the inflammatory response, and phagocytosis (a process of killing pathogens) are considered secondary defenses against infection.

==================================================

Question: A client with a stage 2 pressure ulcer has methicillin-resistant Staphylococcus aureus (MRSA) cultured from the wound. Contact precautions are initiated. Which rule must be observed to follow contact precautions?

1) A clean gown and gloves must be worn when in contact with the client.
2) Everyone who enters the room must wear a N-95 respirator mask.
3) All linen and trash must be marked as contaminated and send to biohazard waste.
4) Place the client in a room with a client with an upper respiratory infection.

Answer: Answer:
1) A clean gown and gloves must be worn when in contact with the client.

Rationale:
A clean gown and gloves must be worn when any contact is anticipated with the client or with contaminated items in the room. A respirator mask is required only with airborne precautions, not contact precautions. All linen must be double-bagged and clearly marked as contaminated. The client should be placed in a private room or in a room with a client with an active infection caused by the same organism and no other infections.

==================================================

Question: A client requires protective isolation. Which client can be safely paired with this client in a client-care assignment? One

1) admitted with unstable diabetes mellitus.
2) who underwent surgical repair of a perforated bowel.
3) with a stage 3 sacral pressure ulcer.
4) admitted with a urinary tract infection.

Answer: Answer:
1) admitted with unstable diabetes mellitus.

Rationale:
The client with unstable diabetes mellitus can safely be paired in a client-care assignment because the client is free from infection. Perforation of the bowel exposes the client to infection requiring antibiotic therapy during the postoperative period. Therefore, this client should not be paired with a client in protective isolation. A client in protective isolation should not be paired with a client who has an open wound, such as a stage 3 pressure ulcer, or with a client who has a urinary tract infection.

==================================================

Question: Which action demonstrates a break in sterile technique?

1) Remaining 1 foot away from nonsterile areas
2) Placing sterile items on the sterile field
3) Avoiding the border of the sterile drape
4) Reaching 1 foot over the sterile field

Answer: Answer:
4) Reaching 1 foot over the sterile field

Rationale:
Reaching over the sterile field while wearing sterile garb breaks sterile technique. While observing sterile technique, healthcare workers should remain 1 foot away from nonsterile areas while wearing sterile garb, place sterile items needed for the procedure on the sterile drape, and avoid coming in contact with the 1-inch border of the sterile drape.

==================================================

Question: A mother who breastfeeds her child passes on which antibody through breast milk?
1) IgA
2) IgE
3) IgG
4) IgM

Answer: Answer:
3) IgG

Rationale:
The antibody IgG is passed to the child through the mother's breast milk during breastfeeding. IgA, IgE, and IgM are produced by the child's body after exposure to an antigen.

==================================================

Question: What is the rationale for hand washing? Hand washing is expected to remove:

1) transient flora from the skin.
2) resident flora from the skin.
3) all microorganisms from the skin.
4) media for bacterial growth.

Answer: Answer:
1) transient flora from the skin.

Rationale:
There are two types of normal flora: transient and resident. Transient flora are normal flora that a person picks up by coming in contact with objects or another person (e.g., when you touch a soiled dressing). You can remove these with hand washing. Resident flora live deep in skin layers where they live and multiply harmlessly. They are permanent inhabitants of the skin and cannot usually be removed with routine hand washing. Removing all microorganisms from the skin (sterilization) is not possible without damaging the skin tissues. To live and thrive in humans, microbes must be able to use the body's precise balance of food, moisture, nutrients, electrolytes, pH, temperature, and light. Food, water, and soil that provide these conditions may serve as nonliving reservoirs. Hand washing does little to make the skin uninhabitable for microorganisms, except perhaps briefly when an antiseptic agent is used for cleansing.

==================================================

Question: Which of the following incidents requires the nurse to complete an occurrence report?

1) Medication given 30 minutes after scheduled dose time
2) Patient's dentures lost after transfer
3) Worn electrical cord discovered on an IV infusion pump
4) Prescription without the route of administration

Answer: Answer:
2) Patient's dentures lost after transfer

Rationale:
You would need to complete an occurrence report if you suspect your patient's personal items to be lost or stolen. A medication can be administered within a half-hour of the administration time without an error in administration; therefore, an occurrence report is not necessary. The worn electrical cord should be taken out of use and reported to the biomedical department. The nurse should seek clarification if the provider's order is missing information; an occurrence report is not necessary.

==================================================

Question: The nurse is orienting a new nurse to the unit and reviews source-oriented charting. Which statement by the nurse best describes source-oriented charting? Source-oriented charting:

1) Separates the health record according to discipline
2) Organizes documentation around the patient's problems
3) Highlights the patient's concerns, problems, and strengths
4) Is designed to streamline documentation

Answer: Answer:
1) Separates the health record according to discipline

Rationale:
In source-oriented charting, each discipline documents findings in a separately labeled section of the chart. Problem-oriented charting organizes notes around the patient's problems. Focus® charting highlights the patient's concerns, problems, and strengths. Charting by exception is a unique charting system designed to streamline documentation.

==================================================

Question: When the nurse completes the patient's admission nursing database, the patient reports that he does not have any allergies. Which acceptable medical abbreviation can the nurse use to document this finding?

1) NA
2) NDA
3) NKA
4) NPO

Answer: Answer:
3) NKA

Rationale:
The nurse can use the medical abbreviation NKA, which means no known allergies, to document this finding. NA is an abbreviation for not applicable. NDA is an abbreviation for no known drug allergies. NPO is an abbreviation that means nothing by mouth.

==================================================

Question: The nurse is working on a unit that uses nursing assessment flow sheets. Which statement best describes this form of charting? Nursing assessment flow sheets:

1) Are comprehensive charting forms that integrate assessments and nursing actions
2) Contain only graphic information, such as I&O, vital signs, and medication administration
3) Are used to record routine aspects of care; they do not contain assessment data
4) Contain vital data collected upon admission, which can be compared with newly collected data

Answer: Answer:
1) Are comprehensive charting forms that integrate assessments and nursing actions

Rationale:
Nursing assessment flow sheets are organized by body systems. The nurse checks the box corresponding to the current assessment findings. Nursing actions, such as wound care, treatments, or IV fluid administration, are also included. Graphic information, such as vital signs, I&O, and routine care, may be found on the graphic record. The admission form contains baseline information.

==================================================

Question: At the end of the shift, the nurse realizes that she forgot to document a dressing change that she performed for a patient. Which action should the nurse take?

1) Complete an occurrence report before leaving.
2) Do nothing; the next nurse will document it was done.
3) Write the note of the dressing change into an earlier note.
4) Make a late entry as an addition to the narrative notes.

Answer: Answer:
4) Make a late entry as an addition to the narrative notes.

Rationale:
If the nurse fails to make an important entry while charting, she should make a late entry as an addition to the narrative notes. An occurrence report is not necessary in this case. If documentation is omitted, there is no legal verification that the procedure was performed. It is illegal to add to a chart entry that was previously documented. The nurse can only document care directly performed or observed. Therefore, the nurse on the incoming shift would not record the wound change as performed.

==================================================

Question: The client asks the nurse why an electronic health record (EHR) system is being used. Which response by the nurse indicates an understanding of the rationale for an EHR system?

1) It includes organizational reports of unusual occurrences that are not part of the client's record.
2) This type of system consists of combined documentation and daily care plans.
3) It improves interdisciplinary collaboration that improves efficiency in procedures.
4) This type of system tracks medication administration and usage over 24 hours.

Answer: Answer:
3) It improves interdisciplinary collaboration that improves efficiency in procedures.

Rationale:
The EHR has several benefits for use, including improving interdisciplinary collaboration and making procedures more accurate and efficient. An occurrence report is an organizational record of an unusual occurrence or accident that is not a part of the client's record. Integrated plans of care (IPOC) are a combined charting and care plan format. A medication administration record (MAR) is used to document medications administered and their usage.

==================================================

Question: In the United States, the first programs for training nurses were affiliated with:

1) The military
2) General hospitals
3) Civil service
4) Religious orders

Answer: Answer:
4) Religious orders

Rationale:
When the Civil War broke out, the Army used nurses who had already been trained in religious orders. Although the Army did provide some training, it occurred later than in the religious orders. Although nurses were trained in hospitals, the training and the hospitals were affiliated with religious orders. Civil service was not mentioned in Chapter 1 and was not a factor in the early 1800s. Nursing started with religious orders. The Hindu faith was the first to write about nursing. In the United States, all training for nurses was affiliated with religious orders until after the Civil War.

==================================================

Question: Which of the following is/are an example(s) of a health restoration activity? Select all that apply.

1) Administering an antibiotic every day
2) Teaching the importance of hand washing
3) Assessing a client's surgical incision
4) Advising a woman to get an annual mammogram after age 50 years

Answer: Answer:
1) Administering an antibiotic every day
3) Assessing a client's surgical incision

Rationale:
Health restoration activities help an ill client return to health. This would include taking an antibiotic every day and assessing a client's surgical incision. Hand washing and mammograms both involve healthy people who are trying to prevent illness.

==================================================

Question: Which of the following aspects of nursing is essential to defining it as both a profession and a discipline?

1) Established standards of care
2) Professional organizations
3) Practice supported by scientific research
4) Activities determined by a scope of practice

Answer: Answer:
3) Practice supported by scientific research

Rationale:
The American Nurses Association (ANA) has developed standards of care, but they are unrelated to defining nursing as a profession or discipline. Having professional organizations is not included in accepted characteristics of either a profession or a discipline. A profession must have knowledge that is based on technical and scientific knowledge. The theoretical knowledge of a discipline must be based on research, so both are scientifically based. Having a scope of practice is not included in accepted characteristics of either a profession or a discipline.

==================================================

Question: The charge nurse on the medical surgical floor assigns vital signs to the nursing assistive personnel (NAP) and medication administration to the licensed vocational nurse (LVN). Which nursing model of care is this floor following?

1) Team nursing
2) Case method nursing
3) Functional nursing
4) Primary nursing

Answer: Answer:
3) Functional nursing

Rationale:
With team nursing, an RN or LVN is paired with a NAP. The pair is then assigned to render care for a group of patients. In case method nursing, one nurse cares for one patient during her entire shift. Private duty nursing is an example of this care model. This medical surgical floor is following the functional nursing model of care, in which care is partitioned and assigned to a staff member with the appropriate skills. For example, the NAP is assigned vital signs, and the LVN is assigned medication administration. When the primary nursing model is utilized, one nurse manages care for a group of patients 24 hours a day, even though others provide care during part of the day.

==================================================

Question: A patient who suffered a stroke has difficulty swallowing. Which healthcare team member should be consulted to assess the patient's risk for aspiration?

1) Respiratory therapist
2) Occupational therapist
3) Dentist
4) Speech therapist

Answer: Answer:
4) Speech therapist

Rationale:
Respiratory therapists provide care for patients with respiratory disorders. Occupational therapists help patients regain function and independence. Dentists diagnose and treat dental disorders. Speech and language therapists provide assistance to clients experiencing swallowing and speech disturbances. They assess the risk for aspiration and recommend a treatment plan to reduce the risk.

==================================================

Question: Which of the following is/are an example(s) of theoretical knowledge as defined in this chapter? Select all that apply.

1) Antibiotics are ineffective in treating viral infections.
2) When you take a patient's blood pressure, the patient's arm should be at heart level.
3) In Maslow's framework, physical needs are most basic.
4) When drawing medication out of a vial, inject air into the vial first.

Answer: Answer:
1) Antibiotics are ineffective in treating viral infections.
3) In Maslow's framework, physical needs are most basic.

Rationale:
Theoretical knowledge consists of research findings, facts (e.g., "Antibiotics are ineffective . . ." is a fact), principles, and theories (e.g., "In Maslow's framework . . ." is a statement from a theory). Instructions for taking a blood pressure and withdrawing medications are examples of practical knowledge—what to do and how to do it.

==================================================

Question: Critical thinking and the nursing process have which of the following in common? Both:

1) Are important to use in nursing practice
2) Use an ordered series of steps
3) Are patient-specific processes
4) Were developed specifically for nursing

Answer: Answer:
1) Are important to use in nursing practice

Rationale:
Nurses make many decisions: some require using the nursing process, whereas others are not client related but require critical thinking. The nursing process has specific steps; critical thinking does not. Neither is linear. Critical thinking applies to any discipline.

==================================================

Question: In which step of the nursing process does the nurse analyze data and identify client problems?

1) Assessment
2) Diagnosis
3) Planning outcomes
4) Evaluation

Answer: Answer:
2) Diagnosis

Rationale:
In the assessment phase, the nurse gathers data from many sources for analysis in the diagnosis phase. In the diagnosis phase, the nurse identifies the client's health status. In the planning outcomes phase, the nurse formulates goals and outcomes. In the evaluation phase, which occurs after implementing interventions, the nurse gathers data about the client's responses to nursing care to determine whether client outcomes were met.

==================================================

Question: In which phase of the nursing process does the nurse decide whether her actions have successfully treated the client's health problem?

Answer: Answer:
Evaluation

Rationale:
In the assessment phase, the nurse gathers data from many sources for analysis in the diagnosis phase. In the diagnosis phase, the nurse identifies the client's health status. In the planning outcomes phase, the nurse and client decide on goals they want to achieve. In the intervention planning phase, the nurse identifies specific interventions to help achieve the identified goal. During the implementation phase, the nurse carries out the interventions or delegates them to other health care team members. During the evaluation phase, the nurse judges whether her actions have been successful in treating or preventing the identified client health problem.

==================================================

Question: What is the most basic reason that self-knowledge is important for nurses? Because it helps the nurse to:

1) Identify personal biases that may affect his thinking and actions
2) Identify the most effective interventions for a patient
3) Communicate more efficiently with colleagues, patients, and families
4) Learn and remember new procedures and techniques

Answer: Answer:
1) Identify personal biases that may affect his thinking and actions

Rationale:
The most basic reason is that self-knowledge directly affects the nurse's thinking and the actions he chooses. Indirectly, thinking is involved in identifying effective interventions, communicating, and learning procedures. However, because identifying personal biases affects all the other nursing actions, it is the most basic reason.

==================================================

Question: Arrange the steps of the nursing process in the sequence in which they generally occur. A. Assessment
B. Evaluation
C. Planning outcomes
D. Planning interventions
E. Diagnosis

1) E, B, A, D, C
2) A, B, C, D, E
3) A, E, C, D, B
4) D, A, B, E, C

Answer: Answer:
3) A, E, C, D, B

Rationale:
Logically, the steps are assessment, diagnosis, planning outcomes, planning interventions, and evaluation. Keep in mind that steps are not always performed in this order, depending on the patient's needs, and that steps overlap.

==================================================

Question: How are critical thinking skills and critical thinking attitudes similar? Both are:

1) Influences on the nurse's problem solving and decision making
2) Like feelings rather than cognitive activities
3) Cognitive activities rather than feelings
4) Applicable in all aspects of a person's life

Answer: Answer:
1) Influences on the nurse's problem solving and decision making

Rationale:
Cognitive skills are used in complex thinking processes, such as problem solving and decision making. Critical thinking attitudes determine how a person uses her cognitive skills. Critical thinking attitudes are traits of the mind, such as independent thinking, intellectual curiosity, intellectual humility, and fair-mindedness, to name a few. Critical thinking skills refer to the cognitive activities used in complex thinking processes. A few examples of these skills involve recognizing the need for more information, recognizing gaps in one's own knowledge, and separating relevant from irrelevant data. Critical thinking, which consists of intellectual skills and attitudes, can be used in all aspects of life.

==================================================

Question: The nurse is preparing to admit a patient from the emergency department. The transferring nurse reports that the patient with chronic lung disease has a 30+ year history of tobacco use. The nurse used to smoke a pack of cigarettes a day at one time and worked very hard to quit smoking. She immediately thinks to herself, "I know I tend to feel negatively about people who use tobacco, especially when they have a serious lung condition; I figure if I can stop smoking, they should be able to. I must remember how physically and psychologically difficult that is, and be very careful not to let be judgmental of this patient." This best illustrates:

1) Theoretical knowledge
2) Self-knowledge
3) Using reliable resources
4) Use of the nursing process

Answer: Answer:
2) Self-knowledge

Rationale:
Personal knowledge (2) is self-understanding—awareness of one's beliefs, values, biases, and so on. That best describes the nurse's awareness that her bias can affect her patient care. Theoretical knowledge consists of information, facts, principles, and theories in nursing and related disciplines; it consists of research findings and rationally constructed explanations of phenomena. Using reliable resources is a critical thinking skill. The nursing process is a problem-solving process consisting of the steps of assessing, diagnosing, planning outcomes, planning interventions, implementing, and evaluating. The nurse has not yet met this patient, so she could not have begun the nursing process.

==================================================

Question: Which organization's standards require that all patients be assessed specifically for pain?

1) American Nurses Association (ANA)
2) State nurse practice acts
3) National Council of State Boards of Nursing (NCSBN)
4) The Joint Commission

Answer: Answer:
4) The Joint Commission

Rationale:
The Joint Commission has developed assessment standards, including that all clients be assessed for pain. The ANA has developed standards for clinical practice, including those for assessment, but not specifically for pain. State nurse practice acts regulate nursing practice in individual states. The NCSBN asserts that the scope of nursing includes a comprehensive assessment but does not specifically include pain.

==================================================

Question: Which of the following is an example of data that should be validated?

1) The urinalysis report indicates there are white blood cells in the urine.
2) The client states she feels feverish; you measure the oral temperature at 98°F.
3) The client has clear breath sounds; you count a respiratory rate of 18.
4) The chest x-ray report indicates the client has pneumonia in the right lower lobe.

Answer: Answer:
2) The client states she feels feverish; you measure the oral temperature at 98°F.

Rationale:
Validation should be done when subjective and objective data do not make sense. For instance, it is inconsistent data when the patient feels feverish and you obtain a normal temperature. The other distractors do not offer conflicting data. Validation is not usually necessary for laboratory test results.

==================================================

Question: Which of the following is an example of appropriate behavior when conducting a client interview?

1) Recording all the information on the agency-approved form during the interview
2) Asking the client, "Why did you think it was necessary to seek health care at this time?"
3) Using precise medical terminology when asking the client questions
4) Sitting, facing the client in a chair at the client's bedside, using active listening

Answer: Answer:
4) Sitting, facing the client in a chair at the client's bedside, using active listening

Rationale:
Active listening should be used during an interview. The nurse should face the patient, have relaxed posture, and keep eye contact. Asking "why" may make the client defensive. Note-taking interferes with eye contact. The client may not understand medical terminology or health care jargon.

==================================================

Question: The nurse wishes to identify nursing diagnoses for a patient. She can best do this by using a data collection form organized according to (select all that apply):

1) A body systems model
2) A head-to-toe framework
3) Maslow's hierarchy of needs
4) Gordon's functional health patterns

Answer: Answer:
3) Maslow's hierarchy of needs
4) Gordon's functional health patterns

Rationale:
Nursing models produce a holistic database that is useful in identifying nursing rather than medical diagnoses. Body systems and head-to-toe are not nursing models, and they are not holistic; they focus on identifying physiological needs or disease. Maslow's hierarchy is not a nursing model, but it is holistic, so it is acceptable for identifying nursing diagnoses. Gordon's functional health patterns are a nursing model.

==================================================

Question: The nurse is recording assessment data. She writes, "The patient seems worried about his surgery. Other than that, he had a good night." Which errors did the nurse make? Select all that apply.

1) Used a vague generality
2) Did not use the patient's exact words
3) Used a "waffle" word (e.g., appears)
4) Recorded an inference rather than a cue

Answer: Answer:
1) Used a vague generality
3) Used a "waffle" word (e.g., appears)
4) Recorded an inference rather than a cue

Rationale:
The nurse recorded a vague generality: "he has had a good night." The nurse did not use the patient's exact words, but she did not quote the patient at all, so that is not one of her errors. The nurse used the "waffle" word, "seems" worried instead of documenting what the patient said or did to lead her to that conclusion. The nurse recorded these inferences: worried and had a good night.

==================================================

Question: A patient is admitted with shortness of breath, so the nurse immediately listens to his breath sounds. Which type of assessment is the nurse performing?
1) Ongoing assessment
2) Comprehensive physical assessment
3) Focused physical assessment
4) Psychosocial assessment

Answer: Answer:
3) Focused physical assessment

Rationale:
The nurse is performing a focused physical assessment, which is done to obtain data about an identified problem, in this case shortness of breath. An ongoing assessment is performed as needed, after the initial data are collected, preferably with each patient contact. A comprehensive physical assessment includes an interview and a complete examination of each body system. A psychosocial assessment examines both psychological and social factors affecting the patient. The nurse conducting a psychosocial assessment would gather information about stressors, lifestyle, emotional health, social influences, coping patterns, communication, and personal responses to health and illness, to name a few aspects.

==================================================

Question: The nurse is assessing vital signs for a patient just admitted to the hospital. Ideally, and if there are no contraindications, how should the nurse position the patient for this portion of the admission assessment?
1) Sitting upright
2) Lying flat on the back with knees flexed
3) Lying flat on the back with arms and legs fully extended
4) Side-lying with the knees flexed

Answer: Answer:
1) Sitting upright

Rationale:
If the patient is able, the nurse should have the patient sit upright to obtain vital signs in order to allow the nurse to easily access the anterior and posterior chest for auscultation of heart and breath sounds. It allows for full lung expansion and is the preferred position for measuring blood pressure. Additionally, patients might be more comfortable and feel less vulnerable when sitting upright (rather than lying down on the back) and can have direct eye contact with the examiner. However, other positions can be suitable when the patient's physical condition restricts the comfort or ability of the patient to sit upright.

==================================================

Question: For all body systems except the abdomen, what is the preferred order for the nurse to perform the following examination techniques?
A. Palpation
B. Auscultation
C. Inspection
D. Percussion
1) D, B, A, C
2) C, A, D, B
3) B, C, D, A
4) A, B, C, D

Answer: Answer:
2) C, A, D, B

Rationale:
Inspection begins immediately as the nurse meets the patient, as she observes the patient's appearance and behavior. Observational data are not intrusive to the patient. When performing assessment techniques involving physical touch, the behavior, posture, demeanor, and responses might be altered. Palpation, percussion, and auscultation should be performed in that order, except when performing an abdominal assessment. During abdominal assessment, auscultation should be performed before palpation and percussion to prevent altering bowel sounds.

==================================================

Question: The nurse is assessing a patient admitted to the hospital with rectal bleeding. The patient had a hip replacement 2 weeks ago. Which position should the nurse avoid when examining this patient's rectal area?
1) Sims'
2) Supine
3) Dorsal recumbent
4) Semi-Fowler's

Answer: Answer:
1) Sims'

Rationale:
Sims' position is typically used to examine the rectal area. However, the position should be avoided if the patient has undergone hip replacement surgery The patient with a hip replacement can assume the supine, dorsal recumbent, or semi-Fowler's positions without causing harm to the joint. Supine position is lying on the back facing upward. The patient in dorsal recumbent is on his back with knees flexed and soles of feet flat on the bed. In semi-Fowler's position, the patient is supine with the head of the bed elevated and legs slightly elevated.

==================================================

Question: How should the nurse modify the examination for a 7-year-old child?
1) Ask the parents to leave the room before the examination.
2) Demonstrate equipment before using it.
3) Allow the child to help with the examination.
4) Perform invasive procedures (e.g., otoscopic) last.

Answer: Answer:
2) Demonstrate equipment before using it.

Rationale:
The nurse should modify his examination by demonstrating equipment before using it to examine a school-age child. The nurse should make sure parents are not present during the physical examination of an adolescent, but they usually help younger children feel more secure. The nurse should allow a preschooler to help with the examination when possible, but not usually a school-age child. Toddlers are often fearful of invasive procedures, so those should be performed last in this age group. It is best to perform invasive procedures last for all age groups; therefore, this does not represent a modification.

==================================================

Question: The nurse must examine a patient who is weak and unable to sit unaided or to get out of bed. How should she position the patient to begin and perform most of the physical examination?
1) Dorsal recumbent
2) Semi-Fowler's
3) Lithotomy
4) Sims'

Answer: Answer:
2) Semi-Fowler's

Rationale:
If a patient is unable to sit up, the nurse should place him lying flat on his back, with the head of the bed elevated. Dorsal recumbent position is used for abdominal assessment if the patient has abdominal or pelvic pain. The patient in dorsal recumbent is on his back with knees flexed and soles of feet flat on the bed. Lithotomy position is used for female pelvic examination. It is similar to dorsal recumbent position, except that the patient's legs are well separated and thighs are acutely flexed. Feet are usually placed in stirrups. Fold sheet or bath blanket crosswise over thighs and legs so that genital area is easily exposed. Keep patient covered as much as possible. The patient in Sim's position is on left side with right knee flexed against abdomen and left knee slightly flexed. Left arm is behind body; right arm is placed comfortably. Sims' position is used to examine the rectal area. In semi-Fowler's position, the patient is supine with the head of the bed elevated and legs slightly elevated.

==================================================

Question: The nurse should use the diaphragm of the stethoscope to auscultate which of the following?
1) Heart murmurs
2) Jugular venous hums
3) Bowel sounds
4) Carotid bruits

Answer: Answer:
3) Bowel sounds

Rationale:
The bell of the stethoscope should be used to hear low-pitched sounds, such as murmurs, bruits, and jugular hums. The diaphragm should be used to hear high-pitched sounds that normally occur in the heart, lungs, and abdomen.

==================================================

Question: The nurse calculates a body mass index (BMI) of 18 for a young adult woman who comes to the physician's office for a college physical. This patient is considered:
1) Obese
2) Overweight
3) Average
4) Underweight

Answer: Answer:
4) Underweight

Rationale:
For adults, BMI should range between 20 and 25; BMI less than 20 is considered underweight; BMI 25 to 29.9 is overweight; and BMI greater than 30 is considered obese.

==================================================